search and seizure cases

100
HILARIO CAMINO MONCADO, recurrente, vs. EL TRIBUNAL DEL PUEBLO Y JUAN M. LADAW, como Procurador As we understand it, the main, if not the sole, purpose of our constitutional inhibitions against unreasonable searches and seizures, was to place a salutary restriction upon the powers of government. That is to say, we believe the framers of the constitutions of the United States and of this and other states merely sought to provide against any attempt, by legislation or otherwise, to authorize, justify, or declare lawful, any unreasonable search or seizure. This wise restriction was intended to operate upon legislative bodies, so as to render ineffectual any effort to legalize by statute what the people expressly stipulated could in no event be made lawful; upon executives, so that no law violative of this constitutional inhibition should ever be enforced; and upon the judiciary, so as to render it the duty of the courts to denounce as unlawful every unreasonable search and seizure, whether confessedly without any color of authority, or sought to be justified under the guise of legislative sanction. For the misconduct of private persons, acting upon their individual responsibility and of their own volition, surely none of the three divisions of government is responsible. If an official, or a mere petty agent of the state, exceeds or abuses the authority with which he is clothed, he is to be deemed as acting, not for the state, but for himself only; and therefore he alone, and not the state, should be held accountable for his acts. If the constitutional rights of a citizen are invaded by a mere individual, the most that any branch of government can do is to afford the citizen such redress as a possible, and bring the wrongdoer to account for his unlawful conduct. . . . . It is contemplated, and such ought ever to be the fact, that the records of courts remains permanently in the places assigned by the law for their custody. It does not logically follow, however, that the records, being obtained, cannot be used as instruments of evidence; for the mere fact of (illegally) obtaining them does not change that which is written in them ... . Suppose the presence of a witness to have been procured by fraud or violence, while the party thus procuring the attendance of the witness would be liable to severe punishment, surely that could not be urged against the competency of the witness. If he could not, why shall a record, although illegally taken from its proper place of custody and brought before the Court, but otherwise free from suspicion, be hold incompetent? Admitting that the lottery tickets and materials were illegally seized, still this is no legal objection to the admission of them in evidence. If the search warrant where illegal, or if the officer serving the warrant exceeded his authority, the party on whose complaint the warrant issued, or the officer, would be responsible for the wrong done. But this is no good reason for excluding the papers seized, as evidence, if they were pertinent to the issue, as they unquestionably were. When papers are offered in evidence the Court can take no notice how they were obtained, — whether lawfully or unlawfully, — nor would they form a collateral issue to determine that question." "We know of no constitutional principal which requires the government to surrender the papers under such circumstances. "The papers having come into possession of the government without a violation of petitioner's rights by governmental authority, we see no reason why the fact that individuals unconnected with the government may have wrongfully taken them, should prevent them from being held for use in prosecuting an offense where the documents are of incriminatory character." (Burdeau vs . McDowell.) We are confirmed in this conclusion when we reflect how far- reaching in its effect upon society the new consequences would be. The pettiest peace officer would have it in his power, through over-zeal or indiscretions, to confer immunity upon an offender for crimes the most flagitious. A room is searched against the law, and the body of a murdered man is found. If the place of discovery may not be proved, the other circumstances may be insufficient to connect the defendant with the crime. The privacy of the home has been infringed, and the murder goes free. Another search, once more against the law, discloses counterfeit money or the implements of forgery. The absence of a warrant means the freedom of the forger. Like instances can be multiplied. 1

Upload: prelle-dacanay

Post on 13-Apr-2015

45 views

Category:

Documents


0 download

DESCRIPTION

Search & Seizure Cases in Constitutional Law II under Atty Fiel at College of Law, New Era University.*20 full casesTags: Search, seizure, warrant, arrest

TRANSCRIPT

Page 1: Search and Seizure Cases

HILARIO CAMINO MONCADO, recurrente, vs.EL TRIBUNAL DEL PUEBLO Y JUAN M. LADAW, como Procurador

As we understand it, the main, if not the sole, purpose of our constitutional inhibitions against unreasonable searches and seizures, was to place a salutary restriction upon the powers of government. That is to say, we believe the framers of the constitutions of the United States and of this and other states merely sought to provide against any attempt, by legislation or otherwise, to authorize, justify, or declare lawful, any unreasonable search or seizure. This wise restriction was intended to operate upon legislative bodies, so as to render ineffectual any effort to legalize by statute what the people expressly stipulated could in no event be made lawful; upon executives, so that no law violative of this constitutional inhibition should ever be enforced; and upon the judiciary, so as to render it the duty of the courts to denounce as unlawful every unreasonable search and seizure, whether confessedly without any color of authority, or sought to be justified under the guise of legislative sanction. For the misconduct of private persons, acting upon their individual responsibility and of their own volition, surely none of the three divisions of government is responsible. If an official, or a mere petty agent of the state, exceeds or abuses the authority with which he is clothed, he is to be deemed as acting, not for the state, but for himself only; and therefore he alone, and not the state, should be held accountable for his acts. If the constitutional rights of a citizen are invaded by a mere individual, the most that any branch of government can do is to afford the citizen such redress as a possible, and bring the wrongdoer to account for his unlawful conduct. . . . .

It is contemplated, and such ought ever to be the fact, that the records of courts remains permanently in the places assigned by the law for their custody. It does not logically follow, however, that the records, being obtained, cannot be used as instruments of evidence; for the mere fact of (illegally) obtaining them does not change that which is written in them ... . Suppose the presence of a witness to have been procured by fraud or violence, while the party thus procuring the attendance of the witness would be liable to severe punishment, surely that could not be urged against the competency of the witness. If he could not, why shall a record, although illegally taken from its proper place of custody and brought before the Court, but otherwise free from suspicion, be hold incompetent?

Admitting that the lottery tickets and materials were illegally seized, still this is no legal objection to the admission of them in evidence. If the search warrant where illegal, or if the officer serving the warrant exceeded his authority, the party on whose complaint the warrant issued, or the officer, would be responsible for the wrong done. But this is no good reason for excluding the papers seized, as evidence, if they were pertinent to the issue, as they unquestionably were. When papers are offered in evidence the Court can take no notice how they were obtained, — whether lawfully or unlawfully, — nor would they form a collateral issue to determine that question."

"We know of no constitutional principal which requires the government to surrender the papers under such circumstances.

"The papers having come into possession of the government without a violation of petitioner's rights by governmental authority, we see no reason why the fact that individuals unconnected with the government may have wrongfully taken them, should prevent them from being held for use in prosecuting an offense where the documents are of incriminatory character." (Burdeau vs. McDowell.)

We are confirmed in this conclusion when we reflect how far-reaching in its effect upon society the new consequences would be. The pettiest peace officer would have it in his power, through over-zeal or indiscretions, to confer immunity upon an offender for crimes the most flagitious. A room is searched against the law, and the body of a murdered man is found. If the place of discovery may not be proved, the other circumstances may be insufficient to connect the defendant with the crime. The privacy of the home has been infringed, and the murder goes free. Another search, once more against the law, discloses counterfeit money or the implements of forgery. The absence of a warrant means the freedom of the forger. Like instances can be multiplied.

To hold that no criminal can, in any case, be arrested and searched for the evidence and tokens of his crime without a warrant, would be to leave society, to a large extent, at the mercy of the shrewdest, the most expert, and the most depraved of criminals, facilitating their escape in many instances." Y en People vs. Mayen, 205 Pac., 435, se dijo: "Upon what theory can it be held that such proceeding (for the return of the articles) is an incident of the trial, in such a sense that the ruling thereon goes up on appeal as part of the record and subject to review by the appellate court? It seems to us rather an independent proceeding to enforce a civil right in no way involved in the criminal case. The right of the defendant is not to exclude the incriminating documents from evidence a civil right in no way involved in the criminal case. The right of the defendant is not to exclude the incriminating documents from evidence, but to recover the possession of articles which were wrongfully taken from him. That right exists entirely apart from any proposed use of the property by the State or whereas it was the invasion of his premises and the taking of his goods that constituted the offense irrespective of what was taken or what use was made of its; and the law having declared that the articles taken are competent and admissible evidence, notwithstanding the unlawful search and seizure, how can the circumstance that the court erred in an independent proceeding for the return of the property on defendant's demand add anything to or detract from the violation of the defendant's constitutional rights in the unlawful search and seizure?

The Constitutional and the laws of the land are not solicitous to aid persons charged with crime in their efforts to conceal or sequester evidence of their iniquity. (8 Wig., 37.)

HARRY S. STONEHILL, ROBERT P. BROOKS, JOHN J. BROOKS and KARL BECK, petitioners, vs.HON. JOSE W. DIOKNO,

Upon application of the officers of the government named on the margin1 — hereinafter referred to as Respondents-Prosecutors — several judges2 — hereinafter referred to as Respondents-Judges — issued, on different dates,3 a total of 42 search warrants against petitioners herein4 and/or the corporations of which they were officers,5 directed to the any peace officer, to search the persons 

1

Page 2: Search and Seizure Cases

above-named and/or the premises of their offices, warehouses and/or residences, and to seize and take possession of the following personal property to wit:

Books of accounts, financial records, vouchers, correspondence, receipts, ledgers, journals, portfolios, credit journals, typewriters, and other documents and/or papers showing all business transactions including disbursements receipts, balance sheets and profit and loss statements and Bobbins (cigarette wrappers).

as "the subject of the offense; stolen or embezzled and proceeds or fruits of the offense," or "used or intended to be used as the means of committing the offense," which is described in the applications adverted to above as "violation of Central Bank Laws, Tariff and Customs Laws, Internal Revenue (Code) and the Revised Penal Code."

Alleging that the aforementioned search warrants are null and void, as contravening the Constitution and the Rules of Court — because, inter alia: (1) they do not describe with particularity the documents, books and things to be seized; (2) cash money, not mentioned in the warrants, were actually seized; (3) the warrants were issued to fish evidence against the aforementioned petitioners in deportation cases filed against them; (4) the searches and seizures were made in an illegal manner; and (5) the documents, papers and cash money seized were not delivered to the courts that issued the warrants, to be disposed of in accordance with law — on March 20, 1962, said petitioners filed with the Supreme Court this original action for certiorari, prohibition, mandamus and injunction, and prayed that, pending final disposition of the present case, a writ of preliminary injunction be issued restraining Respondents-Prosecutors, their agents and /or representatives from using the effects seized as aforementioned or any copies thereof, in the deportation cases already adverted to, and that, in due course, thereafter, decision be rendered quashing the contested search warrants and declaring the same null and void, and commanding the respondents, their agents or representatives to return to petitioners herein, in accordance with Section 3, Rule 67, of the Rules of Court, the documents, papers, things and cash moneys seized or confiscated under the search warrants in question.

In their answer, respondents-prosecutors alleged, 6 (1) that the contested search warrants are valid and have been issued in accordance with law; (2) that the defects of said warrants, if any, were cured by petitioners' consent; and (3) that, in any event, the effects seized are admissible in evidence against herein petitioners, regardless of the alleged illegality of the aforementioned searches and seizures.

On March 22, 1962, this Court issued the writ of preliminary injunction prayed for in the petition. However, by resolution dated June 29, 1962, the writ was partially lifted or dissolved, insofar as the papers, documents and things seized from the offices of the corporations above mentioned are concerned; but, the injunction was maintained as regards the papers, documents and things found and seized in the residences of petitioners herein.7

Thus, the documents, papers, and things seized under the alleged authority of the warrants in question may be split into two (2) major groups, namely: (a) those found and seized in the offices of 

the aforementioned corporations, and (b) those found and seized in the residences of petitioners herein.

As regards the first group, we hold that petitioners herein have no cause of action to assail the legality of the contested warrants and of the seizures made in pursuance thereof, for the simple reason that said corporations have their respective personalities, separate and distinct from the personality of herein petitioners, regardless of the amount of shares of stock or of the interest of each of them in said corporations, and whatever the offices they hold therein may be.8 Indeed, it is well settled that the legality of a seizure can be contested only by the party whose rights have been impaired thereby,9 and that the objection to an unlawful search and seizure is purely personal and cannot be availed of by third parties. 10 Consequently, petitioners herein may not validly object to the use in evidence against them of the documents, papers and things seized from the offices and premises of the corporations adverted to above, since the right to object to the admission of said papers in evidence belongsexclusively to the corporations, to whom the seized effects belong, and may not be invoked by the corporate officers in proceedings against them in their individual capacity. 11 Indeed, it has been held:

. . . that the Government's action in gaining possession of papers belonging to the corporation did not relate to nor did it affect the personal defendants. If these papers were unlawfully seized and thereby the constitutional rights of or any one were invaded, they were the rights of the corporation and not the rights of the other defendants. Next, it is clear that a question of the lawfulness of a seizure can be raised only by one whose rights have been invaded. Certainly, such a seizure, if unlawful, could not affect the constitutional rights of defendants whose property had not been seized or the privacy of whose homes had not been disturbed; nor could they claim for themselves the benefits of the Fourth Amendment, when its violation, if any, was with reference to the rights of another. Remus vs. United States (C.C.A.)291 F. 501, 511. It follows, therefore, that the question of the admissibility of the evidence based on an alleged unlawful search and seizure does not extend to the personal defendants but embraces only the corporation whose property was taken. . . . (A Guckenheimer & Bros. Co. vs. United States, [1925] 3 F. 2d. 786, 789, Emphasis supplied.)

With respect to the documents, papers and things seized in the residences of petitioners herein, the aforementioned resolution of June 29, 1962, lifted the writ of preliminary injunction previously issued by this Court,12 thereby, in effect, restraining herein Respondents-Prosecutors from using them in evidence against petitioners herein.

In connection with said documents, papers and things, two (2) important questions need be settled, namely: (1) whether the search warrants in question, and the searches and seizures made under the authority thereof, are valid or not, and (2) if the answer to the preceding question is in the negative, whether said documents, papers and things may be used in evidence against petitioners herein.1äwphï1.ñët

2

Page 3: Search and Seizure Cases

Petitioners maintain that the aforementioned search warrants are in the nature of general warrants and that accordingly, the seizures effected upon the authority there of are null and void. In this connection, the Constitution13 provides:

The right of the people to be secure in their persons, houses, papers, and effects against unreasonable searches and seizures shall not be violated, and no warrants shall issue but upon probable cause, to be determined by the judge after examination under oath or affirmation of the complainant and the witnesses he may produce, and particularly describing the place to be searched, and the persons or things to be seized.

Two points must be stressed in connection with this constitutional mandate, namely: (1) that no warrant shall issue but upon probable cause, to be determined by the judge in the manner set forth in said provision; and (2) that the warrant shall particularly describe the things to be seized.

None of these requirements has been complied with in the contested warrants. Indeed, the same were issued upon applications stating that the natural and juridical person therein named had committed a "violation of Central Ban Laws, Tariff and Customs Laws, Internal Revenue (Code) and Revised Penal Code." In other words, nospecific offense had been alleged in said applications. The averments thereof with respect to the offense committed were abstract. As a consequence, it was impossible for the judges who issued the warrants to have found the existence of probable cause, for the same presupposes the introduction of competent proof that the party against whom it is sought has performed particular acts, or committed specific omissions, violating a given provision of our criminal laws. As a matter of fact, the applications involved in this case do not allege any specific acts performed by herein petitioners. It would be the legal heresy, of the highest order, to convict anybody of a "violation of Central Bank Laws, Tariff and Customs Laws, Internal Revenue (Code) and Revised Penal Code," — as alleged in the aforementioned applications — without reference to any determinate provision of said laws or

To uphold the validity of the warrants in question would be to wipe out completely one of the most fundamental rights guaranteed in our Constitution, for it would place the sanctity of the domicile and the privacy of communication and correspondence at the mercy of the whims caprice or passion of peace officers. This is precisely the evil sought to be remedied by the constitutional provision above quoted — to outlaw the so-called general warrants. It is not difficult to imagine what would happen, in times of keen political strife, when the party in power feels that the minority is likely to wrest it, even though by legal means.

Such is the seriousness of the irregularities committed in connection with the disputed search warrants, that this Court deemed it fit to amend Section 3 of Rule 122 of the former Rules of Court 14 by providing in its counterpart, under the Revised Rules of Court 15 that "a search warrant shall not issue but upon probable cause in connection with one specific offense." Not satisfied with this qualification, the Court added thereto a paragraph, directing that "no search warrant shall issue for more than one specific offense."

The grave violation of the Constitution made in the application for the contested search warrants was compounded by the description therein made of the effects to be searched for and seized, to wit:

Books of accounts, financial records, vouchers, journals, correspondence, receipts, ledgers, portfolios, credit journals, typewriters, and other documents and/or papers showing all business transactions including disbursement receipts, balance sheets and related profit and loss statements.

Thus, the warrants authorized the search for and seizure of records pertaining to all business transactions of petitioners herein, regardless of whether the transactions were legal or illegal. The warrants sanctioned the seizure of all records of the petitioners and the aforementioned corporations, whatever their nature, thus openly contravening the explicit command of our Bill of Rights — that the things to be seized be particularly described — as well as tending to defeat its major objective: the elimination of general warrants.

Relying upon Moncado vs. People's Court (80 Phil. 1), Respondents-Prosecutors maintain that, even if the searches and seizures under consideration were unconstitutional, the documents, papers and things thus seized are admissible in evidence against petitioners herein. Upon mature deliberation, however, we are unanimously of the opinion that the position taken in the Moncado case must be abandoned. Said position was in line with the American common law rule, that the criminal should not be allowed to go free merely "because the constable has blundered," 16 upon the theory that the constitutional prohibition against unreasonable searches and seizures is protected by means other than the exclusion of evidence unlawfully obtained, 17 such as the common-law action for damages against the searching officer, against the party who procured the issuance of the search warrant and against those assisting in the execution of an illegal search, their criminal punishment, resistance, without liability to an unlawful seizure, and such other legal remedies as may be provided by other laws.

However, most common law jurisdictions have already given up this approach and eventually adopted the exclusionary rule, realizing that this is the only practical means of enforcing the constitutional injunction against unreasonable searches and seizures. In the language of Judge Learned Hand:

As we understand it, the reason for the exclusion of evidence competent as such, which has been unlawfully acquired, is that exclusion is the only practical way of enforcing the constitutional privilege. In earlier times the action of trespass against the offending official may have been protection enough; but that is true no longer. Only in case the prosecution which itself controls the seizing officials, knows that it cannot profit by their wrong will that wrong be repressed.18

In fact, over thirty (30) years before, the Federal Supreme Court had already declared:

If letters and private documents can thus be seized and held and used in evidence against a citizen accused of an offense, the protection of the 4th Amendment, declaring his rights to be secure against such searches and seizures, is of no value, and, so far as those thus placed are concerned, 

3

Page 4: Search and Seizure Cases

might as well be stricken from the Constitution. The efforts of the courts and their officials to bring the guilty to punishment, praiseworthy as they are, are not to be aided by the sacrifice of those great principles established by years of endeavor and suffering which have resulted in their embodiment in the fundamental law of the land.19

This view was, not only reiterated, but, also, broadened in subsequent decisions on the same Federal Court. 20After reviewing previous decisions thereon, said Court held, in Mapp vs. Ohio (supra.):

. . . Today we once again examine the Wolf's constitutional documentation of the right of privacy free from unreasonable state intrusion, and after its dozen years on our books, are led by it to close the only courtroom door remaining open to evidence secured by official lawlessness in flagrant abuse of that basic right, reserved to all persons as a specific guarantee against that very same unlawful conduct. We hold that all evidence obtained by searches and seizures in violation of the Constitution is, by that same authority, inadmissible in a State.

Since the Fourth Amendment's right of privacy has been declared enforceable against the States through the Due Process Clause of the Fourteenth, it is enforceable against them by the same sanction of exclusion as it used against the Federal Government. Were it otherwise, then just as without the Weeks rule the assurance against unreasonable federal searches and seizures would be "a form of words," valueless and underserving of mention in a perpetual charter of inestimable human liberties, so too, without that rule the freedom from state invasions of privacy would be so ephemeral and so neatly severed from its conceptual nexus with the freedom from all brutish means of coercing evidence as not to permit this Court's high regard as a freedom "implicit in the concept of ordered liberty." At the time that the Court held in Wolf that the amendment was applicable to the States through the Due Process Clause, the cases of this Court as we have seen, had steadfastly held that as to federal officers the Fourth Amendment included the exclusion of the evidence seized in violation of its provisions. Even Wolf "stoutly adhered" to that proposition. The right to when conceded operatively enforceable against the States, was not susceptible of destruction by avulsion of the sanction upon which its protection and enjoyment had always been deemed dependent under the Boyd, Weeks and Silverthorne Cases. Therefore, in extending the substantive protections of due process to all constitutionally unreasonable searches — state or federal — it was logically and constitutionally necessarily that the exclusion doctrine — an essential part of the right to privacy — be also insisted upon as an essential ingredient of the right newly recognized by the Wolf Case. In short, the admission of the new constitutional Right by Wolf could not tolerate denial of its most important constitutional privilege, namely, the exclusion of the evidence which an accused had been forced to give by reason of the unlawful seizure. To hold otherwise is to grant the right but in reality to withhold its privilege and enjoyment. Only last year the Court itself recognized that the purpose of the exclusionary rule to "is to deter — to compel respect for the constitutional guaranty in the only effectively available way — by removing the incentive to disregard it" . . . .

The ignoble shortcut to conviction left open to the State tends to destroy the entire system of constitutional restraints on which the liberties of the people rest. Having once recognized that the right to privacy embodied in the Fourth Amendment is enforceable against the States, and that the right to be secure against rude invasions of privacy by state officers is, therefore constitutional in 

origin, we can no longer permit that right to remain an empty promise. Because it is enforceable in the same manner and to like effect as other basic rights secured by its Due Process Clause, we can no longer permit it to be revocable at the whim of any police officer who, in the name of law enforcement itself, chooses to suspend its enjoyment. Our decision, founded on reason and truth, gives to the individual no more than that which the Constitution guarantees him to the police officer no less than that to which honest law enforcement is entitled, and, to the courts, that judicial integrity so necessary in the true administration of justice. (emphasis ours.)

Indeed, the non-exclusionary rule is contrary, not only to the letter, but also, to the spirit of the constitutional injunction against unreasonable searches and seizures. To be sure, if the applicant for a search warrant has competent evidence to establish probable cause of the commission of a given crime by the party against whom the warrant is intended, then there is no reason why the applicant should not comply with the requirements of the fundamental law. Upon the other hand, if he has no such competent evidence, then it is not possible for the Judge to find that there is probable cause, and, hence, no justification for the issuance of the warrant. The only possible explanation (not justification) for its issuance is the necessity of fishing evidence of the commission of a crime. But, then, this fishing expedition is indicative of the absence of evidence to establish a probable cause.

Moreover, the theory that the criminal prosecution of those who secure an illegal search warrant and/or make unreasonable searches or seizures would suffice to protect the constitutional guarantee under consideration, overlooks the fact that violations thereof are, in general, committed By agents of the party in power, for, certainly, those belonging to the minority could not possibly abuse a power they do not have. Regardless of the handicap under which the minority usually — but, understandably — finds itself in prosecuting agents of the majority, one must not lose sight of the fact that the psychological and moral effect of the possibility 21 of securing their conviction, is watered down by the pardoning power of the party for whose benefit the illegality had been committed.

In their Motion for Reconsideration and Amendment of the Resolution of this Court dated June 29, 1962, petitioners allege that Rooms Nos. 81 and 91 of Carmen Apartments, House No. 2008, Dewey Boulevard, House No. 1436, Colorado Street, and Room No. 304 of the Army-Navy Club, should be included among the premises considered in said Resolution as residences of herein petitioners, Harry S. Stonehill, Robert P. Brook, John J. Brooks and Karl Beck, respectively, and that, furthermore, the records, papers and other effects seized in the offices of the corporations above referred to include personal belongings of said petitioners and other effects under their exclusive possession and control, for the exclusion of which they have a standing under the latest rulings of the federal courts of federal courts of the United States. 22

We note, however, that petitioners' theory, regarding their alleged possession of and control over the aforementioned records, papers and effects, and the alleged "personal" nature thereof, has Been Advanced, notin their petition or amended petition herein, but in the Motion for Reconsideration and Amendment of the Resolution of June 29, 1962. In other words, said theory would appear to be readjustment of that followed in said petitions, to suit the approach intimated in the Resolution sought to be reconsidered and amended. Then, too, some of the affidavits or 

4

Page 5: Search and Seizure Cases

copies of alleged affidavits attached to said motion for reconsideration, or submitted in support thereof, contain either inconsistent allegations, or allegations inconsistent with the theory now advanced by petitioners herein.

Upon the other hand, we are not satisfied that the allegations of said petitions said motion for reconsideration, and the contents of the aforementioned affidavits and other papers submitted in support of said motion, have sufficiently established the facts or conditions contemplated in the cases relied upon by the petitioners; to warrant application of the views therein expressed, should we agree thereto. At any rate, we do not deem it necessary to express our opinion thereon, it being best to leave the matter open for determination in appropriate cases in the future.

We hold, therefore, that the doctrine adopted in the Moncado case must be, as it is hereby, abandoned; that the warrants for the search of three (3) residences of herein petitioners, as specified in the Resolution of June 29, 1962, are null and void; that the searches and seizures therein made are illegal; that the writ of preliminary injunction heretofore issued, in connection with the documents, papers and other effects thus seized in said residences of herein petitioners is hereby made permanent; that the writs prayed for are granted, insofar as the documents, papers and other effects so seized in the aforementioned residences are concerned; that the aforementioned motion for Reconsideration and Amendment should be, as it is hereby, denied; and that the petition herein is dismissed and the writs prayed for denied, as regards the documents, papers and other effects seized in the twenty-nine (29) places, offices and other premises enumerated in the same Resolution, without special pronouncement as to costs.

PEOPLE OF THE PHILIPPINES, plaintiff-appellee vs.ANDRE MARTI, accused-appellant.

This is an appeal from a decision * rendered by the Special Criminal Court of Manila (Regional Trial Court, Branch XLIX) convicting accused-appellant of violation of Section 21 (b), Article IV in relation to Section 4, Article 11 and Section 2 (e) (i), Article 1 of Republic Act 6425, as amended, otherwise known as the Dangerous Drugs Act.

The facts as summarized in the brief of the prosecution are as follows:

On August 14, 1987, between 10:00 and 11:00 a.m., the appellant and his common-law wife, Shirley Reyes, went to the booth of the "Manila Packing and Export Forwarders" in the Pistang Pilipino Complex, Ermita, Manila, carrying with them four (4) gift wrapped packages. Anita Reyes (the proprietress and no relation to Shirley Reyes) attended to them. The appellant informed Anita Reyes that he was sending the packages to a friend in Zurich, Switzerland. Appellant filled up the contract necessary for the transaction, writing therein his name, passport number, the date of shipment and the name and address of the consignee, namely, "WALTER FIERZ, Mattacketr II, 8052 Zurich, Switzerland" (Decision, p. 6)

Anita Reyes then asked the appellant if she could examine and inspect the packages. Appellant, however, refused, assuring her that the packages simply contained books, cigars, and gloves and 

were gifts to his friend in Zurich. In view of appellant's representation, Anita Reyes no longer insisted on inspecting the packages. The four (4) packages were then placed inside a brown corrugated box one by two feet in size (1' x 2'). Styro-foam was placed at the bottom and on top of the packages before the box was sealed with masking tape, thus making the box ready for shipment (Decision, p. 8).

Before delivery of appellant's box to the Bureau of Customs and/or Bureau of Posts, Mr. Job Reyes (proprietor) and husband of Anita (Reyes), following standard operating procedure, opened the boxes for final inspection. When he opened appellant's box, a peculiar odor emitted therefrom. His curiousity aroused, he squeezed one of the bundles allegedly containing gloves and felt dried leaves inside. Opening one of the bundles, he pulled out a cellophane wrapper protruding from the opening of one of the gloves. He made an opening on one of the cellophane wrappers and took several grams of the contents thereof (tsn, pp. 29-30, October 6, 1987; Emphasis supplied).

Job Reyes forthwith prepared a letter reporting the shipment to the NBI and requesting a laboratory examination of the samples he extracted from the cellophane wrapper (tsn, pp. 5-6, October 6, 1987).

He brought the letter and a sample of appellant's shipment to the Narcotics Section of the National Bureau of Investigation (NBI), at about 1:30 o'clock in the afternoon of that date, i.e., August 14, 1987. He was interviewed by the Chief of Narcotics Section. Job Reyes informed the NBI that the rest of the shipment was still in his office. Therefore, Job Reyes and three (3) NBI agents, and a photographer, went to the Reyes' office at Ermita, Manila (tsn, p. 30, October 6, 1987).

Job Reyes brought out the box in which appellant's packages were placed and, in the presence of the NBI agents, opened the top flaps, removed the styro-foam and took out the cellophane wrappers from inside the gloves. Dried marijuana leaves were found to have been contained inside the cellophane wrappers (tsn, p. 38, October 6, 1987; Emphasis supplied).

The package which allegedly contained books was likewise opened by Job Reyes. He discovered that the package contained bricks or cake-like dried marijuana leaves. The package which allegedly contained tabacalera cigars was also opened. It turned out that dried marijuana leaves were neatly stocked underneath the cigars (tsn, p. 39, October 6, 1987).

The NBI agents made an inventory and took charge of the box and of the contents thereof, after signing a "Receipt" acknowledging custody of the said effects (tsn, pp. 2-3, October 7, 1987).

Thereupon, the NBI agents tried to locate appellant but to no avail. Appellant's stated address in his passport being the Manila Central Post Office, the agents requested assistance from the latter's Chief Security. On August 27, 1987, appellant, while claiming his mail at the Central Post Office, was invited by the NBI to shed light on the attempted shipment of the seized dried leaves. On the same day the Narcotics Section of the NBI submitted the dried leaves to the Forensic Chemistry Section for laboratory examination. It turned out that the dried leaves were marijuana flowering tops as certified by the forensic chemist. (Appellee's Brief, pp. 9-11, Rollo, pp. 132-134).

5

Page 6: Search and Seizure Cases

Thereafter, an Information was filed against appellant for violation of RA 6425, otherwise known as the Dangerous Drugs Act.

After trial, the court a quo rendered the assailed decision.

In this appeal, accused/appellant assigns the following errors, to wit:

THE LOWER COURT ERRED IN ADMITTING IN EVIDENCE THE ILLEGALLY SEARCHED AND SEIZED OBJECTS CONTAINED IN THE FOUR PARCELS.

THE LOWER COURT ERRED IN CONVICTING APPELLANT DESPITE THE UNDISPUTED FACT THAT HIS RIGHTS UNDER THE CONSTITUTION WHILE UNDER CUSTODIAL PROCEEDINGS WERE NOT OBSERVED.

THE LOWER COURT ERRED IN NOT GIVING CREDENCE TO THE EXPLANATION OF THE APPELLANT ON HOW THE FOUR PARCELS CAME INTO HIS POSSESSION (Appellant's Brief, p. 1;Rollo, p. 55)

1. Appellant contends that the evidence subject of the imputed offense had been obtained in violation of his constitutional rights against unreasonable search and seizure and privacy of communication (Sec. 2 and 3, Art. III, Constitution) and therefore argues that the same should be held inadmissible in evidence (Sec. 3 (2), Art. III).

Sections 2 and 3, Article III of the Constitution provide:

Sec. 2. The right of the people to be secure in their persons, houses, papers and effects against unreasonable searches and seizures of whatever nature and for any purpose shall be inviolable, and no search warrant or warrant of arrest shall issue except upon probable cause to be determined personally by the judge after examination under oath or affirmation of the complainant and the witnesses he may produce, and particularly describing the place to be searched and the persons or things to be seized.

Sec. 3. (1) The privacy of communication and correspondence shall be inviolable except upon lawful order of the court, or when public safety or order requires otherwise as prescribed by law.

(2) Any evidence obtained in violation of this or the preceding section shall be inadmissible for any purpose in any proceeding.

Our present constitutional provision on the guarantee against unreasonable search and seizure had its origin in the 1935 Charter which, worded as follows:

The right of the people to be secure in their persons, houses, papers and effects against unreasonable searches and seizures shall not be violated, and no warrants shall issue but 

uponprobable cause, to be determined by the judge after examination under oath or affirmation of the complainant and the witnesses he may produce, and particularly describing the place to be searched, and the persons or things to be seized. (Sec. 1 [3], Article III)

was in turn derived almost verbatim from the Fourth Amendment ** to the United States Constitution. As such, the Court may turn to the pronouncements of the United States Federal Supreme Court and State Appellate Courts which are considered doctrinal in this jurisdiction.

Thus, following the exclusionary rule laid down in Mapp v. Ohio by the US Federal Supreme Court (367 US 643, 81 S.Ct. 1684, 6 L.Ed. 1081 [1961]), this Court, in Stonehill v. Diokno (20 SCRA 383 [1967]), declared as inadmissible any evidence obtained by virtue of a defective search and seizure warrant, abandoning in the process the ruling earlier adopted in Moncado v. People's Court (80 Phil. 1 [1948]) wherein the admissibility of evidence was not affected by the illegality of its seizure. The 1973 Charter (Sec. 4 [2], Art. IV) constitutionalized the Stonehill ruling and is carried over up to the present with the advent of the 1987 Constitution.

In a number of cases, the Court strictly adhered to the exclusionary rule and has struck down the admissibility of evidence obtained in violation of the constitutional safeguard against unreasonable searches and seizures. (Bache & Co., (Phil.), Inc., v. Ruiz, 37 SCRA 823 [1971]; Lim v. Ponce de Leon, 66 SCRA 299 [1975]; People v. Burgos, 144 SCRA 1 [1986]; Roan v. Gonzales, 145 SCRA 687 [1987]; See also Salazar v. Hon. Achacoso, et al., GR No. 81510, March 14, 1990).

It must be noted, however, that in all those cases adverted to, the evidence so obtained were invariably procured by the State acting through the medium of its law enforcers or other authorized government agencies.

On the other hand, the case at bar assumes a peculiar character since the evidence sought to be excluded was primarily discovered and obtained by a private person, acting in a private capacity and without the intervention and participation of State authorities. Under the circumstances, can accused/appellant validly claim that his constitutional right against unreasonable searches and seizure has been violated? Stated otherwise, may an act of a private individual, allegedly in violation of appellant's constitutional rights, be invoked against the State?

We hold in the negative. In the absence of governmental interference, the liberties guaranteed by the Constitution cannot be invoked against the State.

As this Court held in Villanueva v. Querubin (48 SCRA 345 [1972]:

1. This constitutional right (against unreasonable search and seizure) refers to the immunity of one's person, whether citizen or alien, from interference by government, included in which is his residence, his papers, and other possessions. . . .

6

Page 7: Search and Seizure Cases

. . . There the state, however powerful, does not as such have the access except under the circumstances above noted, for in the traditional formulation, his house, however humble, is his castle. Thus is outlawed any unwarranted intrusion by government, which is called upon to refrain from any invasion of his dwelling and to respect the privacies of his life. . . . (Cf. Schermerber v. California, 384 US 757 [1966] and Boyd v. United States, 116 US 616 [1886]; Emphasis supplied).

In Burdeau v. McDowell (256 US 465 (1921), 41 S Ct. 547; 65 L.Ed. 1048), the Court there in construing the right against unreasonable searches and seizures declared that:

(t)he Fourth Amendment gives protection against unlawful searches and seizures, and as shown in previous cases, its protection applies to governmental action. Its origin and history clearly show that it was intended as a restraint upon the activities of sovereign authority, and was not intended to be a limitation upon other than governmental agencies; as against such authority it was the purpose of the Fourth Amendment to secure the citizen in the right of unmolested occupation of his dwelling and the possession of his property, subject to the right of seizure by process duly served.

The above ruling was reiterated in State v. Bryan (457 P.2d 661 [1968]) where a parking attendant who searched the automobile to ascertain the owner thereof found marijuana instead, without the knowledge and participation of police authorities, was declared admissible in prosecution for illegal possession of narcotics.

And again in the 1969 case of Walker v. State (429 S.W.2d 121), it was held that the search and seizure clauses are restraints upon the government and its agents, not upon private individuals (citing People v. Potter, 240 Cal. App.2d 621, 49 Cap. Rptr, 892 (1966); State v. Brown, Mo., 391 S.W.2d 903 (1965); State v. Olsen, Or., 317 P.2d 938 (1957).

Likewise appropos is the case of Bernas v. US (373 F.2d 517 (1967). The Court there said:

The search of which appellant complains, however, was made by a private citizen — the owner of a motel in which appellant stayed overnight and in which he left behind a travel case containing the evidence***complained of. The search was made on the motel owner's own initiative. Because of it, he became suspicious, called the local police, informed them of the bag's contents, and made it available to the authorities.

The fourth amendment and the case law applying it do not require exclusion of evidence obtained through a search by a private citizen. Rather, the amendment only proscribes governmental action."

The contraband in the case at bar having come into possession of the Government without the latter transgressing appellant's rights against unreasonable search and seizure, the Court sees no cogent reason why the same should not be admitted against him in the prosecution of the offense charged.

Appellant, however, would like this court to believe that NBI agents made an illegal search and seizure of the evidence later on used in prosecuting the case which resulted in his conviction.

The postulate advanced by accused/appellant needs to be clarified in two days. In both instances, the argument stands to fall on its own weight, or the lack of it.

First, the factual considerations of the case at bar readily foreclose the proposition that NBI agents conducted an illegal search and seizure of the prohibited merchandise. Records of the case clearly indicate that it was Mr. Job Reyes, the proprietor of the forwarding agency, who made search/inspection of the packages. Said inspection was reasonable and a standard operating procedure on the part of Mr. Reyes as a precautionary measure before delivery of packages to the Bureau of Customs or the Bureau of Posts (TSN, October 6 & 7, 1987, pp. 15-18; pp. 7-8; Original Records, pp. 119-122; 167-168).

It will be recalled that after Reyes opened the box containing the illicit cargo, he took samples of the same to the NBI and later summoned the agents to his place of business. Thereafter, he opened the parcel containing the rest of the shipment and entrusted the care and custody thereof to the NBI agents. Clearly, the NBI agents made no search and seizure, much less an illegal one, contrary to the postulate of accused/appellant.

Second, the mere presence of the NBI agents did not convert the reasonable search effected by Reyes into a warrantless search and seizure proscribed by the Constitution. Merely to observe and look at that which is in plain sight is not a search. Having observed that which is open, where no trespass has been committed in aid thereof, is not search (Chadwick v. State, 429 SW2d 135). Where the contraband articles are identified without a trespass on the part of the arresting officer, there is not the search that is prohibited by the constitution (US v. Lee 274 US 559, 71 L.Ed. 1202 [1927]; Ker v. State of California 374 US 23, 10 L.Ed.2d. 726 [1963]; Moore v. State, 429 SW2d 122 [1968]).

In Gandy v. Watkins (237 F. Supp. 266 [1964]), it was likewise held that where the property was taken into custody of the police at the specific request of the manager and where the search was initially made by the owner there is no unreasonable search and seizure within the constitutional meaning of the term.

That the Bill of Rights embodied in the Constitution is not meant to be invoked against acts of private individuals finds support in the deliberations of the Constitutional Commission. True, the liberties guaranteed by the fundamental law of the land must always be subject to protection. But protection against whom? Commissioner Bernas in his sponsorship speech in the Bill of Rights answers the query which he himself posed, as follows:

First, the general reflections. The protection of fundamental liberties in the essence of constitutional democracy. Protection against whom? Protection against the state. The Bill of Rights governs the relationship between the individual and the state. Its concern is not the relation between individuals, between a private individual and other individuals. What the Bill of Rights does is to declare some forbidden zones in the private sphere inaccessible to any power holder. 

7

Page 8: Search and Seizure Cases

(Sponsorship Speech of Commissioner Bernas , Record of the Constitutional Commission, Vol. 1, p. 674; July 17, 1986; Emphasis supplied)

The constitutional proscription against unlawful searches and seizures therefore applies as a restraint directed only against the government and its agencies tasked with the enforcement of the law. Thus, it could only be invoked against the State to whom the restraint against arbitrary and unreasonable exercise of power is imposed.

If the search is made upon the request of law enforcers, a warrant must generally be first secured if it is to pass the test of constitutionality. However, if the search is made at the behest or initiative of the proprietor of a private establishment for its own and private purposes, as in the case at bar, and without the intervention of police authorities, the right against unreasonable search and seizure cannot be invoked for only the act of private individual, not the law enforcers, is involved. In sum, the protection against unreasonable searches and seizures cannot be extended to acts committed by private individuals so as to bring it within the ambit of alleged unlawful intrusion by the government.

Appellant argues, however, that since the provisions of the 1935 Constitution has been modified by the present phraseology found in the 1987 Charter, expressly declaring as inadmissible any evidence obtained in violation of the constitutional prohibition against illegal search and seizure, it matters not whether the evidence was procured by police authorities or private individuals (Appellant's Brief, p. 8, Rollo, p. 62).

The argument is untenable. For one thing, the constitution, in laying down the principles of the government and fundamental liberties of the people, does not govern relationships between individuals. Moreover, it must be emphasized that the modifications introduced in the 1987 Constitution (re: Sec. 2, Art. III) relate to the issuance of either a search warrant or warrant of arrest vis-a-vis the responsibility of the judge in the issuance thereof (SeeSoliven v. Makasiar, 167 SCRA 393 [1988]; Circular No. 13 [October 1, 1985] and Circular No. 12 [June 30, 1987]. The modifications introduced deviate in no manner as to whom the restriction or inhibition against unreasonable search and seizure is directed against. The restraint stayed with the State and did not shift to anyone else.

Corolarilly, alleged violations against unreasonable search and seizure may only be invoked against the State by an individual unjustly traduced by the exercise of sovereign authority. To agree with appellant that an act of a private individual in violation of the Bill of Rights should also be construed as an act of the State would result in serious legal complications and an absurd interpretation of the constitution.

Similarly, the admissibility of the evidence procured by an individual effected through private seizure equally applies, in pari passu, to the alleged violation, non-governmental as it is, of appellant's constitutional rights to privacy and communication.

2. In his second assignment of error, appellant contends that the lower court erred in convicting him despite the undisputed fact that his rights under the constitution while under custodial investigation were not observed.

Again, the contention is without merit, We have carefully examined the records of the case and found nothing to indicate, as an "undisputed fact", that appellant was not informed of his constitutional rights or that he gave statements without the assistance of counsel. The law enforcers testified that accused/appellant was informed of his constitutional rights. It is presumed that they have regularly performed their duties (See. 5(m), Rule 131) and their testimonies should be given full faith and credence, there being no evidence to the contrary. What is clear from the records, on the other hand, is that appellant refused to give any written statement while under investigation as testified by Atty. Lastimoso of the NBI, Thus:

Fiscal Formoso:You said that you investigated Mr. and Mrs. Job Reyes. What about the accused here, did you investigate the accused together with the girl?

WITNESS:Yes, we have interviewed the accused together with the girl but the accused availed of his constitutional right not to give any written statement, sir. (TSN, October 8, 1987, p. 62; Original Records, p. 240)

The above testimony of the witness for the prosecution was not contradicted by the defense on cross-examination. As borne out by the records, neither was there any proof by the defense that appellant gave uncounselled confession while being investigated. What is more, we have examined the assailed judgment of the trial court and nowhere is there any reference made to the testimony of appellant while under custodial investigation which was utilized in the finding of conviction. Appellant's second assignment of error is therefore misplaced.

3. Coming now to appellant's third assignment of error, appellant would like us to believe that he was not the owner of the packages which contained prohibited drugs but rather a certain Michael, a German national, whom appellant met in a pub along Ermita, Manila: that in the course of their 30-minute conversation, Michael requested him to ship the packages and gave him P2,000.00 for the cost of the shipment since the German national was about to leave the country the next day (October 15, 1987, TSN, pp. 2-10).

Rather than give the appearance of veracity, we find appellant's disclaimer as incredulous, self-serving and contrary to human experience. It can easily be fabricated. An acquaintance with a complete stranger struck in half an hour could not have pushed a man to entrust the shipment of four (4) parcels and shell out P2,000.00 for the purpose and for appellant to readily accede to comply with the undertaking without first ascertaining its contents. As stated by the trial court, "(a) person would not simply entrust contraband and of considerable value at that as the marijuana flowering tops, and the cash amount of P2,000.00 to a complete stranger like the Accused. The Accused, on the other hand, would not simply accept such undertaking to take custody of the packages and ship the same from a complete stranger on his mere say-so" (Decision, p. 19, Rollo, p. 91). As to why he readily agreed to do the errand, appellant failed to explain. Denials, if 

8

Page 9: Search and Seizure Cases

unsubstantiated by clear and convincing evidence, are negative self-serving evidence which deserve no weight in law and cannot be given greater evidentiary weight than the testimony of credible witnesses who testify on affirmative matters (People v. E

Appellant's bare denial is even made more suspect considering that, as per records of the Interpol, he was previously convicted of possession of hashish by the Kleve Court in the Federal Republic of Germany on January 1, 1982 and that the consignee of the frustrated shipment, Walter Fierz, also a Swiss national, was likewise convicted for drug abuse and is just about an hour's drive from appellant's residence in Zurich, Switzerland (TSN, October 8, 1987, p. 66; Original Records, p. 244; Decision, p. 21; Rollo, p. 93).

Evidence to be believed, must not only proceed from the mouth of a credible witness, but it must be credible in itself such as the common experience and observation of mankind can approve as probable under the circumstances (People v. Alto, 26 SCRA 342 [1968], citing Daggers v. Van Dyke, 37 N.J. Eg. 130; see also People v. Sarda, 172 SCRA 651 [1989]; People v. Sunga, 123 SCRA 327 [1983]); Castañares v. CA, 92 SCRA 567 [1979]). As records further show, appellant did not even bother to ask Michael's full name, his complete address or passport number. Furthermore, if indeed, the German national was the owner of the merchandise, appellant should have so indicated in the contract of shipment (Exh. "B", Original Records, p. 40). On the contrary, appellant signed the contract as the owner and shipper thereof giving more weight to the presumption that things which a person possesses, or exercises acts of ownership over, are owned by him (Sec. 5 [j], Rule 131). At this point, appellant is therefore estopped to claim otherwise.

Premises considered, we see no error committed by the trial court in rendering the assailed judgment.

WHEREFORE, the judgment of conviction finding appellant guilty beyond reasonable doubt of the crime charged is hereby AFFIRMED. No costs.

THE PEOPLE OF THE PHILIPPINES, plaintiff-appellee, vs. BASHER BONGCARAWAN y MACARAMBON, accused-appellant.

This is an appeal from the Decision[1] dated December 27, 1999 of the Regional Trial Court of Iligan   City,   Branch   06,   in   Criminal   Case   No.   06-7542,   finding   accused   Basher   Bongcarawan   y Macarambon guilty beyond reasonable doubt of violation of Section 16, Article III of Republic Act No. 6425[2] as amended, and sentencing him to suffer the penalty ofreclusion perpetua, and to pay a fine of Five Hundred Thousand Pesos (P500,000.00) without subsidiary  imprisonment  in case of insolvency.

Accused Basher Bongcarawan y Macarambon was charged  in an  Information which reads, thus:

“That on or about March 13, 1999, in the City of Iligan, Philippines, and within the jurisdiction of this Honorable Court, the said accused, without authority of law, did then and there wilfully, unlawfully and feloniously have in his possession, custody and control eight (8) packs of Methamphetamine 

Hydrochloride, a regulated drug commonly known as Shabu, weighing approximately 400 grams, without the corresponding license or prescription.

Contrary to and in violation of Section 16, Article III of RA 6425, otherwise known as the Dangerous Drugs Act of 1972, as amended by RA 7659.”[3]

During the arraignment, the accused pleaded not guilty.  Trial ensued.

Evidence for the prosecution shows that on March 11, 1999, an interisland passenger ship, M/V Super Ferry 5, sailed from Manila to Iligan City.  At about 3:00 a.m. on March 13, 1999, the vessel was about to dock at the port of Iligan City when its security officer, Mark Diesmo, received a complaint from passenger Lorena Canoy about her missing jewelry. Canoy suspected one of her co-passengers  at   cabin  no.  106  as   the  culprit.  Diesmo and   four   (4)  other  members  of   the  vessel security force accompanied Canoy to search for the suspect whom they later found at the economy section.[4] The   suspect   was   identified   as   the   accused,   Basher   Bongcarawan.   The   accused   was informed of the complaint and was invited to go back to cabin no. 106.  With his consent, he was bodily searched, but no jewelry was found.  He was then escorted by two (2) security agents back to the economy section to get his baggage.  The accused took a Samsonite suitcase and brought this back to the cabin.  When requested by the security, the accused opened the suitcase, revealing a brown bag and small plastic packs containing white crystalline substance.  Suspecting the substance to be “shabu,” the security personnel immediately reported the matter to the ship captain and took pictures of the accused beside the suitcase and its contents.  They also called the Philippine Coast Guard   for   assistance.[5] At   about   6:00   a.m.,   Lt.   Robert   Patrimonio,   YN   Aurelio   Estoque,   CD2 Phoudinie Lantao and RM3 Merchardo De Guzman of the Philippine Coast Guard arrived and took custody of the accused and the seized items--the Samsonite suitcase, a brown bag[6] and eight (8) small plastic packs of white crystalline substance.[7] When asked about the contraband articles, the accused explained that he was just requested by a certain Alican “Alex” Macapudi to bring the suitcase to the latter’s brother in Iligan City.[8] The accused and the seized items were later turned over   by   the   coast   guard   to   the   Presidential   Anti-Organized   Crime   Task   Force   (PAOCTF).  Chief Inspector Graciano Mijares and his men brought the accused to the PAOCTF Headquarters,[9] while the packs of white crystalline substance were sent to the NBI Regional Office in Cagayan de Oro City for laboratory examination.  NBI Forensic Chemist Nicanor Cruz later confirmed the substance to be methamphetamine hydrochloride, commonly known as “shabu,” weighing 399.3266 grams.[10]

The accused testified and proffered  his  own version.  On March 11,  1999,  at  about 10:00 p.m., he was in Quiapo, Manila where he met Alican “Alex” Macapudi, a neighbor who has a store in Marawi City.  He was requested by Macapudi to bring a Samsonite suitcase containing sunglasses and watches to Iligan City, and to give it to Macapudi’s brother at the Iligan port. He boarded the M/V Super Ferry 5 on the same night,  carrying a big  luggage full  of clothes, a small   luggage or “maleta” containing the sunglasses and brushes he bought from Manila, and the Samsonite suitcase of Macapudi.[11] He stayed at cabin no. 106.  At about 4:00 a.m of March 13, 1999, as the vessel was about to dock at the Iligan port, he took his baggage and positioned himself at the economy section to   be   able   to   disembark   ahead   of   the   other   passengers.  There,   he   met   a   friend,   Ansari Ambor.  While they were conversing, five (5) members of the vessel security force and a woman whom he recognized as his co-passenger at cabin no. 106 came and told him that he was suspected of   stealing   jewelry.  He  voluntarily  went   with   the  group   back   to   cabin  no.   106   where   he  was 

9

Page 10: Search and Seizure Cases

frisked.  Subsequently, he was asked to get his baggage, so he went back to the economy section and took the big luggage and Macapudi’s Samsonite suitcase.  He left the small “maleta” containing sunglasses and brushes for fear that they would be confiscated by the security personnel.  When requested, he voluntarily opened the big luggage, but refused to do the same to the Samsonite suitcase which he claimed was not his and had a secret combination lock.  The security personnel forcibly  opened  the   suitcase  and  found packs  of  white  crystalline  substance   inside  which   they suspected to be “shabu.” They took pictures of him with the merchandise, and asked him to sign a turn over receipt which was later given to the Philippine Coast Guard, then to the PAOCTF.[12]

On December 27, 1999, the trial court rendered judgment, the dispositive portion of which reads:

“WHEREFORE, the court finds the accused Basher Bongcarawan y Macarambon GUILTY beyond reasonable doubt as principal of the offense of violation of Section 16, Art. III, R.A. No. 6425 as amended by R.A. No. 7659 and hereby imposes upon him the penalty of RECLUSION PERPETUA and a fine of FIVE HUNDRED THOUSAND (P500,000.00) PESOS, without subsidiary imprisonment in case of insolvency.

Having been under preventive imprisonment since March 13, 1999 until the present, the period of such preventive detention shall be credited in full in favor of the accused in the service of his sentence.

The 399.3266 grams of methamphetamine hydrochloride or shabu is hereby ordered delivered to the National Bureau of Investigation for proper disposition.

Hence, this appeal where the accused raises the following assignment of errors:

“I.THE COURT A QUO ERRED IN SO HOLDING THAT THE DRUG CONFISCATED IS ADMISSIBLE IN EVIDENCE AGAINST THE ACCUSED/APPELLANT.

II.THE COURT A QUO ERRED IN SO HOLDING THAT THE APPELLANT OWNED THE CONFISCATED EVIDENCE AND THEREFORE ADMISSIBLE IN EVIDENCE AGAINST HIM.“[14]

On the first assignment of error, the accused-appellant contends that the Samsonite suitcase containing   the  methamphetamine  hydrochloride  or   “shabu”  was   forcibly  opened  and  searched without his consent, and hence, in violation of his constitutional right against unreasonable search and seizure.  Any evidence acquired pursuant to such unlawful  search and seizure,  he claims,   is inadmissible in evidence against him.  He also contends that People v. Marti[15] is not applicable in this case because a vessel security personnel is deemed to perform the duties of a policeman.

The contentions are devoid of merit.

The right against unreasonable search and seizure is a fundamental right protected by the Constitution.[16] Evidence acquired in violation of this right shall be inadmissible for any purpose in any proceeding.[17] Whenever this right is challenged, an individual may choose between invoking the constitutional protection or waiving his right by giving consent to the search and seizure.  It should be stressed, however, that protection is against transgression committed by the government 

or   its   agent.  As   held   by   this   Court   in   the   case   of People v. Marti,[18] “[i]n   the   absence   of governmental interference, liberties guaranteed by the Constitution cannot be invoked against the State.”[19] The   constitutional   proscription   against   unlawful   searches   and   seizures   applies   as   a restraint directed only against the government and its agencies tasked with the enforcement of the law.  Thus, it could only be invoked against the State to whom the restraint against arbitrary and unreasonable exercise of power is imposed.[20]

In   the  case  before  us,   the  baggage of   the  accused-appellant  was  searched by   the  vessel security personnel. It was only after they found “shabu” inside the suitcase that they called the Philippine Coast Guard for assistance.  The search and seizure of the suitcase and the contraband items was therefore carried out without government intervention, and hence, the constitutional protection against unreasonable search and seizure does not apply.

There  is  no merit   in  the contention of   the accused-appellant   that   the search and seizure performed by the vessel security personnel should be considered as one conducted by the police authorities for like the latter, the former are armed and tasked to maintain peace and order.  The vessel   security   officer   in   the   case   at   bar   is   a   private   employee   and   does   not   discharge   any governmental function.  In contrast, police officers are agents of the state tasked with the sovereign function of enforcement of the law.  Historically and until now, it is against them and other agents of the state that the protection against unreasonable searches and seizures may be invoked.

On the second assignment of error, the accused-appellant contends that he is not the owner of the Samsonite suitcase and he had no knowledge that the same contained “shabu.”  He submits that without knowledge or intent to possess the dangerous drug, he cannot be convicted of the crime charged.[21]

We are not persuaded.

In a prosecution for illegal possession of dangerous drugs, the following facts must be proven beyond reasonable doubt, viz:  (1) that the accused is in possession of the object identified as a prohibited or a regulated drug;  (2) that such possession is not authorized by law; and (3) that the accused freely and consciously possessed the said drug.[22] The first two elements were sufficiently proven in this case, and were in fact undisputed.  We are left with the third.

As early  as  1910  in  the case of United States v. Tan Misa,[23] this  Court  has ruled that  to warrant conviction, the possession of dangerous drugs must be with knowledge of the accused, or that animus possidendi existed together  with the possession or  control  of  such articles.[24] It  has been   ruled,   however,   that   possession   of   dangerous   drugs   constitutesprima facie evidence   of knowledge or animus possidendi sufficient to convict an accused in the absence of a satisfactory explanation of such possession.[25] Hence, the burden of evidence is shifted to the accused to explain the absence of knowledge or animus possidendi.[26]

In this respect, the accused-appellant has utterly failed.  His testimony, uncorroborated, self-serving   and   incredulous,   was   not   given   credence   by   the   trial   court.  We   find   no   reason   to disagree.  Well-settled is the rule that in the absence of palpable error or grave abuse of discretion on the part of the trial judge, the trial court’s evaluation of the credibility of witnesses will not be disturbed on  appeal.[27] Moreover,  evidence  must  be  credible   in   itself   to  deserve  credence  and weight   in   law.  In   this   case,   the  accused-appellant   admits   that   when   he   was  asked   to  get   his 

10

Page 11: Search and Seizure Cases

baggage, he knew it would be inspected.[28] Why he got the Samsonite suitcase allegedly not owned by him and which had a combination lock known only to the owner remains unclear.  He also claims that he did not present his small “maleta” for inspection for fear that its contents consisting of expensive sunglasses and brushes would be confiscated,[29] but he brought the Samsonite suitcase which is not his and also contained expensive sunglasses, and even watches.[30]

The   things   in   possession   of   a   person   are   presumed   by   law   to   be   owned   by   him.[31] To overcome   this   presumption,   it   is   necessary   to   present   clear   and   convincing   evidence   to   the contrary.  In this case, the accused points to a certain Alican “Alex” Macapudi as the owner of the contraband, but presented no evidence to support his claim. As aptly observed by the trial judge:

“First, who is Alex Macap[u]di aka Ali[c]an Macap[u]di?  Does he really exist or simply a figment of the imagination?  He says that Alex Macap[u]di is a friend and a fellow businessman who has a stall selling sunglasses in Marawi City.  But no witnesses were presented to prove that there is such a living, breathing, flesh and blood person named Alex Macap[u]di who entrusted the Samsonite to the accused.  Surely, if he does exist, he has friends, fellow businessmen and acquaintances who could testify and support the claim of the accused.”[32]

Mere denial of ownership will not suffice especially if, as in the case at bar, it is the keystone of the defense of the accused-appellant.  Stories can easily be fabricated. It will take more than bare-bone allegations to convince this Court that a courier of dangerous drugs is not its owner and has no knowledge or intent to possess the same.

WHEREFORE, the decision of the Regional Trial Court of Iligan City, Branch 06, in Criminal Case No. 06-7542, convicting accused-appellant Basher Bongcarawan of violation of Section 16, Article III of  Republic  Act  No.  6425,   as   amended,   and   sentencing  him  to   suffer   the  penalty  of Reclusion Perpetua and   to   pay   a   fine   of   Five   Hundred   Thousand   Pesos   (P500,000.00)   without   subsidiary imprisonment in case of insolvency, is AFFIRMED.

RICARDO C. VALMONTE AND UNION OF LAWYERS AND ADVOCATES FOR PEOPLE'S RIGHTS (ULAP),petitioners, vs.GEN. RENATO DE VILLA AND NATIONAL CAPITAL REGION DISTRICT COMMAND, respondents.

This is a petition for prohibition with preliminary injunction and/or temporary restraining order, seeking the declaration of checkpoints in Valenzuela, Metro Manila or elsewhere, as unconstitutional and the dismantling and banning of the same or, in the alternative, to direct the respondents to formulate guidelines in the implementation of checkpoints, for the protection of the people.

Petitioner Ricardo C. Valmonte sues in his capacity as citizen of the Republic, taxpayer, member of the Integrated Bar of the Philippines (IBP), and resident of Valenzuela, Metro Manila; while petitioner Union of Lawyers and Advocates for People's Rights (ULAP) sues in its capacity as an association whose members are all members of the IBP.

The factual background of the case is as follows:

On 20 January 1987, the National Capital Region District Command (NCRDC) was activated pursuant to Letter of Instruction 02/87 of the Philippine General Headquarters, AFP, with the mission of conducting security operations within its area of responsibility and peripheral areas, for the purpose of establishing an effective territorial defense, maintaining peace and order, and providing an atmosphere conducive to the social, economic and political development of the National Capital Region. 1 As part of its duty to maintain peace and order, the NCRDC installed checkpoints in various parts of Valenzuela, Metro Manila.

Petitioners aver that, because of the installation of said checkpoints, the residents of Valenzuela are worried of being harassed and of their safety being placed at the arbitrary, capricious and whimsical disposition of the military manning the checkpoints, considering that their cars and vehicles are being subjected to regular searches and check-ups, especially at night or at dawn, without the benefit of a search warrant and/or court order. Their alleged fear for their safety increased when, at dawn of 9 July 1988, Benjamin Parpon, a supply officer of the Municipality of Valenzuela, Bulacan, was gunned down allegedly in cold blood by the members of the NCRDC manning the checkpoint along McArthur Highway at Malinta, Valenzuela, for ignoring and/or refusing to submit himself to the checkpoint and for continuing to speed off inspire of warning shots fired in the air. Petitioner Valmonte also claims that, on several occasions, he had gone thru these checkpoints where he was stopped and his car subjected to search/check-up without a court order or search warrant.

Petitioners further contend that the said checkpoints give the respondents a blanket authority to make searches and/or seizures without search warrant or court order in violation of the Constitution; 2 and, instances have occurred where a citizen, while not killed, had been harassed.

Petitioners' concern for their safety and apprehension at being harassed by the military manning the checkpoints are not sufficient grounds to declare the checkpoints as per se illegal. No proof has been presented before the Court to show that, in the course of their routine checks, the military indeed committed specific violations of petitioners' right against unlawful search and seizure or other rights.

In a case filed by the same petitioner organization, Union of Lawyers and Advocates for People's Right (ULAP) vs. Integrated National Police, 3 it was held that individual petitioners who do not allege that any of their rights were violated are not qualified to bring the action, as real parties in interest.

The constitutional right against unreasonable searches and seizures is a personal right invocable only by those whose rights have been infringed, 4 or threatened to be infringed. What constitutes a reasonable or unreasonable search and seizure in any particular case is purely a judicial question, determinable from a consideration of the circumstances involved. 5

Petitioner Valmonte's general allegation to the effect that he had been stopped and searched without a search warrant by the military manning the checkpoints, without more, i.e., without stating the details of the incidents which amount to a violation of his right against unlawful search and seizure, is not sufficient to enable the Court to determine whether there was a violation of 

11

Page 12: Search and Seizure Cases

Valmonte's right against unlawful search and seizure. Not all searches and seizures are prohibited. Those which are reasonable are not forbidden. A reasonable search is not to be determined by any fixed formula but is to be resolved according to the facts of each case. 6

Where, for example, the officer merely draws aside the curtain of a vacant vehicle which is parked on the public fair grounds, 7 or simply looks into a vehicle, 8 or flashes a light therein, 9 these do not constitute unreasonable search.

The setting up of the questioned checkpoints in Valenzuela (and probably in other areas) may be considered as a security measure to enable the NCRDC to pursue its mission of establishing effective territorial defense and maintaining peace and order for the benefit of the public. Checkpoints may also be regarded as measures to thwart plots to destabilize the government, in the interest of public security. In this connection, the Court may take judicial notice of the shift to urban centers and their suburbs of the insurgency movement, so clearly reflected in the increased killings in cities of police and military men by NPA "sparrow units," not to mention the abundance of unlicensed firearms and the alarming rise in lawlessness and violence in such urban centers, not all of which are reported in media, most likely brought about by deteriorating economic conditions — which all sum up to what one can rightly consider, at the very least, as abnormal times. Between the inherent right of the state to protect its existence and promote public welfare and an individual's right against a warrantless search which is howeverreasonably conducted, the former should prevail.

True, the manning of checkpoints by the military is susceptible of abuse by the men in uniform, in the same manner that all governmental power is susceptible of abuse. But, at the cost of occasional inconvenience, discomfort and even irritation to the citizen, the checkpoints during these abnormal times, when conducted within reasonable limits, are part of the price we pay for an orderly society and a peaceful community.

Finally, on 17 July 1988, military and police checkpoints in Metro Manila were temporarily lifted and a review and refinement of the rules in the conduct of the police and military manning the checkpoints was ordered by the National Capital Regional Command Chief and the Metropolitan Police Director. 10

PEOPLE OF THE PHILIPPINES, appellee, vs. ROBERT CHIU y WAN and MARK ANTHONY MOLINA y DELA PEÑA, accused.ROBERT CHIU y WAN, appellant.

This is an appeal from the Decision[1] of the Regional Trial Court of Quezon City, Branch 95, convicting the appellant Robert Chiu y Wan a.k.a. Robert Chu in Criminal Case No. 98-79368 for violation of Section 16 of Republic Act No. 6425, as amended by Republic Act No. 7659 (possession of methylamphetamine hydrochloride, otherwise known as shabu) and sentencing him to suffer the penalty of reclusion perpetua and to pay a fine of  P1,000,000, and convicting him and accused Mark Anthony Molina in Criminal Case No. 98-79369 for violation of Section 15 of Republic Act No. 6425, as amended by Republic  Act No. 7659,  involving the sale of 1.13 grams of methylamphetamine hydrochloride and sentencing them to suffer the indeterminate penalty from two (2) years, four (4) months, and one (1) day of prision correccional medium, as the minimum penalty, to six (6) years of prision correccional maximum, as the maximum penalty.[2]

The IndictmentsOn November 3, 1998, appellant Robert Chiu was charged of violating Section 16, Article III in 

relation to Section 2(e)(2), Article 1 of Rep. Act No. 6425, as amended by Rep. Act No. 7659, in an Information   docketed   as   Criminal   Case   No.   98-79368.  The   accusatory   portion   of   the   said Information reads:

That on or about the 1st day of November 1998, in Quezon City, Philippines, the said accused, did then and there, wilfully, unlawfully and knowingly possess and/or use 220.40 grams of white crystalline substance containing Methylamphetamine Hydrochloride known as Shabu, a regulated drug without the necessary license and/or prescription, therefore, in violation of said law.[3]

On the same date, another Information docketed as Criminal Case No. 98-79369 was filed, charging the appellant and Mark Anthony Molina for violation of Section 15, Article III in relation to Section 2(e)(f)(o), Article 1 of Rep. Act No. 6425, as amended by Rep. Act No. 7659. The accusatory portion of the Information reads:

That on or about the 1st day of November 1998, in Quezon City, Philippines, the said accused, conspiring, confederating and mutually helping each other, not having been authorized by law to sell, dispense, deliver, transport or distribute any regulated drug, did then and there wilfully and unlawfully sell or offer for sale 1.13 grams of white crystalline substance containing Methylamphetamine Hydrochloride known as Shabu, which is a regulated drug.[4]

The appellant and Mark Anthony Molina were arraigned, assisted by counsel, and pleaded not guilty to the charges against them.

The Case for the Prosecution[5]

Sometime in September 1998, the Central Police District Criminal Investigation Unit, Special Operations Group,  headed by SPO1 Edgardo G.  Fernandez and PO1 Jose R.  Salazar,  conducted surveillance operations on a suspected shabu dealer, Daniel Henares.  In a test-buy operation held on October 5, 1998, Salazar, with the assistance of a civilian informant, bought a sachet of shabu worth P2,000.00 from Henares at his residence in San Juan, Metro Manila.  A few days later, on October 11, 1998,[6] Henares was apprehended for the said sale.[7]

During   the   tactical   interrogation  conducted   by   the   policemen,   Henares   admitted   that  he acquired the illegal drugs from appellant Robert Chiu, a resident of No. 29 North Road, Barangay Bagong Lipunan, Cubao, Q.C.[8] Fernandez and other police officers of the Special Operations Group conducted surveillance operations at the appellant’s residence. They learned that Molina’s father owned the house that the appellant was renting.[9] The police officers then decided to conduct a test-buy operation against the appellant. Fernandez and Salazar were designated as the poseur-buyers.  A female informant, who also happened to be a “close friend” of the appellant, would then introduce them to the latter.

At about 8:00 p.m. of October 19, 1998, Fernandez and Salazar,  together with the female informant, proceeded to the house at No. 29 North Road, Barangay Bagong Lipunan, Cubao, Quezon 

12

Page 13: Search and Seizure Cases

City. When the guard on duty saw the female informant, he opened the gate and led her, Salazar and Fernandez to the house, which was approximately fifteen meters from the road.  The informant introduced   Fernandez   and   Salazar   to   the   appellant   as   buyers   of   shabu.   Salazar   was   able   to purchase P3,000.00 worth of the prohibited drug from the appellant. PNP Forensic Chemist Edwin Zata examined the drugs and submitted Physical Sciences Report No. D-3418-98, which stated that the drug gave positive results for methylamphetamine hydrochloride, a regulated drug.[10]

On October 26, 1998, Fernandez filed with the RTC of Pasay City an application for a search warrant  for   the search of   the house at  No.  29 North Road, Barangay Bagong Lipunan (Crame), Cubao, Quezon City, entitled and docketed as People vs. Robert Chiu, Search Warrant No. 98-0059.[11] Attached to the application were the following: (a) Fernandez’ affidavit showing that the house subject of the search was occupied by the appellant; (b) the deposition of Salazar; [12] (c) the request for the examination of 2.19 grams of shabu earlier purchased from the appellant; (d) the results of the forensic examination;[13] and, (e) a sketch of the house, prepared by Salazar.[14]

On October 26, 1998, Executive Judge Lilia C. Lopez of the RTC of Pasay City, Branch 109, conducted an  inquiry   into  the application.  Fernandez  testified that  although the subject  of   the search and the objects to be seized were located in Quezon City, the application for the search warrant was filed in Pasay City because of the possibility that the regulated drug would be removed therefrom by the appellant.[15] Moreover, there was a need for confidentiality; if the policemen filed their application in the RTC of Quezon City, there was a possibility that the information would reach Molina and the appellant. Salazar gave the same response when questioned by the court.[16] The court then issued an Order granting the application and issued Search Warrant No. 98-0059[17] which commanded the search any time of the day or night of the house at No. 29 North Road, Barangay Bagong Lipunan (Crame),  Cubao, Quezon City,  and to seize the substances,  articles and objects therein described.[18]

To make certain that the appellant was in the house to be searched when the search warrant was to be implemented, police operatives led by Fernandez and Salazar decided to conduct another buy-bust   operation   against   the   appellant.  Fernandez   prepared   a P1,000.00   peso   bill   for   the purchase of shabu and placed his initials thereon.[19]

On November  1,  1998,  Fernandez,  Salazar,   two  other  officers  and   the   female   informant, stationed   themselves   at   the   Petron   gasoline   station   located   two   blocks   from   the   subject premises.  At about 9:00 a.m., Salazar and the lady informant proceeded to the house at No. 29 North Road, Barangay Bagong Lipunan, Cubao, Quezon City.  The security guard opened the gate and   led   the   two   inside.  Salazar   informed   the   appellant   that   he   wanted   to   buy   shabu worth P1,000.00 because he had a prospective buyer.  The appellant asked Molina to get the shabu from the room upstairs.  The latter did as he was told and when he returned, handed over to the appellant a plastic sachet containing approximately 1.13 grams of the white crystalline substance which, in turn, was handed over to Salazar.[20] Salazar later reported to Fernandez that the appellant was in the house. Armed with the search warrant, Fernandez, Salazar, PO1 Gerardo Granado, PO1 Corpuz and other police operatives forthwith proceeded to the house. The appellant was just about to leave.  Fernandez and the members of the team identified themselves as police officers and told the appellant that they were in the house to execute the search warrant issued by Judge Lopez. They showed the warrant to the appellant.  After the appellant read the same, Fernandez suggested that the appellant voluntarily surrender the articles and substances listed therein.  The appellant 

accompanied Salazar and Fernandez to the second floor and pointed to his room where the shabu was kept.[21]

Fernandez  had  Barangay  Chairman  Emmanuel  Gozun and  Barangay  Kagawad Oscar   Joves summoned   to   the  house   to  witness   the   search   to  be   conducted.  When   the  barangay  officials arrived, Fernandez, Salazar and the other police officers, accompanied by the barangay officials, searched the rooms of the house and found a Giordano bag containing the following items:

1.       undetermined quantity of white crystalline granules placed inside a transparent plastic envelope;2.       one (1) pc. weighing scale;3.       one (1) cal. .38 revolver “ARMSCOR” bearing SN 71539;4.       five (5) rds. of cal. 38 ammunitions;5.       two (2) rolls of aluminum foils;6.       seven (7) pcs. tooter;7.       one (1) pc. forceps.8.       one (1) bottle of ethyl alcohol;9.       seven (7) pcs. lighter;10.     several pcs. of transparent plastic envelopes;11.     three (3) pcs. (sic) of scissors.[22]

During the search, the appellant and Mark Anthony Molina stayed in the sala. The appellant and Mark Anthony Molina were brought to Camp Karingal, Quezon City, where they were detained.  An Inventory[23] of the articles seized based on the search warrant was prepared in the presence of the barangay  officials.   The  appellant  did  not   sign   the   inventory  but   signed  an  Affidavit  of  Orderly Search[24] in the presence of the barangay chairman and barangay kagawad. Fernandez, Salazar and Granado executed a Joint Affidavit of Apprehension.[25]

On November 1, 1998, Police Superintendent Cecilio Aguila transmitted to the PNP Crime Laboratory   two   heat-sealed   plastic   bags   containing   white   crystalline   substances   weighing   1.13 grams,[26] and another sachet containing white crystalline substances weighing 220.40 grams.[27] The first  sachet  contained the shabu purchased by Salazar  on November  1,  1998 while   the second sachet  contained  the  shabu which  Fernandez  and Salazar   found when  the  search  warrant  was implemented.   As   requested,   Forensic   Chemist   Isidro   Cariño   conducted   a   qualitative examination[28] of about 10 grams of the 220.40 grams contained in the second sachet and of the substances contained in the first sachet. He signed Physical Sciences Report No. D-3594-98 stating that the specimens gave positive results for methylamphetamine hydrochloride.[29]

The Case for the AppellantThe appellant  testified and adduced documentary evidence that he and his wife,  Macrina 

Chiu, were residents of No. 29-B Times Street, West Triangle, Quezon City. He was a naturalized Filipino citizen. He and his wife had been renting the said house from its owner, Aurora Perez, since November 1, 1987.[30] However, when they could no longer afford to pay the monthly rental, he and his  family were  impelled to transfer  to his  parents’  house at Estacio Street,  Sta.  Mesa Heights, Quezon City.  His wife operated a beauty parlor while he was engaged in the business of buying and selling   motor   vehicles,   motorbikes   and   generators   for   which   he   earned   between P40,000.00 to P60,000.00 a month.  The appellant first met Mark Anthony Molina in 1997, and thereafter, was a frequent visitor at No. 29 North Road, Barangay Bagong Lipunan, Cubao, Quezon City.  He had sold Molina’s motorbike but had not yet taken delivery thereof because it needed some repairs.

The appellant testified that he was out with friends in the evening of October 31, 1998.  At about 5:00 a.m. the next day, or on November 1, 1998, he was on his way home. However, since the members of his household were probably still asleep and no one would open the gate for him, he 

13

Page 14: Search and Seizure Cases

decided to go to Molina’s house at North Road and find out how the repairs on the motorcycle were coming along. He stayed there until  late that morning. He had P5,000.00 in his wallet which he intended to lend to his friend who lives in the Molina compound.  He also intended to visit  the graves of his loved ones at the cemetery.

When the appellant was about to leave at 8:30 a.m., Fernandez and Salazar barged into the house, identified themselves as policemen and demanded to know if he was Robert Chiu.  When he replied that he was, the policemen handcuffed him. The police officers went to the second floor where they herded Molina,  his  son and his  girlfriend to  the ground floor.  The policemen were carrying  a  Giordano bag.  Fernandez had Salazar   fetch  Barangay Chairman Gozun and Barangay Kagawad Joves. The appellant then signed the Inventory Report and the Affidavit of Orderly Search. The policemen confiscated two guns, one of which was placed in a box.  The appellant and Molina were then brought to Camp Karingal on board the latter’s L-300 van. Fernandez then divested him of his wallet, and was told that he would be released if he could furnish information on the dealings and   whereabouts   of   a   drug   pusher   named   Palit   Ulo.   He   pleaded   to   Fernandez   to   return the P5,000.00 to him, but Fernandez got a chair and hit him with it.  The appellant parried the chair with his left elbow. Fernandez then brought him out of the room.  When he asked Fernandez what he wanted, the latter remained silent. Fernandez later returned him to the sala.  At 9:30 p.m. that evening, he and Molina were brought to the police station for inquest.

The appellant further testified that on October 19, 1998, he was in Alabang visiting a friend. He denied selling shabu to Salazar in the house at North Road.

The Case for the Accused Mark Anthony Molina

Molina   testified   that   he   was   the   Vice-President   for   Operations   of   the   ARB   Construction Company,   a   family   corporation   which   developed   residential   subdivisions   in   Las   Piñas,   Cavite, Novaliches and Muntinlupa. He was also a member of the Board of Directors of the Immaculada Concepcion Colleges in Bacoor, Cavite, also owned and controlled by his family. As vice-president of the ARB Construction Company, he received P20,000.00 a month, and as a member of the Board of Directors of the school, he received P12,000.00 a month. He had a six-bedroom house in Soldier’s Hill, Muntinlupa City, constructed on a 1,400-square-meter lot.  He had it rented for P20,000.00 a month since 1994 as he and his wife Ditas Alcorez had by then separated.  He and his four-year-old son lived with his parents at No. 54 Van Durren, North Greenhills, San Juan, Metro Manila.

The office of the ARB Construction Company was located at No. 27 North Road, Barangay Bagong Lipunan, Cubao, Q.C., adjacent to the office at No. 29 North Road, was a two-storey house owned by the company. There were two bedrooms in the second floor where he and his son Miguel Raphael and his girlfriend Rosemarie “Pinky” Abaya slept. There was a sala and a kitchen on the ground   floor,   and   there   was  a  motor   pool  beside   the   house.   The   ARB  Construction   Company engaged the services of the Viscayno Security Agency to provide security services to the office. Rodelito Adriano was assigned to guard the house at No. 29 North Road.

Molina stated that he met the appellant Chiu for the first time when they were introduced to each other by Chester Tan, a dealer of computer equipments. As the appellant was an electrician and a very good salesman of used cars, they became business partners. The appellant often went to his house at No. 29 North Road, and even used to sleep there two or three times a week.  Molina confirmed that the appellant had just sold his 1957 Model EMW motorcycle.

In the afternoon of October 30, 1998, a certain Mang Elio visited Molina at No. 29 North Road.  He was carrying a plastic bag and wanted to see the motorcycle in the garage. Shortly after midnight, Mang Elio told  Molina that he was leaving to visit a relative. Mang Elio left his plastic bag, and Abaya later took it. In the meantime, Molina sought the appellant’s help in putting up his 1957 Model   EMW   motorcycle   for   sale.   The   appellant   arrived   in   the   house   in   the   early   morning   of November 1, 1998. Molina, his son, and Abaya, were still asleep in one of the rooms at the second floor. At about 9:00 a.m., Fernandez suddenly barged into the room and identified himself as a policeman.  Fernandez   pointed   a   gun   at   Molina   and   ordered   the   three   of   them   to   go downstairs.  They did as they were told and in the sala, saw Salazar, security guards Adriano and Cortes, and the appellant, who was already handcuffed.

Fernandez showed Molina the search warrant issued by Judge Lopez. Momentarily, Joves and Gozun arrived and witnessed the police officers search the two bedrooms in the second floor.  After thirty minutes, the barangay officials and the police officers came down, carrying with them the Giordano bag left by Mang Elio.  The policemen prepared an inventory of the items contained in the bag.

Molina complained that one of his guns which was taken by the policemen was not included in the inventory.  The policemen demanded P200,000.00 from him, and had him call his mother, but the  latter refused to give money.  Molina,  his  maid,  the appellant,  and Abaya, were brought to Camp Karingal on board the L-300 van owned by the ARB Construction Company.  Fernandez later asked   him   to   testify   against   the   appellant   and   declare   that   the   latter   owned   the   plastic bag.  Fernandez warned that he would be charged for the sale of shabu if he refused to do so. Molina did not accept Fernandez’ offer.

Molina testified that he had no idea whether shabu was sold to Salazar in the morning of November 1, 1998.

On January 27, 2000, the trial court promulgated a decision finding the appellants guilty as charged. The dispositive portion reads:

WHEREFORE, judgment is hereby rendered in the following:

1.       In Crim. Case No. Q-98-79368, the Court finds the accused Robert Chiu y Wan GUILTY beyond reasonable doubt of the offense of Violation of Section 16 of Republic Act 6425, as amended by Republic Act 7659, involving the illegal possession of 220.40 grams of Methylamphetamine Hydrochloride locally known as “shabu,” a regulated drug, and is hereby sentenced to suffer the penalty ofreclusion perpetua and to pay a  FINE of One Million Pesos; and

2.       In Crim. Case No. Q-98-79369, the Court finds both accused, Robert Chiu y Wan and Mark Anthony Molina y dela Peña, GUILTY beyond reasonable doubt of the offense of Violation of Section 15 of Republic Act 6425, as amended by Republic Act 7659, involving the sale of 1.13 grams of Methylamphetamine Hydrochloride locally known as “shabu,” a regulated drug, and are hereby sentenced each to suffer the indeterminate penalty of from two (2) years, four (4) months, and one (1) day of prision correccional medium, as the minimum penalty, to six (6) years of prision correccional maximum, as the maximum penalty.

14

Page 15: Search and Seizure Cases

Both accused are hereby ordered to pay the costs.

The plastic sachets containing Methylamphetamine Hydrochloride, locally known as “shabu” (Exhs. “D-1” and “D-2”) are hereby forfeited in favor of the government and the Branch Clerk of Court is hereby ordered to deliver or cause the safe delivery of the said items to the Dangerous Drugs Board for safekeeping and disposition after the finality of this judgment.[31]

The   appellant   filed   a   motion   for   the   reconsideration   of   the   decision.   According   to   the appellant, Fernandez and Salazar did not adduce evidence before Pasay City Judge Lopez to prove the urgency of issuing a search warrant in a court having jurisdiction other than the place where the said warrant would be enforced. Consequently, any evidence obtained based on the said search warrant   was   inadmissible.   Furthermore,   the   search   warrant   was   antedated.  It   was,   likewise, asserted   that   the   Physical   Sciences   Report   submitted   by   Forensic   Chemist   Isidro   Cariño   was unreliable because of the court’s failure to conduct a qualitative examination of the specimen.

The appellant contended that the prosecution failed to prove his guilt  beyond reasonable doubt for the crime charged because (a) Forensic Chemist Edwin Zata did not testify and identify Physical   Sciences   Report   No.   D-3418-98;[32] (b)   the   appellant   signed   the   Affidavit   of   Orderly Search[33] but   did   not   sign   the   Inventory   Receipt;[34] (c)   the   appellant   was   a   victim  of   extortion perpetrated by Fernandez and Salazar; (d) the collective testimonies of Fernandez and Salazar were incredible;  and,   (e)  Daniel  Henares  was  not   informed of  his   constitutional   rights  when he  was interrogated by the policemen.

The trial court issued an order denying the appellant’s motion. He then appealed the decision. The accused Mark Anthony Molina did not appeal the decision.

In his Brief, the appellant reiterated the grounds in his motion for the reconsideration of the trial court’s decision and assigned the same grounds as errors which merit the Court’s perusal.

The appellant asserts that there was no compelling reason for Fernandez and Salazar to apply for and secure a search warrant from the Executive Judge of the Pasay City RTC. The appellant asserts that confidentiality is not a compelling consideration for urgency contemplated in SC Circular No. 19 dated August 4, 1987, and as held by this Court inMalaloan vs. Court of Appeals,[35] and Ilano vs. Court of Appeals.[36] The appellant finally posits that the application for a search warrant should have   been   filed   in   the   RTC   of   Quezon   City   which   had   primary   jurisdiction   over   the   matter. Consequently, the appellant insists, the search warrant issued by Judge Lopez was defective and the articles/objects seized on the basis thereof were inadmissible in evidence.

For its part, the Office of the Solicitor General asserts that:

The Regional Trial Court of Pasay City correctly issued the search warrant in this case, albeit it was served in Quezon City. It has been settled that there is no law or rule which prohibits a branch of a regional trial court to issue a warrant for the search of a place outside its territorial jurisdiction.  After all, a search warrant is in the nature of a criminal process akin to a writ of discovery, and not a criminal action to be entertained by a court pursuant to its original jurisdiction. Thus, in Ilano v. Court of Appeals (244 SCRA 346 [1995]), this Honorable Court reiterating the ruling in Malaloan, et al. v. Court of Appeals, et al. (232 SCRA 249 [1994]) held that when necessitated and 

justified by compelling considerations of urgency, subject, time and place, a court may issue a search warrant covering a  place outside its territorial jurisdiction. What is important is the strict implementation of the search warrant within the premises specifically described therein which may or may not be within the territorial jurisdiction of the issuing court (Florenz D. Regalado, Remedial Law Compendium, Volume Two, 1995 Edition, pp. 533-535).[37]

The contention of the appellant is barren of merit.

Section 1, SC Circular No. 19 dated August 4, 1987, which was in force when the application for a search warrant was filed, provides viz:

1.       All applications for search warrants relating to violations of the Anti-subversion Act, crimes against public order as defined in the Revised Penal Code, as amended, illegal possession of firearms and/or ammunitions and violations of the Dangerous Drugs Act of 1972, as amended, shall no longer be raffled and shall immediately be taken cognizance of and acted upon by the Executive Judge of the Regional Trial Court, Metropolitan Trial Court, and Municipal Trial Court under whose jurisdiction the place to be searched is located.

In Malaloan vs. Court of Appeals,[38] we held that a search warrant is merely a judicial process designed by the Rules to respond only to an incident in the main case,  if one has already been instituted, or in anticipation thereof.  In the latter contingency, such application for a search warrant may be filed in territorial jurisdiction other than where the illegal articles sought to be seized are located.  We also held that Circular No. 19[39] was never intended to confer exclusive jurisdiction on the Executive Judge mentioned therein; it is not a mandate for the exclusion of all other courts and that a court whose territory does not embrace the place to be searched may issue a search warrant where the application is necessitated and justified by compelling consideration of urgency, subject, time and place, thus:

Evidently, that particular provision of Circular No. 19 was never intended to confer exclusive jurisdiction on said executive judges. In view of the fact, however, that they were themselves directed to personally act on the applications, instead of farming out the same among the other judges as was the previous practice, it was but necessary and practical to require them to so act only on applications involving search of places located within their respective territorial jurisdictions. The phrase above- quoted was, therefore, in the nature of an allocation in the assignment of applications among them, in recognition of human capabilities and limitations, and not a mandate for the exclusion of all other courts…[40]

“Urgent” means pressing; calling for immediate attention.[41] The court must take into account and consider  not  only   the  “subject”  but   the time and place  of   the enforcement  of   the search warrant as well.  The determination of the existence of compelling considerations of urgency, and the subject,  time and place necessitating and  justifying the filing of  an application for a  search warrant   with   a   court   other   than   the   court   having   territorial   jurisdiction   over   the   place   to   be searched and things  to  be seized or  where the  materials  are  found  is  addressed to  the  sound discretion of the trial court where the application is filed, subject to review by the appellate court in case of grave abuse of discretion amounting to excess or lack of jurisdiction.

15

Page 16: Search and Seizure Cases

In this case, Fernandez filed the application for a search warrant with the Pasay City RTC instead of the Quezon City RTC because of the possibility that the shabu would be removed by the appellant   from No.  29  North  Road,  Barangay  Bagong Lipunan,  Cubao,  Quezon City.  Indeed,  as shown by the evidence, the appellant had a residence other than No. 29 North Road where he sold shabu.  There was also the pervading concern of the police officers that if they filed the application in   Quezon   City   where   the   appellant   plied   his   illicit   activities,   it   may   somehow   come   to   the knowledge of Molina and the appellant, thus, rendering the enforcement of any search warrant issued by the court to be a useless effort. We find and so hold that Judge Lopez did not err in taking cognizance of and granting the questioned application for a search warrant.

Additionally, the appellant did not raise, at the trial court, the issues of the validity of the search warrant, the propriety of its enforcement in Quezon City, as well as the admissibility of the shabu against him on the ground that it had been illegally seized. The appellant’s objection to the admissibility   of   the   search   warrant   was   grounded   merely   on   the   “lack   of   veracity   (sic)” thereof.  Such omission constituted a waiver by the appellant of the protection under Section 2, Article II of the Constitution.[42]

The appellant’s contention that the date of the trial court’s issuance of the search warrant which appears  to be “October  21,  1998” was altered  and made to appear “October  26,  1998” without authority from the issuing judge is belied by the records.  Even a cursory reading of the search warrant will  readily show that the date “October 21, 1998” originally typewritten on the search warrant was altered and changed with the authority of Judge Lopez as shown by the latter’s initials beside the date “26th day of October 1998.”  The alteration was authenticated by no less than the Executive Judge herself.

Case law has it that the forensic chemist is not mandated to examine the entire mass of shabu confiscated by the policemen, in this case, 220.40 grams.  It is enough that a sample of the said substance be subjected to qualitative examination.  In People vs. Julian Fernandez,[43] and People vs. Medenilla,[44] we   held   that   a   sample   taken   from   one   package   is   logically   presumed   to   be representative  of   the  entire   contents  of   the  package  unless  proven  otherwise  by   the  accused himself. The appellant failed to adduce such evidence.

There was no need for the prosecution to present Forensic Chemist Edwin Zata because the shabu   he   examined   was   not   the   subject   of   the   cases   filed   against   the   appellant   in   the   trial court.  The prosecution presented Forensic Chemist Isidro Cariño who affirmed the veracity of his report on his examination of the shabu subject of the charges against the appellant.

The bare  fact   that  Daniel  Henares  was not   informed of  his  constitutional  rights  when he confessed to the police officers that the shabu he had sold to Salazar on October 19, 1998 was from the   appellant,   is   irrelevant   in   this   case.   The   appellant   was   not   arrested   on   the   basis   of   the information relayed by Daniel Henares to the police officers.  A “test-buy” operation was conducted against the appellant on October 19, 1998 before the application for a search warrant was filed by the police officers with Judge Lopez on October 26, 1998.   The appellant was arrested by the police officers   only   after   the   sale   of   shabu   by   the   appellant   to   Salazar   on   March   1,   1998   and   the subsequent implementation of the search warrant on the said date.

IN LIGHT OF ALL THE FOREGOING, the appeal is DISMISSED.  The Decision of the Regional Trial Court of Quezon City, Branch 95, in Criminal Cases Nos. 98-79368 and 98-79369 is AFFIRMED.  Costs against the appellant.

REPUBLIC OF THE PHILIPPINES, represented by PRESIDENTIAL COMMISSION ON GOOD GOVERNMENT, petitioner, vs.SANDIGANBAYAN, SIPALAY TRADING CORPORATION and ALLIED BANKING CORPORATION, respondents.

Neither can it be validly argued by the PCGG that its authority to issue a search and seizure order possessing the essential features of a search warrant is derivable from subparagraphs (b) and (c) of Section 3 of E.O. No. 1 or from No. 4 of the simplified enumeration in the “Cojuangco” case, by implication. “Baseco” has clarified once and for all the essential nature of the provisional measures of sequestration, freeze orders and provisional takeover that the PCGG is explicitly equipped with:

“As thus described, sequestration, freezing and provisional takeover are akin to the provisional remedy of preliminary attachment, or receivership.  By attachment, a sheriff seizes property of a defendant in a civil suit so that it may stand as security for the satisfaction of any judgment that may be obtained, and not disposed of, or dissipated, or lost intentionally or otherwise, pending the action.  By receivership, property, real or personal, which is subject of litigation, is placed in the possession and control of a receiver appointed by the Court, who shall conserve it pending final determination of the title or right or possession over it.  All these remedies- sequestration, freezing, provisional takeover, attachment and receivership-are provisional, temporary, designed for particular exigencies, attended by no character or permanency or finality, and always subject to the control of the issuing court or agency.”

Attachment and receivership are legal processes purely conservatory in character, not involving an active and drastic intrusion into and confiscation of properties as what a search warrant (or search and seizure order) necessarily entails.  All processes that the PCGG is allowed to issue in discharging the duty for which it was created, therefore, ought to be viewed strictly in this context.  And this finds   further   support   in “Philippine Coconut Producers Federation, Inc. [COCOFED] v. PCGG”[68] where the Court stressed anew that:

“The question of the validity of PCGG sequestration and freeze orders as provisional measures to collect and conserve the assets believed to be ill-gotten wealth has been laid to rest in BASECO vs. PCGG (150 SCRA 181) where this Court held that such orders are not confiscatory but only preservative in character, not designed to effect a confiscation of, but only to conserve_properties believed to be ill-gotten wealth of the ex-president, his family and associates, and to prevent their concealment dissipation, or transfer, pending the determination of their true- ownership.” (Italics supplied)

Being in fact a search warrant, the SEARCH AND SEIZURE ORDER cannot escape, and must pass, the acid test for validity as provided by the prevailing constitution under which it was issued - the   FREEDOM   CONSTITUTION   which   adopted   verbatim   the   provision   of   the   1973   Constitution (Section 3, Article IV) relating to search warrants, to wit:

16

Page 17: Search and Seizure Cases

“The right of the people to be secure in their persons, houses, papers, and effects against unreasonable searches and seizures of whatever nature and for any purpose shall not be violated, and no search warrant or warrant of arrest shall issue except upon probable cause to be determined by the judge, or such other responsible officer as may be authorized by law, after examination under oath or affirmation of the complainant and the witnesses he may produce, and particularly describing the place to be searched, and the person or things to be seized.”

Supporting jurisprudence thus outlined the following requisites for a search warrant’s validity, the absence of even one will cause its downright nullification:

(1)     it must be issued upon probable cause;

(2)     the probable cause must be determined by the judge himself and not by the applicant or any other person;

(3)     in the determination of probable cause, the judge must examine, under oath or affirmation, the complainant and such witnesses as the latter may produce; and

(4) the warrant issued must particularly describe the place to be searched and persons or things to be seized.[69]

In addition to its unauthorized issuance (as just discussed), the SEARCH AND SEIZURE ORDER is so constitutionally defective.

Firstly, as it suffered from the same inherent weakness or emptiness as that which marred dr. Doromal’s testimony (as earlier discussed extensively). deceased Commissioner Bautista’s in-court declarations did not in any way establish probable cause which has been consistently defined as:

“x x x such facts and circumstances which would lead a reasonably discreet and prudent man to believe that an offense has been committed, and that objects sought in connection with the offense are in the place sought to be searched.  This probable cause must be shown to be within the personal knowledge of the complainant or the witnesses he may produce and not based on mere hearsay.”[70]

This   is   so  because,  as  what  her   testimony  irresistibly   suggested,   the  purported   facts  and circumstances supporting the order are exclusively traceable from documents she identified but which were never formally offered in evidence in the SANDIGANBAYAN.  She never testified to any fact of her own personal knowledge to bolster the PCGG’s claim that ALLIED was in possession and control of illegally-amassed wealth by Lucio Tan.  Her testimony, therefore, is plain hearsay, self-serving, or uncorroborated suspicion.  And the rule is that search warrants are not issued on loose, vague or doubtful basis of fact, nor on mere suspicion or belief.[71]

Secondly,   the   PCGG   has   no   authority   to   issue   the   order   in   the   first   place.   Only a “judge” and “such other responsible officer as may be authorized by law” were empowered by 

the FREEDOM CONSTITUTION to do so, and the PCGG is neither.  It is not a judge, as clarified by the Court in “Baseco,” to wit:

“It should also by now be reasonably evident from what has thus far been said that the PCGG is not, and was never intended to act as, judge. Its general function is to conduct investigations in order tocollect evidence establishing instances of ‘ill-gotten wealth’; issue sequestration, and such orders as may be warranted by the evidence thus collected and as may be necessary to preserve and conserve the assets of which it takes custody and control and prevent their disappearance, loss or dissipation; and eventually file and prosecute in the proper court of competent jurisdiction all cases investigated by it as may be warranted by its findings. It does not try and decide, or hear and determine, or adjudicate with any character of finality or compulsion, cases involving the essential issue of whether or not property should be forfeited and transferred to the State because ‘ill-gotten’ within the meaning of the Constitution and the executive orders.  This function is reserved to the designated court, in this case, the Sandiganbayan.  There can therefore be no serious regard accorded to the accusation, leveled by BASECO, that the PCGG plays the perfidious role of prosecutor and judge at the same time.” (Italics supplied.)

And the PCGG cannot be considered as “such other responsible officer as may be authorized by law” because Executive Order No. 1, to reiterate, did not expressly nor impliedly grant the PCGG the power to issue search warrants/orders.

Thirdly,   the   order   does   not   provide   a   specification   of   the   documents   sought   to   be searched/seized from ALLIED. The body thereof, to quote again, reads:

“By virtue of the powers vested in the Commission by the President of the Republic of the Philippines, you are hereby directed to submit for search and seizure all bank documents in the aforementioned premises which our representative may find necessary and relevant to the investigation being conducted by this Commission.

It expressly refers to “all bank documents” which is too all-embracing, the obvious intent of which is to subject virtually all records pertaining to all business transactions of ALLIED of whatever nature, to search and seizure.  Such tenor of a seizure warrant is not a particular description,[72] thus contravening the explicit  command of the Constitution that there be a particular description of things to be seized.[73] Being a general warrant, the SEARCH AND SEIZURE ORDER is constitutionally objectionable[74] and to be more precise, void for lack of particularity[75] We end our discussion on this matter with the Court’s admonition in “People v. Veloso.”[76]

“A search warrant must conform strictly to the requirements of the constitutional and statutory provisions under which it was issued.  Otherwise, it is void.  The proceedings upon search warrants, it has rightly been held, must be absolutely legal, ‘for there is not a description of process known to the law, the execution of which is more distressing to the citizen.  Perhaps there is none which excites such intense feeling in consequence of its humiliating and degrading effect.’ The warrant will always be construed strictly without, however, going the full length of requiring technical accuracy.  No presumptions of regularity are to be invoked in aid of the process when an officer undertakes to justify under it.”

17

Page 18: Search and Seizure Cases

The third key issue should therefore be answered in the affirmative, i.e., the nullification of the sequestration and search and seizure orders was in order.

The last key issue involves another constitutional imperative - i.e., that the corresponding suit or   suits   against   a   sequestered   entity   of   entities   should   be   brought   in   the   proper   court,   the Sandiganbayan to be precise, [77] within the prescribed period - failure of which automatically lifts the   sequestration   order   or   orders   issued.  Up   for   determination   is   whether   under   the   factual features  of   the   case,   there  was  compliance  with   this   rule  as  professed  by   the  PCGG,  or  non-observance thereof, as argued and declared by respondent corporations and the SANDIGANBAYAN, respectively.  Stress should be given to the fact that the Court’s resolution of this crucial issue would particularly apply to SIPALAY inasmuch as it involves a sequestration order - not to ALLIED against whom   was   issued   a   search   and   seizure   order   that   we   have   just   heretofore   declared   as void.  Nonetheless,   for simplicity’s  sake,  such resolution can be made to cover ALLIED’s case as well.  We thus forego with the distinction in this instance and assume that ALLIED was sequestered via sequestration order similar to that issued against SIPALAY.

ESTEBAN MORANO, CHAN SAU WAH and FU YAN FUN, petitioners-appellants, vs.HON. MARTINIANO VIVO in his capacity as Acting Commissioner of Immigration, respondent-appellant.

Chan Sau Wah, a Chinese citizen born in Fukien, China on January 6, 1932, arrived in the Philippines on November 23, 1961 to visit her cousin, Samuel Lee Malaps. She left in mainland China two of her children by a first marriage: Fu Tse Haw and Fu Yan Kai With her was Fu Yan Fun, her minor son also by the first marriage, born in Hongkong on September 11, 1957.

Chan Sau Wah and her minor son Fu Yan Fun were permitted only into the Philippines under a temporary visitor's visa for two (2) months and after they posted a cash bond of P4,000.00.

On January 24, 1962, Chan Sau Wah married Esteban Morano, a native-born Filipino citizen. Born to this union on September 16, 1962 was Esteban Morano, Jr.

To prolong their stay in the Philippines, Chan Sau Wah and Fu Yan Fun obtained several extensions. The last extension expired on September 10, 1962.1äwphï1.ñët

In a letter dated August 31, 1962, the Commissioner of Immigration ordered Chan Sau Wah and her son, Fu Yan Fun, to leave the country on or before September 10, 1962 with a warning that upon failure so to do, he will issue a warrant for their arrest and will cause the confiscation of their bond.

Instead of leaving the country, on September 10, 1962, Chan Sau Wah (with her husband Esteban Morano) and Fu Yan Fun petitioned the Court of First Instance of Manila for mandamus to compel the Commissioner of Immigration to cancel petitioners' Alien Certificates of Registration; prohibition to stop the Commissioner from issuing a warrant for their arrest, and preliminary injunction to restrain the Commissioner from confiscating their cash bond and from issuing warrants of arrest pending resolution of this case.1 The trial court, on November 3, 1962, issued the writ of preliminary 

injunction prayed for, upon a P2,000-bond. After trial and the stipulations of facts filed by the parties, the Court of First Instance rendered judgment, viz:

IN VIEW OF ALL THE FOREGOING, judgment is hereby rendered as follows:

(a) Granting this petition for Mandamus and Prohibition with respect to petitioner CHAN SAU WAH, who is hereby declared a citizen of the Philippines; ordering the respondent to cancel her Alien Certificate of Registration and other immigration papers, upon the payment of proper dues; and declaring the preliminary injunction with respect to her permanent, prohibiting the respondent, his representatives or subordinates from arresting and/or deporting said petitioner;

(b) Dismissing this petition with respect to petitioner FU YAN FUN, and dissolving the writ of preliminary injunction issued herein, restraining the respondent, his representatives or subordinates from arresting and/or deporting said petitioner;

(c) Authorizing respondent Commissioner to forfeit the bond filed by herein petitioners CHAN SAU WAH and FU YAN FUN in the amount of P4,000.00; and

(d) Denying, for lack of merit, the prayer to declare Sec. 37 (a) of the Philippine Immigration Act of 1940 unconstitutional;

Petitioners and respondent Commissioner both appealed.

We will deal with the claims of both appellants in their proper sequence.

1. The Solicitor General's brief assails the trial court's declaration that Chan Sau Wah is a citizen of the Philippines. The court a quo took the position that "Chan Sau Wah became, by virtue of, and upon, her marriage to Esteban Morano, a natural-born Filipino, a Filipino citizen.2

Placed to the fore is paragraph 1, Section 15 of Commonwealth Act 473 [Revised Naturalization Act], which reads:

Sec. 15. Effect of the naturalization on wife children. — Any woman who is now or may hereafter be married to a citizen of the Philippines, and who might herself be lawfully naturalized shall be deemed a citizen of the Philippines.

To apply this provision, two requisites must concur: (a) valid marriage of an alien woman to a citizen of the Philippines and (b) the alien woman herself might be lawfully naturalized.

We may concede that the first requisite has been properly met. The validity of the marriage is presumed.

18

Page 19: Search and Seizure Cases

But can the same be said of the second requisite? This question by all means is not new. In a series of cases, this Court has declared that the marriage of an alien woman to a Filipino citizen does not ipso facto make her a Filipino citizen. She must satisfactorily show that she has all the qualifications and none of the disqualifications required by the Naturalization Law.3 Ly Giok Ha alias Wy Giok Ha et al. vs. Emilio Galang, L-21332, March 18, 1966,* clearly writes down the philosophy behind the rule in the following expressive language, viz:

Reflection will reveal why this must be so. The qualifications prescribed under section 2 of the Naturalization Act, and the disqualifications enumerated in its section 4, are not mutually exclusive; and if all that were to be required is that the wife of a Filipino be not disqualified under section 4, the result might well be that citizenship would be conferred upon persons in violation of the policy of the statute. For example, section 4 disqualifies only —

"(c) Polygamists or believers in the practice of polygamy; and

(b) Persons convicted of crimes involving moral turpitude,"

so that a blackmailer, or a maintainer of gambling or bawdy houses, not previously convicted by a competent court, would not be thereby disqualified; still it is certain that the law did not intend such a person to, be admitted as a citizen in view of the requirement of section 2 that an applicant for citizenship "must be of good moral character."

Similarly, the citizen's wife might be a convinced believer in racial supremacy, in government by certain selected classes, in the right to vote exclusively by certain "herrenvolk," and thus disbelieve in the principles underlying the Philippine Constitution; yet she would not be disqualified under section 4, as long as she is not "opposed to organized government," nor affiliated to groups "upholding or teaching doctrines opposing all organized governments," nor "defending or teaching the necessity or propriety of violence, personal assault or assassination for the success or predominance of their ideas." Et sic de caeteris.

Upon the principle of selective citizenship, we cannot afford to depart from the wise precept affirmed and reaffirmed in the cases heretofore noted.

In the additional stipulation of facts of July 3, 1963, petitioners admit that Chan Sau Wah is not possessed of all the qualifications required by the Naturalization Law.

Because of all these we are left under no doubt that petitioner Chan Sau Wah did not become a Filipino citizen.

2. Squarely put in issue by petitioners is the constitutionality of Section 37 (a) of the Immigration Act of 1940, which reads:

Sec. 37. (a) The following aliens shall be arrested upon the warrant of the Commissioner of Immigration or of any other officer designated by him for the purpose and deported upon the warrant of the Commissioner of Immigration after a determination by the Board of Commissioners of the existence of the ground for deportation as charged against the alien:

 (7) Any alien who remains in the Philippines in violation of any limitation or condition under which he was admitted as a nonimmigrant.

Petitioners argue that the legal precept just quoted trenches upon the constitutional mandate in Section 1 (3), Article III [Bill of Rights] of the Constitution, to wit:

(3) The right of the people to be secure in their persons, houses, papers, and effects against unreasonable searches and seizures shall not be violated, and no warrants shall issue but upon probable cause, to be determined by the judge after examination under oath or affirmation of the complainant and the witnesses he may produce, and particularly describing the place to be searched, and the persons or things to be seized.

They say that the Constitution limits to judges the authority to issue warrants of arrest and that the legislative delegation of such power to the Commissioner of Immigration is thus violative of the Bill of Rights.

Section 1 (3), Article III of the Constitution, we perceive, does not require judicial intervention in the execution of a final order of deportation issued in accordance with law. The constitutional limitation contemplates an order of arrest in the exercise of judicial power4 as a step preliminary or incidental to prosecution or proceedings for a given offense or administrative action, not as a measure indispensable to carry out a valid decision by a competent official, such as a legal order of deportation, issued by the Commissioner of Immigration, in pursuance of a valid legislation.

The following from American Jurisprudence,5 is illuminating:

It is thoroughly established that Congress has power to order the deportation of aliens whose presence in the country it deems hurtful. Owing to the nature of the proceeding, the deportation of an alien who is found in this country in violation of law is not a deprivation of liberty without due process of law. This is so, although the inquiry devolves upon executive officers, and their findings of fact, after a fair though summary hearing, are made conclusive.

The determination of the propriety of deportation is not a prosecution for, or a conviction of, crime; nor is the deportation a punishment, even though the facts underlying the decision may constitute a crime under local law. The proceeding is in effect simply a refusal by the government to harbor persons whom it does not want. The coincidence of local penal law with the policy of Congress is purely accidental, and, though supported by the same facts, a criminal prosecution and a proceeding for deportation are separate and independent.

19

Page 20: Search and Seizure Cases

In consequence, the constitutional guarantee set forth in Section 1 (3), Article III of the Constitution aforesaid, requiring that the issue of probable cause be determined by a judge, does not extend to deportation proceedings.6

The view we here express finds support in the discussions during the constitutional convention. The convention recognized, as sanctioned by due process, possibilities and cases of deprivation of liberty, other than by order of a competent court.7

Indeed, the power to deport or expel aliens is an attribute of sovereignty. Such power is planted on the "accepted maxim of international law, that every sovereign nation has the power, as inherent in sovereignty, and essential to self-preservation, to forbid the entrance of foreigners within its dominions."8 So it is, that this Court once aptly remarked that there can be no controversy on the fact that where aliens are admitted as temporary visitors, "the law is to the effect that temporary visitors who do not depart upon the expiration of the period of stay granted them are subject to deportation by the Commissioner of Immigration, for having violated the limitation or condition under which they were admitted as non-immigrants (Immigration Law, Sec. 37 (a), subsection (7); C.A. 613, as amended)."9

And, in a case directly in point, where the power of the Commissioner to issue warrants of arrest was challengedas unconstitutional, because "such power is only vested in a judge by Section 1, paragraph 3, Article III of our Constitution," this Court declared —

This argument overlooks the fact that the stay of appellant Ng Hua To as temporary visitor is subject to certain contractual stipulations as contained in the cash bond put up by him, among them, that in case of breach the Commissioner may require the recommitment of the person in whose favor the bond has been filed. The Commissioner did nothing but to enforce such condition. Such a step is necessary to enable the Commissioner to prepare the ground for his deportation under section 37 (a) of Commonwealth Act 613. A contrary interpretation would render such power nugatory to the detriment of the State.10

It is in this context that we rule that Section 37 (a) of the Immigration Act of 1940 is not constitutionally proscribed.

3. A sequel to the questions just discussed is the second error set forth in the government's brief. The Solicitor General balks at the lower court's ruling that petitioner Chan Sau Wah is entitled to permanent residence in the Philippines without first complying with the requirements of Sections 9 and 13 of the Immigration Act of 1940, as amended by Republic Act 503.

We first go to the law, viz: SEC. 9 [last paragraph] An alien who is admitted as a nonimmigrant cannot remain in the Philippines permanently. To obtain permanent admission, a nonimmigrant alien must depart voluntarily to some foreign country and procure from the appropriate Philippine consul the proper visa and thereafter undergo examination by the officers of the Bureau of Immigration at a Philippine port of entry for determination of his admissibility in accordance with the requirements of this Act.

SEC. 13. Under the conditions set forth in this Act there may be admitted into the Philippines immigrants, termed "quota immigrants" not in excess of fifty (50) of any one nationality or without nationality for any one calendar year, except that the following immigrants, termed "nonquota immigrants," maybe admitted without regard to such numerical limitations.

The corresponding Philippine Consular representative abroad shall investigate and certify the eligibility of a quota immigrant previous to his admission into the Philippines. Qualified and desirable aliens who are in the Philippines under temporary stay may be admitted within the quota, subject to the provisions of the last paragraph of section 9 of this Act.

(a) The wife or the husband or the unmarried child under twenty-one years of age of a Philippine citizen, if accompanying or following to join such citizen;

(b) A child of alien parents born during the temporary visit abroad of the mother, the mother having been previously lawfully admitted into the Philippine for permanent residence, if the child is accompanying or coming to join a parent and applies for admission within five years from the date of its birth;

Concededly, Chan Sau Wah entered the Philippines on a tourist-temporary visitor's visa. She is a non-immigrant. Under Section 13 just quoted, she may therefore be admitted if she were a qualified and desirable alien and subject to the provisions of the last paragraph of Section 9. Therefore, first, she must depart voluntarily to some foreign country; second, she must procure from the appropriate consul the proper visa; and third, she must thereafter undergo examination by the officials of the Bureau of Immigration at the port of entry for determination of her admissibility in accordance with the requirements of the immigration Act.

This Court in a number of cases has ruled, and consistently too, that an alien admitted as a temporary visitor cannot change his or her status without first departing from the country and complying with the requirements of Section 9 of the Immigration Act. 11

The gravamen of petitioners' argument is that Chan Sau Wah has, since her entry, married in Manila a native-born Filipino, Esteban Morano. It will not particularly help analysis for petitioners to appeal to family solidarity in an effort to thwart her deportation. Chan Sau Wah, seemingly is not one who has a high regard for such solidarity. Proof: She left two of her children by the first marriage, both minors, in the care of neighbors in Fukien, China.

Then, the wording of the statute heretofore adverted to is a forbidding obstacle which will prevent this Court from writing into the law an additional provision that marriage of a temporary alien visitor to a Filipino would ipso factomake her a permanent resident in his country. This is a field closed to judicial action. No breadth of discretion is allowed us. We cannot insulate her from the State's power of deportation.

Really, it would be an easy matter for an alien woman to enter the Philippines as a temporary visitor, go through a mock marriage, but actually live with another man as husband and wife, and 

20

Page 21: Search and Seizure Cases

thereby skirt the provisions of our immigration law. Also, a woman of undesirable character may enter this country, ply a pernicious trade, marry a Filipino, and again throw overboard Sections 9 and 13 of the Act. Such a flanking movement, we are confident, is impermissible.

Recently we confirmed the rule that an alien wife of a Filipino may not stay permanently without first departing from the Philippines. Reason: Discourage entry under false pretenses. 12

The ruling of the trial court on this score should be reversed.

4. It is petitioners' turn to point as error the dismissal of the petition for mandamus and prohibition with respect to petitioner Fu Yan Fun.

Petitioners' line of thought is this: Fu Yan Fun follows the citizenship of his mother. They cite Section 15, paragraph 3, Commonwealth Act 473, which says that:

A foreign-born minor child, if dwelling in the Philippines at the time of the naturalization of the parent, shall automatically become a Philippine citizen. . . .

Petitioners' position is based on the assumption that Chan Sau Wah, the mother, is a Filipino citizen. We have held that she is not. At best, Fu Yan Fun is a step-son of Esteban Morano, husband of Chan Sau Wah. A step-son is not a foreign-born child of the step-father. The word child, we are certain, means legitimate child, not a step-child. We are not wanting in precedents. Thus, when the Constitution provides that "[t]hose whose fathers are citizens of the Philippines" are citizens thereof, 13 the fundamental charter intends "those" to apply to legitimate children. 14 In another case, the term "minor children" or "minor child" in Section 15 of the Revised Naturalization Law refers only to legitimate children of Filipino citizens. This Court, thru Mr. Chief Justice Roberto Concepcion, there said: 15

It is claimed that the phrases "minor children" and "minor child," used in these provisions, include adopted children. The argument is predicated upon the theory that an adopted child is, for all intents and purposes, a legitimate child. Whenever, the word "children" or "child" is used in statutes, it is generally understood, however, to refer to legitimate children, unless the context of the law and its spirit indicate clearly the contrary. Thus, for instance, when the Constitution provides that "those whose fathers are citizens of the Philippines," and "those whose mothers are citizens of the Philippines" who shall elect Philippine citizenship upon reaching the age of majority, are citizens of the Philippines (Article IV, Section 1, subdivisions [3] and [4]), our fundamental law clearly refers to legitimate children (Chiongbian vs. De Leon, 46 Off. Gaz., 3652-3654; Serra v. Republic, L-4223, May 12, 1952).

At any rate, Fu Yan Fun entered the Philippines as a temporary visitor. The status of a temporary visitor cannot be converted into, that of a permanent resident, as we have heretofore held, without first complying with Section 9 of the Immigration Law.

5. Petitioners finally aver that the lower court erred in authorizing respondent Commissioner to forfeit the bond filed by petitioners Chan Sau Wah and Fu Yan Fun in the amount of P4,000.00.

Here is petitioners' posture. They enjoyed their stay in the Philippines upon a bond. Now they come to court and say that as the prescribed form of this bond was not expressly approved by the Secretary of Justice in accordance with Section 3 of Commonwealth Act 613, which reads —

SEC. 3. . . . He [Commissioner of Immigration] shall issue, subject to the approval of the Department Head, such rules and regulations and prescribes such forms of bond, reports, and other papers, and shall issue from time to time such instruction, not inconsistent with law, as he shall deem best calculated to carry out the provisions of the immigration laws. . . .

that bond is void.

Reasons there are which prevent us from giving our imprimatur to this argument.

The provision requiring official approval of a bond is merely directory. "Irregularity or entire failure in this respect does not affect the validity of the bond. 16 The reason for the rule, is found in 9 C.J., p. 26 (footnote), which reads:

(a) Reason for rule. — "Statutes requiring bonds to be approved by certain officials are not for the purpose of protecting the obligors in the bond, but are aimed to protect the public, to insure their solvency, and to create evidence of an unimpeachable character of the fact of their execution. When they are executed for a legal purpose, before a proper tribunal, and are in fact accepted and approved by the officer or body, whose duty it was to approve them, it could serve no useful purpose of the law to hold them invalid, to release all the obligors thereon, and to defeat every purpose of its execution, simply because the fact of approval was not indorsed precisely as had been directed by the Legislature." American Book Co. vs. Wells, 83 SW 622, 627, 26 Ky L-1159. (emphasis supplied)

And another. This bond was accepted by the government. It had been there. The form of the bond here used is of long continued usage. If the government did not question the form of the bond at all, then we must assume that it counted with the Secretary's approval. For the presumption is that official duty has been legally performed.

Surely enough, equitable considerations will stop petitioners from pleading invalidity of the bond. They offered that bond to enable them to enter and stay in this country. They enjoyed benefits therefrom. They cannot, "in law, and good conscience, be allowed to reap the fruits" of that bond, and then jettison the same. They are "precluded from attacking the validity" of such bond. 17

Actually, to petitioners the bond was good while they sought entry into the Philippines; they offered it as security for the undertaking; that they "will actually depart from the Philippines" when their term of stay expires. Now that the bond is being confiscated because they overstayed, they make an 

21

Page 22: Search and Seizure Cases

about-face and say that such bond is null and void. They shall not profit from this inconsistent position. Their bond should be confiscated.

Conformably to the foregoing, the judgment under review is hereby modified as follows:

(1) The portion thereof which reads:

(a) Granting their petition for Mandamus and Prohibition with respect to petitioner CHAN SAU WAH, who is hereby declared a citizen of the Philippines; ordering the respondent to cancel her Alien Certificate of Registration and other immigration papers, upon the payment of proper dues; and declaring preliminary injunction with respect to her permanent, prohibiting the respondent, his representatives or subordinates from arresting and/or deporting said petitioner;

is hereby reversed: and, in consequence —

The petition for mandamus and prohibition with respect to petitioner Chan Sau Wah is hereby denied; and the judgment declaring her a citizen of the Philippines, directing respondent to cancel her Alien Certificate of Registration and other immigration papers, and declaring the preliminary injunction with respect to her permanent, are all hereby set aside; and

(2) In all other respects, the decision appealed from is hereby affirmed.

JOSE BURGOS, SR., JOSE BURGOS, JR., BAYANI SORIANO and J. BURGOS MEDIA SERVICES, INC.,petitioners, vs.THE CHIEF OF STAFF, ARMED FORCES OF THE PHILIPPINES, THE CHIEF, PHILIPPINE CONSTABULARY, THE CHIEF LEGAL OFFICER, PRESIDENTIAL SECURITY COMMAND, THE JUDGE ADVOCATE GENERAL, ET AL.,

Assailed in this petition for certiorari prohibition and mandamus with preliminary mandatory and prohibitory injunction is the validity of two [2] search warrants issued on December 7, 1982 by respondent Judge Ernani Cruz-Pano, Executive Judge of the then Court of First Instance of Rizal [Quezon City], under which the premises known as No. 19, Road 3, Project 6, Quezon City, and 784 Units C & D, RMS Building, Quezon Avenue, Quezon City, business addresses of the "Metropolitan Mail" and "We Forum" newspapers, respectively, were searched, and office and printing machines, equipment, paraphernalia, motor vehicles and other articles used in the printing, publication and distribution of the said newspapers, as well as numerous papers, documents, books and other written literature alleged to be in the possession and control of petitioner Jose Burgos, Jr. publisher-editor of the "We Forum" newspaper, were seized.

Petitioners further pray that a writ of preliminary mandatory and prohibitory injunction be issued for the return of the seized articles, and that respondents, "particularly the Chief Legal Officer, Presidential Security Command, the Judge Advocate General, AFP, the City Fiscal of Quezon City, their representatives, assistants, subalterns, subordinates, substitute or successors" be enjoined from using the articles thus seized as evidence against petitioner Jose Burgos, Jr. and the other 

accused in Criminal Case No. Q- 022782 of the Regional Trial Court of Quezon City, entitled People v. Jose Burgos, Jr. et al. 1

In our Resolution dated June 21, 1983, respondents were required to answer the petition. The plea for preliminary mandatory and prohibitory injunction was set for hearing on June 28, 1983, later reset to July 7, 1983, on motion of the Solicitor General in behalf of respondents.

At the hearing on July 7, 1983, the Solicitor General, while opposing petitioners' prayer for a writ of preliminary mandatory injunction, manifested that respondents "will not use the aforementioned articles as evidence in the aforementioned case until final resolution of the legality of the seizure of the aforementioned articles. ..." 2 With this manifestation, the prayer for preliminary prohibitory injunction was rendered moot and academic.

Respondents would have this Court dismiss the petition on the ground that petitioners had come to this Court without having previously sought the quashal of the search warrants before respondent judge. Indeed, petitioners, before impugning the validity of the warrants before this Court, should have filed a motion to quash said warrants in the court that issued them. 3 But this procedural flaw notwithstanding, we take cognizance of this petition in view of the seriousness and urgency of the constitutional issues raised not to mention the public interest generated by the search of the "We Forum" offices, which was televised in Channel 7 and widely publicized in all metropolitan dailies. The existence of this special circumstance justifies this Court to exercise its inherent power to suspend its rules. In the words of the revered Mr. Justice Abad Santos in the case of C. Vda. de Ordoveza v. Raymundo, 4 "it is always in the power of the court [Supreme Court] to suspend its rules or to except a particular case from its operation, whenever the purposes of justice require it...".

Respondents likewise urge dismissal of the petition on ground of laches. Considerable stress is laid on the fact that while said search warrants were issued on December 7, 1982, the instant petition impugning the same was filed only on June 16, 1983 or after the lapse of a period of more than six [6] months.

Laches is failure or negligence for an unreasonable and unexplained length of time to do that which, by exercising due diligence, could or should have been done earlier. It is negligence or omission to assert a right within a reasonable time, warranting a presumption that the party entitled to assert it either has abandoned it or declined to assert it. 5

Petitioners, in their Consolidated Reply, explained the reason for the delay in the filing of the petition thus:

Respondents should not find fault, as they now do [p. 1, Answer, p. 3, Manifestation] with the fact that the Petition was filed on June 16, 1983, more than half a year after the petitioners' premises had been raided.

The climate of the times has given petitioners no other choice. If they had waited this long to bring their case to court, it was because they tried at first to exhaust other remedies. The events of the 

22

Page 23: Search and Seizure Cases

past eleven fill years had taught them that everything in this country, from release of public funds to release of detained persons from custody, has become a matter of executive benevolence or largesse

Hence, as soon as they could, petitioners, upon suggestion of persons close to the President, like Fiscal Flaminiano, sent a letter to President Marcos, through counsel Antonio Coronet asking the return at least of the printing equipment and vehicles. And after such a letter had been sent, through Col. Balbino V. Diego, Chief Intelligence and Legal Officer of the Presidential Security Command, they were further encouraged to hope that the latter would yield the desired results.

After waiting in vain for five [5] months, petitioners finally decided to come to Court. [pp. 123-124, Rollo]

Although the reason given by petitioners may not be flattering to our judicial system, We find no ground to punish or chastise them for an error in judgment. On the contrary, the extrajudicial efforts exerted by petitioners quite evidently negate the presumption that they had abandoned their right to the possession of the seized property, thereby refuting the charge of laches against them.

Respondents also submit the theory that since petitioner Jose Burgos, Jr. had used and marked as evidence some of the seized documents in Criminal Case No. Q- 022872, he is now estopped from challenging the validity of the search warrants. We do not follow the logic of respondents. These documents lawfully belong to petitioner Jose Burgos, Jr. and he can do whatever he pleases with them, within legal bounds. The fact that he has used them as evidence does not and cannot in any way affect the validity or invalidity of the search warrants assailed in this petition.

Several and diverse reasons have been advanced by petitioners to nullify the search warrants in question.

1. Petitioners fault respondent judge for his alleged failure to conduct an examination under oath or affirmation of the applicant and his witnesses, as mandated by the above-quoted constitutional provision as wen as Sec. 4, Rule 126 of the Rules of Court . 6 This objection, however, may properly be considered moot and academic, as petitioners themselves conceded during the hearing on August 9, 1983, that an examination had indeed been conducted by respondent judge of Col. Abadilla and his witnesses.

2. Search Warrants No. 20-82[a] and No. 20- 82[b] were used to search two distinct places: No. 19, Road 3, Project 6, Quezon City and 784 Units C & D, RMS Building, Quezon Avenue, Quezon City, respectively. Objection is interposed to the execution of Search Warrant No. 20-82[b] at the latter address on the ground that the two search warrants pinpointed only one place where petitioner Jose Burgos, Jr. was allegedly keeping and concealing the articles listed therein, i.e., No. 19, Road 3, Project 6, Quezon City. This assertion is based on that portion of Search Warrant No. 20- 82[b] which states:

Which have been used, and are being used as instruments and means of committing the crime of subversion penalized under P.D. 885 as amended and he is keeping and concealing the same at 19 Road 3, Project 6, Quezon City.

The defect pointed out is obviously a typographical error. Precisely, two search warrants were applied for and issued because the purpose and intent were to search two distinct premises. It would be quite absurd and illogical for respondent judge to have issued two warrants intended for one and the same place. Besides, the addresses of the places sought to be searched were specifically set forth in the application, and since it was Col. Abadilla himself who headed the team which executed the search warrants, the ambiguity that might have arisen by reason of the typographical error is more apparent than real. The fact is that the place for which Search Warrant No. 20- 82[b] was applied for was 728 Units C & D, RMS Building, Quezon Avenue, Quezon City, which address appeared in the opening paragraph of the said warrant. 7 Obviously this is the same place that respondent judge had in mind when he issued Warrant No. 20-82 [b].

In the determination of whether a search warrant describes the premises to be searched with sufficient particularity, it has been held "that the executing officer's prior knowledge as to the place intended in the warrant is relevant. This would seem to be especially true where the executing officer is the affiant on whose affidavit the warrant had issued, and when he knows that the judge who issued the warrant intended the building described in the affidavit, And it has also been said that the executing officer may look to the affidavit in the official court file to resolve an ambiguity in the warrant as to the place to be searched." 8

3. Another ground relied upon to annul the search warrants is the fact that although the warrants were directed against Jose Burgos, Jr. alone, articles b belonging to his co-petitioners Jose Burgos, Sr., Bayani Soriano and the J. Burgos Media Services, Inc. were seized.

Section 2, Rule 126 of the Rules of Court, enumerates the personal properties that may be seized under a search warrant, to wit:

Sec. 2. Personal Property to be seized. — A search warrant may be issued for the search and seizure of the following personal property:

[a] Property subject of the offense;[b] Property stolen or embezzled and other proceeds or fruits of the offense; and[c] Property used or intended to be used as the means of committing an offense.

The above rule does not require that the property to be seized should be owned by the person against whom the search warrant is directed. It may or may not be owned by him. In fact, under subsection [b] of the above-quoted Section 2, one of the properties that may be seized is stolen property. Necessarily, stolen property must be owned by one other than the person in whose possession it may be at the time of the search and seizure. Ownership, therefore, is of no consequence, and it is sufficient that the person against whom the warrant is directed has control or 

23

Page 24: Search and Seizure Cases

possession of the property sought to be seized, as petitioner Jose Burgos, Jr. was alleged to have in relation to the articles and property seized under the warrants.

4. Neither is there merit in petitioners' assertion that real properties were seized under the disputed warrants. Under Article 415[5] of the Civil Code of the Philippines, "machinery, receptables, instruments or implements intended by the owner of the tenement for an industry or works which may be carried on in a building or on a piece of land and which tend directly to meet the needs of the said industry or works" are considered immovable property. In Davao Sawmill Co. v. Castillo 9 where this legal provision was invoked, this Court ruled that machinery which is movable by nature becomes immobilized when placed by the owner of the tenement, property or plant, but not so when placed by a tenant, usufructuary, or any other person having only a temporary right, unless such person acted as the agent of the owner.

In the case at bar, petitioners do not claim to be the owners of the land and/or building on which the machineries were placed. This being the case, the machineries in question, while in fact bolted to the ground remain movable property susceptible to seizure under a search warrant.

5. The questioned search warrants were issued by respondent judge upon application of Col. Rolando N. Abadilla Intelligence Officer of the P.C. Metrocom. 10 The application was accompanied by the Joint Affidavit of Alejandro M. Gutierrez and Pedro U. Tango, 11 members of the Metrocom Intelligence and Security Group under Col. Abadilla which conducted a surveillance of the premises prior to the filing of the application for the search warrants on December 7, 1982.

It is contended by petitioners, however, that the abovementioned documents could not have provided sufficient basis for the finding of a probable cause upon which a warrant may validly issue in accordance with Section 3, Article IV of the 1973 Constitution which provides:

SEC. 3. ... and no search warrant or warrant of arrest shall issue except upon probable cause to be determined by the judge, or such other responsible officer as may be authorized by law, after examination under oath or affirmation of the complainant and the witnesses he may produce, and particularly describing the place to be searched and the persons or things to be seized.

We find petitioners' thesis impressed with merit. Probable cause for a search is defined as such facts and circumstances which would lead a reasonably discreet and prudent man to believe that an offense has been committed and that the objects sought in connection with the offense are in the place sought to be searched. And when the search warrant applied for is directed against a newspaper publisher or editor in connection with the publication of subversive materials, as in the case at bar, the application and/or its supporting affidavits must contain a specification, stating with particularity the alleged subversive material he has published or is intending to publish. Mere generalization will not suffice. Thus, the broad statement in Col. Abadilla's application that petitioner "is in possession or has in his control printing equipment and other paraphernalia, news publications and other documents which were used and are all continuously being used as a means of committing the offense of subversion punishable under Presidential Decree 885, as amended ..." 12 is a mere conclusion of law and does not satisfy the requirements of probable 

cause. Bereft of such particulars as would justify a finding of the existence of probable cause, said allegation cannot serve as basis for the issuance of a search warrant and it was a grave error for respondent judge to have done so.

Equally insufficient as basis for the determination of probable cause is the statement contained in the joint affidavit of Alejandro M. Gutierrez and Pedro U. Tango, "that the evidence gathered and collated by our unit clearly shows that the premises above- mentioned and the articles and things above-described were used and are continuously being used for subversive activities in conspiracy with, and to promote the objective of, illegal organizations such as the Light-a-Fire Movement, Movement for Free Philippines, and April 6 Movement." 13

In mandating that "no warrant shall issue except upon probable cause to be determined by the judge, ... after examination under oath or affirmation of the complainant and the witnesses he may produce; 14 the Constitution requires no less than personal knowledge by the complainant or his witnesses of the facts upon which the issuance of a search warrant may be justified. In Alvarez v. Court of First Instance, 15 this Court ruled that "the oath required must refer to the truth of the facts within the personal knowledge of the petitioner or his witnesses, because the purpose thereof is to convince the committing magistrate, not the individual making the affidavit and seeking the issuance of the warrant, of the existence of probable cause." As couched, the quoted averment in said joint affidavit filed before respondent judge hardly meets the test of sufficiency established by this Court in Alvarez case.

Another factor which makes the search warrants under consideration constitutionally objectionable is that they are in the nature of general warrants. The search warrants describe the articles sought to be seized in this wise:

1] All printing equipment, paraphernalia, paper, ink, photo (equipment, typewriters, cabinets, tables, communications/recording equipment, tape recorders, dictaphone and the like used and/or connected in the printing of the "WE FORUM" newspaper and any and all documents communication, letters and facsimile of prints related to the "WE FORUM" newspaper.

2] Subversive documents, pamphlets, leaflets, books, and other publication to promote the objectives and piurposes of the subversive organization known as Movement for Free Philippines, Light-a-Fire Movement and April 6 Movement; and,

3] Motor vehicles used in the distribution/circulation of the "WE FORUM" and other subversive materials and propaganda, more particularly,

1] Toyota-Corolla, colored yellow with Plate No. NKA 892; 2] DATSUN pick-up colored white with Plate No. NKV 969 3] A delivery truck with Plate No. NBS 524; 4] TOYOTA-TAMARAW, colored white with Plate No. PBP 665; and,

5] TOYOTA Hi-Lux, pick-up truck with Plate No. NGV 427 with marking "Bagong Silang."

24

Page 25: Search and Seizure Cases

In Stanford v. State of Texas 16 the search warrant which authorized the search for "books, records, pamphlets, cards, receipts, lists, memoranda, pictures, recordings and other written instruments concerning the Communist Party in Texas," was declared void by the U.S. Supreme Court for being too general. In like manner, directions to "seize any evidence in connectionwith the violation of SDC 13-3703 or otherwise" have been held too general, and that portion of a search warrant which authorized the seizure of any "paraphernalia which could be used to violate Sec. 54-197 of the Connecticut General Statutes [the statute dealing with the crime of conspiracy]" was held to be a general warrant, and therefore invalid. 17 The description of the articles sought to be seized under the search warrants in question cannot be characterized differently.

In the Stanford case, the U.S. Supreme Courts calls to mind a notable chapter in English history: the era of disaccord between the Tudor Government and the English Press, when "Officers of the Crown were given roving commissions to search where they pleased in order to suppress and destroy the literature of dissent both Catholic and Puritan Reference herein to such historical episode would not be relevant for it is not the policy of our government to suppress any newspaper or publication that speaks with "the voice of non-conformity" but poses no clear and imminent danger to state security.

As heretofore stated, the premises searched were the business and printing offices of the "Metropolitan Mail" and the "We Forum newspapers. As a consequence of the search and seizure, these premises were padlocked and sealed, with the further result that the printing and publication of said newspapers were discontinued.

Such closure is in the nature of previous restraint or censorship abhorrent to the freedom of the press guaranteed under the fundamental law, 18 and constitutes a virtual denial of petitioners' freedom to express themselves in print. This state of being is patently anathematic to a democratic framework where a free, alert and even militant press is essential for the political enlightenment and growth of the citizenry.

Respondents would justify the continued sealing of the printing machines on the ground that they have been sequestered under Section 8 of Presidential Decree No. 885, as amended, which authorizes "the sequestration of the property of any person, natural or artificial, engaged in subversive activities against the government and its duly constituted authorities ... in accordance with implementing rules and regulations as may be issued by the Secretary of National Defense." It is doubtful however, if sequestration could validly be effected in view of the absence of any implementing rules and regulations promulgated by the Minister of National Defense.

Besides, in the December 10, 1982 issue of the Daily Express, it was reported that no less than President Marcos himself denied the request of the military authorities to sequester the property seized from petitioners on December 7, 1982. Thus:

The President denied a request flied by government prosecutors for sequestration of the WE FORUM newspaper and its printing presses, according to Information Minister Gregorio S. Cendana.

On the basis of court orders, government agents went to the We Forum offices in Quezon City and took a detailed inventory of the equipment and all materials in the premises.

Cendaña said that because of the denial the newspaper and its equipment remain at the disposal of the owners, subject to the discretion of the court. 19

That the property seized on December 7, 1982 had not been sequestered is further confirmed by the reply of then Foreign Minister Carlos P. Romulo to the letter dated February 10, 1983 of U.S. Congressman Tony P. Hall addressed to President Marcos, expressing alarm over the "WE FORUM " case. 20 In this reply dated February 11, 1983, Minister Romulo stated:

2. Contrary to reports, President Marcos turned down the recommendation of our authorities to close the paper's printing facilities and confiscate the equipment and materials it uses. 21

IN VIEW OF THE FOREGOING, Search Warrants Nos. 20-82[a] and 20-82[b] issued by respondent judge on December 7, 1982 are hereby declared null and void and are accordingly set aside. The prayer for a writ of mandatory injunction for the return of the seized articles is hereby granted and all articles seized thereunder are hereby ordered released to petitioners. No costs.

MAXIMO V. SOLIVEN, ANTONIO V. ROCES, FREDERICK K. AGCAOLI, and GODOFREDO L. MANZANAS,petitioners, vs.THE HON. RAMON P. MAKASIAR, Presiding Judge of the Regional Trial Court of Manila, Branch 35, UNDERSECRETARY SILVESTRE BELLO III, of the Department of Justice, LUIS C. VICTOR, THE CITY FISCAL OF MANILA and PRESIDENT CORAZON C. AQUINO, respondents.

In these consolidated cases, three principal issues were raised: (1) whether or not petitioners were denied due process when informations for libel were filed against them although the finding of the existence of a prima faciecase was still under review by the Secretary of Justice and, subsequently, by the President; (2) whether or not the constitutional rights of Beltran were violated when respondent RTC judge issued a warrant for his arrest without personally examining the complainant and the witnesses, if any, to determine probable cause; and (3) whether or not the President of the Philippines, under the Constitution, may initiate criminal proceedings against the petitioners through the filing of a complaint-affidavit.

Subsequent events have rendered the first issue moot and academic. On March 30, 1988, the Secretary of Justice denied petitioners' motion for reconsideration and upheld the resolution of the Undersecretary of Justice sustaining the City Fiscal's finding of a prima facie case against petitioners. A second motion for reconsideration filed by petitioner Beltran was denied by the Secretary of Justice on April 7, 1988. On appeal, the President, through the Executive Secretary, affirmed the resolution of the Secretary of Justice on May 2, 1988. The motion for reconsideration was denied by the Executive Secretary on May 16, 1988. With these developments, petitioners' contention that they have been denied the administrative remedies available under the law has lost factual support.

25

Page 26: Search and Seizure Cases

It may also be added that with respect to petitioner Beltran, the allegation of denial of due process of law in the preliminary investigation is negated by the fact that instead of submitting his counter- affidavits, he filed a "Motion to Declare Proceedings Closed," in effect waiving his right to refute the complaint by filing counter-affidavits. Due process of law does not require that the respondent in a criminal case actually file his counter-affidavits before the preliminary investigation is deemed completed. All that is required is that the respondent be given the opportunity to submit counter-affidavits if he is so minded.

The second issue, raised by petitioner Beltran, calls for an interpretation of the constitutional provision on the issuance of warrants of arrest. The pertinent provision reads:

Art. III, Sec. 2. The right of the people to be secure in their persons, houses, papers and effects against unreasonable searches and seizures of whatever nature and for any purpose shall be inviolable, and no search warrant or warrant of arrest shall issue except upon probable cause to be determined personally by the judge after examination nder oath or affirmation of the complainant and the witnesses he may produce, and particularly describing the place to be searched and the persons or things to be seized.

The addition of the word "personally" after the word "determined" and the deletion of the grant of authority by the 1973 Constitution to issue warrants to "other responsible officers as may be authorized by law," has apparently convinced petitioner Beltran that the Constitution now requires the judge to personally examine the complainant and his witnesses in his determination of probable cause for the issuance of warrants of arrest. This is not an accurate interpretation.

What the Constitution underscores is the exclusive and personal responsibility of the issuing judge to satisfy himself of the existence of probable cause. In satisfying himself of the existence of probable cause for the issuance of a warrant of arrest, the judge is not required to personally examine the complainant and his witnesses. Following established doctrine and procedure, he shall: (1) personally evaluate the report and the supporting documents submitted by the fiscal regarding the existence of probable cause and, on the basis thereof, issue a warrant of arrest; or (2) if on the basis thereof he finds no probable cause, he may disregard the fiscal's report and require the submission of supporting affidavits of witnesses to aid him in arriving at a conclusion as to the existence of probable cause.

Sound policy dictates this procedure, otherwise judges would be unduly laden with the preliminary examination and investigation of criminal complaints instead of concentrating on hearing and deciding cases filed before their courts.

On June 30, 1987, the Supreme Court unanimously adopted Circular No. 12, setting down guidelines for the issuance of warrants of arrest. The procedure therein provided is reiterated and clarified in this resolution.

It has not been shown that respondent judge has deviated from the prescribed procedure. Thus, with regard to the issuance of the warrants of arrest, a finding of grave abuse of discretion amounting to lack or excess of jurisdiction cannot be sustained.

Anent the third issue, petitioner Beltran argues that "the reasons which necessitate presidential immunity from suit impose a correlative disability to file suit." He contends that if criminal proceedings ensue by virtue of the President's filing of her complaint-affidavit, she may subsequently have to be a witness for the prosecution, bringing her under the trial court's jurisdiction. This, continues Beltran, would in an indirect way defeat her privilege of immunity from suit, as by testifying on the witness stand, she would be exposing herself to possible contempt of court or perjury.

The rationale for the grant to the President of the privilege of immunity from suit is to assure the exercise of Presidential duties and functions free from any hindrance or distraction, considering that being the Chief Executive of the Government is a job that, aside from requiring all of the office holder's time, also demands undivided attention.

But this privilege of immunity from suit, pertains to the President by virtue of the office and may be invoked only by the holder of the office; not by any other person in the President's behalf. Thus, an accused in a criminal case in which the President is complainant cannot raise the presidential privilege as a defense to prevent the case from proceeding against such accused.

Moreover, there is nothing in our laws that would prevent the President from waiving the privilege. Thus, if so minded the President may shed the protection afforded by the privilege and submit to the court's jurisdiction. The choice of whether to exercise the privilege or to waive it is solely the President's prerogative. It is a decision that cannot be assumed and imposed by any other person.

As regards the contention of petitioner Beltran that he could not be held liable for libel because of the privileged character or the publication, the Court reiterates that it is not a trier of facts and that such a defense is best left to the trial court to appreciate after receiving the evidence of the parties.

As to petitioner Beltran's claim that to allow the libel case to proceed would produce a "chilling effect" on press freedom, the Court finds no basis at this stage to rule on the point.

The petitions fail to establish that public respondents, through their separate acts, gravely abused their discretion as to amount to lack of jurisdiction. Hence, the writs of certiorari and prohibition prayed for cannot issue.

WHEREFORE, finding no grave abuse of discretion amounting to excess or lack of jurisdiction on the part of the public respondents, the Court Resolved to DISMISS the petitions in G. R. Nos. 82585, 82827 and 83979. The Order to maintain the status quo contained in the Resolution of the Court en banc dated April 7, 1988 and reiterated in the Resolution dated April 26, 1988 is LIFTED.

26

Page 27: Search and Seizure Cases

PEOPLE OF THE PHILIPPINES, petitioner, vs.HONORABLE ENRIQUE B. INTING, PRESIDING JUDGE, REGIONAL TRIAL COURT, BRANCH 38, DUMAGUETE CITY, AND OIC MAYOR DOMINADOR S. REGALADO, JR., respondents.

Does a preliminary investigation conducted by a Provincial Election Supervisor involving election offenses have to be coursed through the Provincial Fiscal now Provincial Prosecutor, before the Regional Trial Court may take cognizance of the investigation and determine whether or not probable cause exists?

On February 6, 1988, Mrs. Editha Barba filed a letter-complaint against OIC-Mayor Dominador Regalado of Tanjay, Negros Oriental with the Commission on Elections (COMELEC), for allegedly transferring her, a permanent Nursing Attendant, Grade I, in the office of the Municipal Mayor to a very remote barangay and without obtaining prior permission or clearance from COMELEC as required by law.

Acting on the complaint, COMELEC directed Atty. Gerardo Lituanas, Provincial Election Supervisor of Dumaguete City: (1) to conduct the preliminary investigation of the case; (2) to prepare and file the necessary information in court; (3) to handle the prosecution if the evidence submitted shows a prima facie case and (3) to issue a resolution of prosecution or dismissal as the case may be. The directive to conduct the preliminary investigation was pursuant to COMELEC Resolution No. 1752 dated January 14, 1986. The resolution, in turn, is based on the constitutional mandate that the COMELEC is charged with the enforcement and administration of all laws relative to the conduct of elections for the purpose of ensuring free, orderly and honest elections (sec. 2, Article XII-C of the 1973 Constitution) and on the Omnibus Election Code which implements the constitutional provision. The Resolution provides, among others:

Further, Regional Election Directors and Provincial Election Supervisors are hereby authorized to conduct preliminary investigations of election offenses committed in their respective jurisdictions, file the corresponding complaints and/or informations in court whenever warranted, and to prosecute the same pursuant to Section 265 of the Omnibus Election Code. (Rollo, p. 15)

After a preliminary investigation of Barba's complaint, Atty. Lituanas found a prima facie case. Hence, on September 26, 1988, he filed with the respondent trial court a criminal case for violation of section 261, Par. (h), Omnibus Election Code against the OIC-Mayor.

In an Order dated September 30, 1988, the respondent court issued a warrant of arrest against the accused OIC Mayor. It also fixed the bail at five thousand pesos (P5,000.00) as recommended by the Provincial Election Supervisor.

However, in an order dated October 3, 1988 and before the accused could be arrested, the trial court set aside its September 30, 1988 order on the ground that Atty. Lituanas is not authorized to determine probable cause pursuant to Section 2, Article III of the 1987 Constitution. The court stated that it "will give due course to the information filed in this case if the same has the written

approval of the Provincial Fiscal after which the prosecution of the case shall be under the supervision and control of the latter." (at p. 23, Rollo, emphasis supplied)

In another order dated November 22, 1988, the court gave Atty. Lituanas fifteen (15) days from receipt to file another information charging the same offense with the written approval of the Provincial Fiscal.

Atty. Lituanas failed to comply with the order. Hence, in an order dated December 8, 1988, the trial court quashed the information. A motion for reconsideration was denied.

Hence, this petition. The respondent trial court justifies its stand on the ground that the COMELEC through its Provincial Election Supervisor lacks jurisdiction to determine the existence of probable cause in an election offense which it seeks to prosecute in court because:

While under Section 265 of the Omnibus Election Code approved on December 3, 1985 duly authorized legal officers of the Commission on Elections have the exclusive power to conduct preliminary investigation of all election offenses and to prosecute the same, it is doubtful whether said authority under the auspices of the 1973 Constitution, still subsists under the 1987 Constitution which has deleted in its Section 2, Article III, the phrase "and such other responsible officer as may be authorized by law" in the equivalent section and article of the 1973 Constitution. (Rollo, p. 24)

The petition is impressed with merit. We emphasize important features of the constitutional mandate that " ... no search warrant or warrant of arrest shall issue except upon probable cause to be determined personally by the judge ... " (Article III, Section 2, Constitution)

First, the determination of probable cause is a function of the Judge. It is not for the Provincial Fiscal or Prosecutor nor for the Election Supervisor to ascertain. Only the Judge and the Judge alone makes this determination.

Second, the preliminary inquiry made by a Prosecutor does not bind the Judge. It merely assists him to make the determination of probable cause. The Judge does not have to follow what the Prosecutor presents to him. By itself, the Prosecutor's certification of probable cause is ineffectual. It is the report, the affidavits, the transcripts of stenographic notes (if any), and all other supporting documents behind the Prosecutor's certification which are material in assisting the Judge to make his determination.

And third, Judges and Prosecutors alike should distinguish the preliminary inquiry which determines probable cause for the issuance of a warrant of arrest from the preliminary investigation proper which ascertains whether the offender should be held for trial or released. Even if the two inquiries are conducted in the course of one and the same proceeding, there should be no confusion about the objectives. The determination of probable cause for the warrant of arrest is made by the Judge. The preliminary investigation proper-whether or not there is reasonable ground to believe that the accused is guilty of the offense charged and, therefore, whether or not he should be subjected to the expense, rigors and embarrassment of trial is the function of the Prosecutor.

27

Page 28: Search and Seizure Cases

The Court made this clear in the case of Castillo v. Villaluz (171 SCRA 39 [1989]):

Judges of Regional Trial Courts (formerly Courts of First Instance) no longer have authority to conduct preliminary investigations. That authority, at one time reposed in them under Sections 13, 14 and 16 Rule 112 of the Rules of Court of 1964, (See Sec. 4, Rule 108, Rules of Court of 1940; People v. Solon, 47 Phil. 443, cited in Moran, Comments on the Rules, 1980 ed., Vol. 4, pp. 115-116) was removed from them by the 1985 Rules on Criminal Procedure, effective on January 1, 1985, (Promulgated on November 11, 1984) which deleted all provisions granting that power to said Judges. We had occasion to point this out in Salta v. Court of Appeals, 143 SCRA 228, and to stress as well certain other basic propositions, namely: (1) that the conduct of a preliminary investigation is "not a judicial function ... (but) part of the prosecution's job, a function of the executive," (2) that wherever "there are enough fiscals or prosecutors to conduct preliminary investigations, courts are counseled to leave this job which is essentially executive to them," and the fact "that a certain power is granted does not necessarily mean that it should be indiscriminately exercised."

The 1988 Amendments to the 1985 Rules on Criminal Procedure, declared effective on October 1, 1988, (The 1988 Amendments were published in the issue of Bulletin Today of October 29, 1988) did not restore that authority to Judges of Regional Trial Courts; said amendments did not in fact deal at all with the officers or courts having authority to conduct preliminary investigations.

This is not to say, however, that somewhere along the line RTC Judges also lost the power to make apreliminary examination for the purpose of determining whether probable cause exists to justify the issuance of a warrant of arrest (or search warrant). Such a power — indeed, it is as much a duty as it is a power — has been and remains vested in every judge by the provision in the Bill of Rights in the 1935, the 1973 and the present (1987) Constitutions securing the people against unreasonable searches and seizures, thereby placing it beyond the competence of mere Court rule or statute to revoke. The distinction must, therefore, be made clear while an RTC Judge may no longer conduct preliminary investigations to ascertain whether there is sufficient ground for the filing of a criminal complaint or information, he retains the authority, when such a pleading is filed with his court, to determine whether there is probable cause justifying the issuance of a warrant of arrest. It might be added that this distinction accords, rather than conflicts, with the rationale of Salta because both law and rule, in restricting to judges the authority to order arrest, recognize that function to be judicial in nature.

We reiterate that preliminary investigation should be distinguished as to whether it is an investigation for the determination of a sufficient ground for the filing of the information or it is an investigation for the determination of a probable cause for the issuance of a warrant of arrest. The first kind of preliminary investigation is executive in nature. It is part of the prosecution's job. The second kind of preliminary investigation which is more properly called preliminary examination is judicial in nature and is lodged with the judge. It is in this context that we address the issue raised in the instant petition so as to give meaning to the constitutional power vested in the COMELEC regarding election offenses.

Article IX C Section 2 of the Constitution provides: Sec. 2. The Commission on Elections shall exercise the following powers and functions

(1) Enforce and administer all laws and regulations relative to the conduct of an election, plebiscite, initiative, referendum, and recall.

 (6) File, upon a verified complaint, or on its own initiative, petitions in court for inclusion or exclusion of votes, investigate and, where appropriate, prosecute cases of violation of election laws, including acts or omission constituting election frauds, offenses, and practices. (Emphasis supplied)

In effect the 1987 Constitution mandates the COMELEC not only to investigate but also to prosecute cases of violation of election laws. This means that the COMELEC is empowered to conduct preliminary investigations in cases involving election offenses for the purpose of helping the Judge determine probable cause and for filing an information in court. This power is exclusive with COMELEC.

The grant to the COMELEC of the power, among others, to enforce and administer all laws relative to the conduct of election and the concomittant authority to investigate and prosecute election offenses is not without compelling reason. The evident constitutional intendment in bestowing this power to the COMELEC is to insure the free, orderly and honest conduct of elections, failure of which would result in the frustration of the true will of the people and make a mere idle ceremony of the sacred right and duty of every qualified citizen to vote. To divest the COMELEC of the authority to investigate and prosecute offenses committed by public officials in relation to their office would thus seriously impair its effectiveness in achieving this clear constitutional mandate.

From a careful scrutiny of the constitutional provisions relied upon by the Sandiganbayan, We perceived neither explicit nor implicit grant to it and its prosecuting arm, the Tanodbayan, of the authority to investigate, prosecute and hear election offenses committed by public officers in relation to their office as contradistinguished from the clear and categorical bestowal of said authority and jurisdiction upon the COMELEC and the courts of first instance under Sections 182 and 184, respectively, of the Election Code of 1978.

An examination of the provisions of the Constitution and the Election Code of 1978 reveals the clear intention to place in the COMELEC exclusive jurisdiction to investigate and prosecute election offenses committed by any person, whether private individual or public officer or employee, and in the latter instance, irrespective of whether the offense is committed in relation to his official duties or not. In other words, it is the nature of the offense and not the personality of the offender that matters. As long as the offense is an election offense jurisdiction over the same rests exclusively with the COMELEC, in view of its all-embracing power over the conduct of elections. (Corpus v. Tanodbayan, 149 SCRA 281 [1987])

Hence, the Provincial Fiscal, as such, assumes no role in the prosecution of election offenses. If the Fiscal or Prosecutor files an information charging an election offense or prosecutes a violation of election law, it is because he has been deputized by the COMELEC. He does not do so under the sole 

28

Page 29: Search and Seizure Cases

authority of his office. (People v. Basilla, et al., G.R. Nos. 83938-40, November 6, 1989).i•t•c-aüsl In the instant case, there is no averment or allegation that the respondent Judge is bringing in the Provincial Fiscal as a deputy of COMELEC. He wants the Fiscal to "approve" the COMELEC's preliminary investigation.

It is to be noted that on February 27, 1987 (when the 1987 Constitution was already in effect) the President issued Executive Order No. 134 which was the ENABLING ACT FOR ELECTIONS FOR MEMBERS OF CONGRESS ON MAY 11, 1987 AND FOR OTHER PURPOSES." Section 11 thereof provides:

Prosecution. The Commission shall, through its duly authorized legal officers, have exclusive power to conduct preliminary investigation of all election offenses punishable as provided for in the preceding section, and to prosecute the same: Provided, That in the event that the Commission fails to act on any complaint within two (2) months from filing, the complainant may file the complaint with the Office of the Fiscal or with the Department of Justice for proper investigation and prosecution, if warranted.

The Commission may avail of the assistance of other prosecuting arms of the government.

It is only after a preliminary examination conducted by the COMELEC through its officials or its deputies that section 2, Article III of the 1987 Constitution comes in. This is so, because, when the application for a warrant of arrest is made and the information is filed with the court, the judge will then determine whether or not a probable cause exists for the issuance of a warrant of arrest.

Bearing these principles in mind, it is apparant that the respondent trial court misconstrued the constitutional provision when it quashed the information filed by the Provincial Election Supervisor. As indicated above what the respondent trial court should have done was to enforce its September 30, 1988 order, to wit:

Pursuant to Circular No. 12 of the Chief Justice of the Supreme Court dated June 30, 1987 and considering that after a personal examination of the evidence submitted by the investigating Provincial Election Supervisor III Negros Oriental (Designated Legal Officer), there is reasonable ground for this Court to rely on the certification of said Provincial Election Supervisor III in the information that a probable cause exists, let a warrant issue for the arrest of the accused filing the bail at FIVE THOUSAND (P5,000.00) PESOS as recommended by the Provincial Election Supervisor III.

The order to get the approval of the Provincial Fiscal is not only superfluous but unwarranted.

WHEREFORE, the instant petition is GRANTED. The questioned Orders dated October 3, 1988, November 22, 1988 and December 8, 1988 are REVERSED and SET ASIDE. The respondent trial court's Order dated September 30, 1988 is REINSTATED. The respondent court is ordered to proceed hearing the case with deliberate speed until its termination.

COLUMBIA PICTURES, INC., ORION PICTURES CORP., PARAMOUNT PICTURES CORP., TWENTIETH CENTURY FOX FILM CORP., UNITED ARTISTS CORP., UNIVERSAL CITY STUDIOS, INC., THE WALT DISNEY COMPANY, and WARNER BROS., INC., petitioners, vs.HON. JUDGE ALFREDO C. FLORES, FGT VIDEO NETWORK, INC., MANUEL MENDOZA, ALFREDO C. ONGYANCO, ERIC APOLONIO, SUSAN YANG and EDUARDO A. YOTOKO, respondents.

Before us is a petition for certiorari seeking to set aside the order dated May 29, 1987 of the Regional Trial Court of the National Capital Region (Branch 167, Pasig) directing the immediate release and return of television sets, video cassette recorders, rewinders, tape head cleaners, accessories, equipment, and other paraphernalia or pieces of machinery which had been seized by operatives of the National Bureau of Investigation by virtue of a search warrant.

Petitioners herein are all foreign corporations organized and existing under the laws of the United States of America and represented in the Philippines by their attorney-in-fact, Rebecca Benitez-Cruz of the Motion Picture Association of America, Inc. (MPAA for brevity). Private respondent FGT Video Network, Inc. is a merger of Fox, Galactic, and Technica Video. It is registered with and licensed by the Videogram Regulatory Board as a distributor under License No. 1333 VMM. Technica Video, Inc. which is part of the merger, is registered with and licensed as a reproducer by the said board under License No. 967 VMM (p. 11, Rollo).

In a letter dated April 20, 1987, the MPAA, through counsel Rico V. Domingo, lodged a complaint before then Director Antonio Carpio of the National Bureau of Investigation (NBI) against certain video establishments for violation of Presidential Decree No. 49 (Protection of Intellectual Property), as amended by Presidential Decree No. 1988, in connection with its anti-piracy campaign. Specifically complaining of the "unauthorized sale, rental, reproduction and/or disposition of copyrighted film", the MPAA sought the NBI's "urgent assistance in the conduct of search and seizure operations in Metro Manila and elsewhere". (p. 29, Rollo.)

On the basis of said letter, NBI and private agents conducted discreet surveillance operations on certain video establishments, among them private respondent FGT Video Network, Inc. (FGT). Thus, on April 20, 1987, Danilo Manalang, a.k.a. Ronaldo Lim, allegedly an NBI agent, went to the office of FGT to have the copyrighted motion pictures "Cleopatra" owned by Twentieth Century Fox Film Corp. and "The Ten Commandments" owned by Paramount Pictures, Inc. reproduced or retaped in video format. For the reproduction services, FGT issued Order Slip No. 3482 dated April 20, 1987 and Delivery Slip No. 118667 dated April 22, 1987, for which services Danilo Manalang paid P45.00. On May 5, 1987, Manalang also had MGM's copyrighted film "Walk Like a Man" reproduced or retaped by FGT for P15.00 (p. 5, Rollo).

Consequently, on May 14, 1987, NBI Agent III Lauro C. Reyes, with Manalang and Rebecca Benitez-Cruz as witnesses, applied for a search warrant with the Regional Trial Court in Pasig. Introduced as evidence in support of the application were the following: the letter dated April 20, 1987 of the MPAA through Rico V. Domingo (Exh. A) FGT's Order Slip No. 3842 (Exh. B); FGT's Delivery Slip No. 118667 (Exh. B-1); video cassettes containing the film "The Ten Commandments" (Exhs. B-1-A, B-1-B); video cassette containing the film "Cleopatra" (Exh. B-1-C); video cassette containing the film 

29

Page 30: Search and Seizure Cases

"Walk Like a Man" (Exh. B-1-D); FGT's Order Slip No. 3923 dated May 5, 1987 (Exh. B-2); FGT's Delivery Slip No. 123321 dated May 6, 1987 (Exh. B-3); list of copyrighted MPAA member company titles (Exh. C); sketch of location of FGT's office or premises (Exh. D); affidavit of Rebecca Benitez-Cruz (Exh. E); special power of attorney designating Ms. Benitez-Cruz as petitioners' attorney-in- fact (Exh. F to F-8); and affidavit of Danilo Manalang (Exh. G).

Upon the offer of these pieces of evidence, Judge Alfredo C. Flores of the aforesaid court, issued Search Warrant No. 45 which reads:

TO ANY PEACE OFFICER: GREETINGS: It appearing to the satisfaction of the Undersigned after examining under oath NBI Senior Agent Lauro C. Reyes and his witnesses Mr. Danilo Manalang and Ms. Rebecca Benitez-Cruz, that there is a probable cause to believe that Violation of Section 56 P.D. No. 49 as amended by P.D. No. 1988 (otherwise known as the Decree on Protection of Intellectual Property) has been committed and that there are good and sufficient reasons to believe that FGT Video Network, Inc., Manuel Mendoza, Alfredo C. Ongyanco, Eric Apolonio, Susan Yang and Eduardo Yotoko are responsible and have in control/possession at No. 4 Epifanio de los Santos corner Connecticut, Greenhills, San Juan, Metro Manila (per attached sketch and list of MPAA member Company Titles) the following properties to wit:

(a) Pirated video tapes of the copyrighted motion pictures/films the titles of which are mentioned in the attached list;

(b) Posters, advertising leaflets, flyers, brochures, invoices, lists of titles being reproduced or retaped, journals, ledgers, jon (sic) order slips, delivery slips and books of accounts bearing and/or mentioning the pirated films with titles (as per attached list), or otherwise used in the reproduction/repating business of the defendants;

(c) Television sets, video cassette recorders, rewinders, tape head cleaners, accessories, equipment and other machines and paraphernalia or materials used or intended to be used in the unlawful sale, lease, distribution, or possession for purpose of sale, lease, distribution, circulation or public exhibition of the above-mentioned pirated video tapes which they are keeping and concealing in the premises above-described, which should be seized and brought to the Undersigned.

You are hereby commanded to make an immediate search at any time in the day between 8:00 A.M. to 5:00 P.M. of the premises above-described and forthwith seize and take possession of the above-enumerated personal properties, and bring said properties to the undersigned immediately upon implementation to be dealt with as the law directs.

WITNESS MY HAND this 14th day of May 1987, at Pasig, Metro Manila. (pp. 30-31, Rollo; Emphasis supplied.)

At or about high noon of the same day, agents from the NBI, led by Lauro C. Reyes and Mamerto Espartero, with the assistance of the personnel of the Videogram Regulatory Board headed by Elmer San Pascual, duly served Search Warrant No. 45 on the operators or representatives of FGT. In the 

course of the search of the premises of FGT, the NBI agents found and seized various video tapes of duly copyrighted motion pictures or films owned and exclusively distributed by petitioners. Also seized were machines and equipment, television sets, paraphernalia, materials, accessories, rewinders, tape head cleaners, statements of order, return slips, video prints, flyers, production orders, and posters. Inventories of these seized articles were then prepared and copies thereof were furnished Jess Ayson, production manager of FGT. On May 18, 1987, the NBI agents filed a return of the search warrant with a motion to retain custody of the seized items (p. 32, Rollo).

Meanwhile, FGT filed an urgent motion for the immediate release of equipment and accessories "not covered" by the search warrant, without prejudice to the filing of a motion to quash the said search warrant (p. 101, Rollo). It argued that as a licensed video reproducer, FGT had the right to maintain possession of the seized reproduction equipment and paraphernalia which are not contraband or illegal per se, but are rather "exclusively used and intended to be used for reproduction" and not in the "sale, lease, distribution or possession for purposes of sale, lease distribution, circulation or public exhibition of pirated video tapes". (p. 102, Rollo.)

Petitioners opposed the motion, asserting that the seized articles were all lawfully taken. They explained that since FGT was a videogram distributor and not a reproducer, "it may be logically concluded that such 634 VCRs, accessories, etc." were "used or intended to be used in the unlawful sale, lease, distribution or possession for purposes of sale, lease, distribution, circulation or public exhibition of, at the very least, the 310 videocassette tapes containing the copyrighted films/motion pictures." They asserted that Search Warrant No. 45 was issued upon the proper determination of probable cause and that, therefore, it is not for FGT "to second-guess the wisdom" of the court's directive to seize the questioned VCRs and accessories "as an inquiry thereon would involve evidentiary matters which are better ventilated in the criminal prosecution proper". (pp. 107-116, Rollo.)

Finding that FGT was a "registered and duly licensed distributor and in certain instances and under special instructions and conditions . . . reproducer of videograms" and that, therefore, its right to possess and use the seized equipment had been "placed in serious doubt", the lower court resolved the doubt "against the Government and in favor of a lawful business enterprise." Applying the constitutional precept of presumption of innocence and considering that the seized articles are not contraband, respondent court ruled that to allow the Government "to keep possession of the equipment(s) and machines where there is no actual criminal charge" would amount to a "confiscation in violation of the due process clause of the constitution, notwithstanding the filing by the Director of the NBI of a letter to the Department of Justice recommending that the defendants be charged with violation of Section 56 of P.D. No. 49, as amended by P.D. No. 1988." (pp. 131-132, Rollo.)

Thus, in its order on May 29, 1987, the lower court granted FGT's motion and ordered the immediate release and return of the "television sets, video cassette recorders, rewinders, tape head cleaners, accessories, equipment and other machines or paraphernalias, as reflected in the "Receipt for Properties Seized" attached to the records of the case beginning from page 84 to page 130, to the defendants, excluding video cassette tapes reflected in the "Receipts for Properties Seized", beginning from page 132 to page 146 of the records." Respondent court also ordered the inventory 

30

Page 31: Search and Seizure Cases

of all articles returned with individual descriptions "to evidence their existence" copies of which inventory should be furnished the NBI and the court (p. 132, Rollo).

Hence, the present recourse. As prayed for by petitioners, on June 17, 1987, the Court issued a temporary restraining order enjoining respondents from implementing the lower court's order of May 29, 1987 upon a bond in the amount of P750,000.00 which petitioners accordingly posted on June 19, 1987, (pp. 138-141, Rollo.)

The sole issue to be resolved is whether or not the lower court acted with grave abuse of discretion amounting to lack of jurisdiction in ordering the immediate release and return of some of the items seized by virtue of the search warrant.

Petitioners insist that the search warrant was issued upon due determination of probable cause. They argue that FGT's act of illegally reproducing copyrighted films had been clearly established by evidence on record and that FGT's principal ground in praying for the immediate release of the seize articles is a matter of defense which should be ventilated at the trial of the case on the merits.

Private respondents, on the other hand, claim that the issuance of Search Warrant No. 45 is tainted with illegality as no particular or specific acts or omissions constituting the offense charged had been alleged in the application for its issuance.

The right to security against unreasonable searches and seizures is guaranteed under Section 2, Article III of the 1987 Constitution which provides:

Sec. 2. The right of the people to be secure in their persons, houses, papers and effects against unreasonable searches and seizures of whatever nature and for any purpose shall be inviolable, and no search warrant or warrant of arrest shall issue except upon probable cause to be determined by the judge after examination under oath or affirmation of the complainant and the witnesses he may produce, and particularly describing the place to be searched and the persons or things to be seized.

Thus, Sections 3 and 4 of Rule 126 of the Rules of Court provide for the requisites in the issuance of search warrants:

Sec. 3. Requisites for issuing search warrant. — A search warrant shall not issue but upon probable cause in connection with one specific offense to be determined personally by the judge after examination under oath or affirmation of the complainant and the witnesses he may produce, and particularly describing the place to be searched and the things to be seized.

Sec. 4. Examination of complainant; record. — The judge must, before issuing the warrant, personally examine in the form of searching questions and answers, in writing and under oath the complainant and the witnesses he may produce on facts personally known to them and attach to the record their sworn statements together with any affidavits submitted.

In issuing a search warrant, the judge must strictly comply with the constitutional and statutory requirements. He must determine the existence of probable cause by personally examining the applicant and his witnesses in the form of searching questions (Silva vs. Presiding Judge, RTC of Negros Oriental, Br. XXXIII (203 SCRA 140 (1991]). The search warrant must contain a specific description of the place to be searched and the articles sought to be seized must be described with particularity (Pendon vs. Court of Appeals, 191 SCRA 429 [1990]).

Withal, measured by the aforegoing constitutional and legal provisions as well as the existing jurisprudence on the matter, we find that Search Warrant No. 45 fails to satisfy the test of legality. More so because the Court has previously decided a case dealing with virtually the same search warrant.

In 20th Century Fox Film Corp. vs. Court of Appeals (164 SCRA 655 [1988]), wherein therein petitioner is also one of the petitioners herein, we upheld the legality of the order of the lower court lifting the search warrant issued under circumstances similar to those obtaining in the case at bar.

A striking similarity between the case at bar and 20th Century Fox is the fact that Search Warrant No. 45, specifically paragraph (c) thereof describing the articles to be seized, contains an almost identical description as the warrant issued in the 20th Century Fox case, to wit:

(c) Television sets, Video Cassettes Recorders, rewinders, tape head cleaners, accessories, equipments and other machines used or intended to be used in the unlawful reproduction, sale, rental/lease, distribution of the above-mentioned video tapes which she is keeping and concealing in the premises above-described. (at p. 664.)

On the propriety of the seizure of the articles above-described, we held in said case:

Television sets, video cassette recorders, rewinders and tape cleaners are articles which can be found in a video tape store engaged in the legitimate business of lending or renting out betamax tapes. In short, these articles and appliances are generally connected with, or related to a legitimate business not necessarily involving piracy of intellectual property or infringement of copyright laws. Hence, including these articles without specification and/or particularity that they were really instruments in violating an Anti-Piracy law makes the search warrant too general which could result in the confiscation of all items found in any video store. (at p. 665.)

The language used in paragraph (c) of Search Warrant No. 45 is thus too all-embracing as to include all the paraphernalia of FGT in the operation of its business. As the search warrant is in the nature of a general one, it is constitutionally objectionable (Corro vs. Lising, 137 SCRA 541 [1985]).

In consequence, respondent court was merely correcting its own erroneous conclusions in issuing Search Warrant No. 45 when it ordered the return of the seized television sets and other paraphernalia specified in the motion filed by FGT. This can be gleaned from its statement that ". . . the machines and equipment could have been used or intended to be used in the illegal 

31

Page 32: Search and Seizure Cases

reproduction of tapes of the copyrighted motion pictures/films, yet, it cannot be said with moral certainty that the machines or equipment(s) were used in violating the law by the mere fact that pirated video tapes of the copyrighted motion pictures/films were reproduced. As already stated, FGT Video Network, Inc. is a registered and duly licensed distributor and in certain instances and under special instructions . . . reproducer of videograms, and as such, it has the right to keep in its possession, maintain and operate reproduction equipment (s) and paraphernalia (s)." (pp. 131-132, Rollo.)

Far from being despotic or arbitrary, respondent judge must be commended for rectifying his error when he found that his initial conclusions were inaccurate and erroneous, colliding as they did with the constitutional rights of private respondent.

Much has been said in the media about piracy of films and videotapes and that violators of the law must be brought to the courts but, as the Court said in Bagalihog vs. Fernandez (198 SCRA 614 [1991]), "[z]eal in the pursuit of criminals cannot ennoble the use of arbitrary methods that the Constitution itself abhors." (at p. 622.)

WHEREFORE, the petition is DISMISSED, the assailed order of May 29, 1987 AFFIRMED, and the temporary restraining order issued on June 18, 1987, vacated and lifted.

PEOPLE OF THE PHILIPPINES, petitioner, vs. CESAR O. DELOS REYES, respondent.

Before the Court is a petition for review on certiorari of the Decision[1] of the Court of Appeals (CA)   in  CA-G.R.   SP  No.  51759  granting   the  petition  for   certiorari  of  Cesar  O.  delos  Reyes  and nullifying Search Warrant No. 98-905 issued on June 18, 1998 by Judge Manuela F. Lorenzo of the Regional Trial Court (RTC) of Manila, Branch 43.

The AntecedentsOn June 18, 1998, SPO3 Benjamin Nuguid of the Western Police District applied for a search 

warrant with the RTC of Manila, Branch 43, against Cesar Reyes alias “Cesar Itlog.” In support of his application, Nuguid submitted his affidavit and that of Alexis Tan, a housewife. Nuguid and Tan also testified in support of the application. After the court conducted examination of the said witnesses, it issued on even date Search Warrant No. 98-905 authorizing the search of the house allegedly under the possession and custody of one Cesar Reyes alias “Cesar Itlog,” at No. 2600 Oroquieta Street, Sta. Cruz, Manila, worded as follows:

SEARCH WARRANT

TO ANY PEACE OFFICER: GREETINGS: Upon sufficient showing of probable cause, after determination personally made by the undersigned on examination under oath of the applicant and his witness, by means of searching questions and answers, that respondent Cesar Reyes alias “Cesar Itlog” has in his possession, custody and control at the house and premises at 2600 Oroquieta St., Sta. Cruz, Manila, the following items:

a) undetermined amount of methamphetamine hydrochloride; andb) drug paraphernalia

in violation of Republic Act No. 6425 as amended;

You are hereby commanded to make an immediate search at anytime of the day or night of the house and premises above-mentioned and forthwith seize and take possession of the above-cited items and to bring said items to the undersigned to be dealt with as the law require.  Further, you are required to submit the return within ten (10) days from today.

GIVEN UNDER MY HAND AND SEAL this 18th day of June 1998 at the City of Manila.

MANUELA F. LORENZO

The policemen conducted a search not only of the house at No. 2600 Oroquieta Street, Sta Cruz, Manila, which turned out to be the house of respondent Cesar delos Reyes, but also of the car and motorcycle owned by the latter, bearing Plate Nos. UBS 463 and TA 8077, respectively. The car and the motorcycle happened to be parked near the house.

As per the receipt of the property signed by Nuguid, the search of the house, the car and the motorcycle yielded the following:

That in the course of orderly search at the premises of Cesar Reyes alias “Cesar Itlog,” inside his room at the ground floor was a steel vault and when forced open it yields 13 transparent plastic bags containing [an] undetermined amount of white crystalline substance suspected to be Methamphetamine Hydrochloride or Shabu, three (3) weighing scales “Tamita” broad, drugs paraphernalia and 38 pcs. of Valium-10, also found atop his drawer; a .9mm “Smith & Wesson” pistol, Model 39mm with SN-A643638 with magazines loaded with ammo, one (1) loaded magazine of 9mm and 36 rounds of .25 cal. ammunition inside his drawer, one (1) plastic transparent bag containing white crystalline substance suspected to be Methamphetamine Hydrochloride or Shabu and three (3) 12-gauge shotgun ammo.  His personal car, a black VITARA bearing plate No. UBS 463 parked beside his house was also search[ed] in the presence of [a] Bgy. Kagawad and found inside tucked beneath the driver’s seat are three (3) sealed transparent plastic bags containing white crystalline substance wrapped in a mail envelope suspected to be Methamphetamine Hydrochloride or Shabu and in his sport Honda Motorcycle 900cc with plate No. TA 8077 also yields one (1) transparent plastic sachet containing white crystalline substance suspected to be Methamphetamine Hydrochloride or Shabu at the motorbike back compartment.[3]

According to the Certification prepared by the NBI Forensic Chemistry Division, the crystalline substances contained in the transparent plastic bags which were seized in the respondent’s house, car and motorcycle tested positive for methamphetamine hydrochloride.[4]

Thereafter,   two   Informations  were  filed  with   the  RTC  of  Manila,  Branch  41,   against   the respondent for violation of Republic Act No. 6425, as amended by Rep. Act No. 8294, docketed as Criminal Cases Nos. 98-165628 and 98-165629, viz:

32

Page 33: Search and Seizure Cases

That on or about June 18, 1998, in the City of Manila, Philippines, the said accused without being authorized by law to possess or use any regulated drug, did then and there willfully, unlawfully and knowingly have in his possession and under his custody and control eighteen (18) transparent plastic bags (small and big) with [a] total net weight of eight hundred eighty-six point eight (886.8) grams of white crystalline substance known as “shabu” containing methamphetamine hydrochloride, a regulate drug, without the corresponding license or prescription thereof. Contrary to law.[5]

That on or about June 18, 1998, in the City of Manila, Philippines, said accused did then and there willfully, unlawfully and feloniously have in his possession and under his custody and control of one (1) .9mm Smith & Wesson pistol, Model 39 with Serial Number-A643638 with two magazines loaded with ammunitions, 36 rounds of .25 caliber ammunition, three (3) 12-gauge shotgun ammunitions, without first having secured from the proper authorities the necessary license therefor. Contrary to law.[6]

The respondent filed a motion to quash the informations on the following grounds: (a) as shown by  their   testimony before  the  trial  court,  applicant  Nuguid  and his  witness  Tan had no personal   knowledge   of   the   factual   allegations   in   their   affidavits   which   were   appended   to   the application for a search warrant; (b) the factual allegations contained in the said affidavits and their testimonies  do not  support  a  finding  of  probable  cause   for  violation of  Rep.  Act  No.  6425,  as amended; and (c) Nuguid and Tan did not personally know the respondent as well as the latter’s house because first, Tan identified the illicit drug seller as Cesar Reyes alias “Cesar Itlog” while the respondent’s name is Cesar delos Reyes; and, second, the said witnesses described the house as consisting  of  a   two-storey   structure  with  one  bedroom  located  at  Oroquieta  Street,  Sta.  Cruz, Manila.

On August 11, 1998, the trial court issued an Order denying the respondent’s motion. The court also denied his motion for reconsideration of said order.

The   respondent   forthwith   filed   a   petition   for   certiorari   against   Nuguid   and   the   Public Prosecutor   therein   in   the  CA,   alleging, inter alia, that   the  questions  propounded  by  RTC   Judge Manuela F. Lorenzo on Nuguid and Tan were leading and not searching. He also alleged that Judge Lorenzo delegated the examination of Tan to Nuguid, and allowed the latter to question her.  He, likewise, reiterated that Tan and Nuguid did not know him personally because they identified him as “Cesar Reyes” when his full name was Cesar delos Reyes. Furthermore, contrary to the claim of Tan and Nuguid that his house was a two-storey edifice located at Oroquieta Street, Sta. Cruz, Manila, said house was only a one-storey structure located at No. 2600 Oroquieta Street, Sta. Cruz, Manila.

The respondent also assailed the search of his house, car and motorcycle on the ground that he was not there when the search was conducted and that no barangay officials were present as required by Section 7, Rule 126 of the 1997 Rules of Criminal Procedure.

On October 15,  1999,  the CA rendered a Decision granting the petition and nullifying the search warrant.  The decretal portion reads:

WHEREFORE, all the foregoing considered, the petition is GRANTED.  The questioned Order dated January 11, 1999 as well as Search Warrant No. 98-905 dated June 18, 1998 are both hereby ANNULLED and SET ASIDE.  Consequently, let a Writ of Prohibition be issued permanently enjoining respondents from using in evidence the articles seized by virtue of Search Warrant No. 98-905 in Criminal Case Nos. 98-165628 and 98-165629.  The seized articles obtained by virtue of Search Warrant 98-905 consisting of regulated drug, guns and ammunitions, are hereby ORDERED delivered and turned over to the proper authorities concerned for disposition in accordance with law.

The appellate court ruled that (a) the RTC delegated its duty to determine probable cause to the applicant; (b) the application for a search warrant was based on hearsay evidence; and (c) the application for the search warrant issued was filed more than four (4) weeks from the alleged time the offense took place; hence, was considered “stale.”

After   the denial  of   its  motion for  reconsideration of   the said  decision,   the  People  of   the Philippines filed the instant petition for review of the decision, alleging that –

THE COURT OF APPEALS [EIGHTH DIVISION] GRAVELY ERRED IN DECLARING SEARCH WARRANT NO. 98-905 NULL AND VOID DESPITE (i) THE UNMISTAKABLE MANNER BY WHICH THE INVESTIGATING JUDGE CONDUCTED A PERSONAL EXAMINATION OF THE APPLICANT AND HIS WITNESS; (ii) THE SEARCHING QUESTIONS PROPOUNDED AND ANSWERS OBTAINED; AND (iii) THE PERSONAL KNOWLEDGE OF THE APPLICANT AND HIS WITNESS OF THE FACTS THAT INEVITABLY JUSTIFIES THE ISSUANCE OF THE SEARCH WARRANT.[8]

The petitioner avers that Judge Lorenzo did not delegate the determination of probable cause to Nuguid before issuing the subject warrant. While she allowed Nuguid to propound questions on Alexis   Tan,   the   same   consisted   of   only   three   preliminary   questions,   and,   as   such,   was inconsequential.   The   petitioner   also   asserts   that   the   leading   questions   propounded   by   Judge Lorenzo on Tan does not detract from the fact that searching questions were also propounded on the   witnesses,   and   that  based   on   the   entirety  of   such   propounded   questions   and   the   latter’s answers, there was probable cause for the issuance of a search warrant. The petitioner maintains that Tan had personal knowledge of the respondent’s delictual acts which were in violation of Rep. Act No. 6425, as amended.  Moreover, as gleaned from the affidavits of Tan and Nuguid and their collective   testimonies   before   the   RTC,   the   respondent’s   house   was   sufficiently   described   and identified, which description Nuguid was able to confirm through his surveillance of the house, the place where the crime was committed.

The petitioner  further contends that  although there was an interregnum of  six  (6)  months from the time the commission of the crime came to the knowledge of Tan up to the filing of the application of the search warrant by Nuguid, the same did not obscure the finding of probable cause made by Judge Lorenzo.

The Court gave due course to the petition and required the parties to submit their respective memoranda.[9]

33

Page 34: Search and Seizure Cases

After a comprehensive and well-studied review of the Rollo and the records of the Court of Appeals, we resolve to deny the petition.

The Petition WasFiled Out of Time

The Office of the Solicitor General (OSG) admitted in the petition at bar that it received a copy of the assailed decision of the CA on October 21, 1999. Under Section 2, Rule 45 of the Rules of  Court, the OSG had until November 5, 1999 within which to file its petition for review on certiorari.  However, it did so only on November 25, 1999, long after the period therefor had lapsed.  We reject as   totally   unacceptable   the   pretext   of   Solicitor   Ma.   Theresa   Dolores   C.   Gomez-Estoesta   that, because of  heavy pressure of  work,[10] the actual  filing of   the motion to file the petition at bar prepared on November 3, 1999, was “accidentally slighted.” The Solicitor is mandated to insure that her motion for extension was filed within the period therefor.[11] Volume of work is a lame excuse.[12] She cannot escape the adverse effects of her forgetfulness.

Even if we gloss over the gross negligence of the OSG and resolve the petition on its merits, we find the same to be barren of merit.

A search warrant must (a) be based on probable cause; (b) contain a particular description of the place to be searched; and (c) must describe the items or property to be seized.[13] Probable cause comprehends such facts and circumstances as will induce a cautious man to rely upon and act in pursuance thereof.[14]

It   bears   stressing   that   the   requirement   of   particularity   is   related   to   the   probable   cause requirement in that, at least, under severe circumstances, the lack of a more specific description will make it apparent that there has not been a sufficient showing to the Judge that the described items are to be found in a particular place.  Probable cause must first focus on a specific location.  If the applicant or official is unable to state with sufficient precision the place to be searched and why he reasonably believes that contraband or evidence of criminal activity will be found therein, it is highly doubtful that he possesses probable cause for a warrant.[15]

In   issuing a search warrant,   the Judge must  strictly  comply with the requirements  of   the Constitution and the statutory provisions.[16]

A search warrant shall not issue except upon probable cause to be determined personally by the Judge after examination under oath or affirmation of the complainant and the witnesses he may produce.[17] Before  issuing a search warrant,  the Judge must personally  examine,  in the form of searching questions and answers, in writing and under oath, the complainant and his witnesses he may produce, on facts personally known to them.[18]

The mandate of   the  Judge  is   for  him to conduct  a  full  and searching examination of   the complainant and the witnesses he may produce.   In the absence of  a rule to the contrary,   the determination of probable cause cannot be delegated by the Judge, in part, or in whole, regardless of the qualifications of the person on whom reliance is placed.  It is not permissible for the Judge to share the required determination with another.[19]

The searching questions propounded to the applicant and the witnesses must depend on a large extent upon the discretion of the Judge. Although there is no hard-and-fast rule as to how a 

Judge may conduct his examination, it is axiomatic that the said examination must be probing and exhaustive and not merely routinary, general, peripheral or perfunctory.[20] He must make his own inquiry on the intent and factual and legal justifications for a search warrant. The questions should not merely be repetitious of the averments not stated in the affidavits/deposition of the applicant and the witnesses.[21] If the Judge fails to determine probable cause by personally examining the applicant and his witnesses in the form of searching questions before issuing a search warrant, it constitutes grave abuse of discretion.[22]

A search warrant  proceeding  is   independent of  any criminal  case.   It   is ex parte and non-adversarial.[23] Hence, the Judge acting on an application for a search warrant is not bound to apply strictly the rules of evidence.  As ruled in Brinegar v. United States:[24]

The inappropriateness of applying the rules of evidence as a criterion to determine probable cause is apparent in the case of an application for a warrant before a magistrate, the context in which the issue of probable cause most frequently arises.  The ordinary rules of evidence are generally not applied in ex parte proceedings, partly because there is no opponent to invoke them, partly because the Judge’s determination is usually discretionary, partly because it is seldom that, but mainly because the system of evidence rules was devised for the special control of trials by jury.

The Judge is not proscribed, at all times, from propounding leading questions on the applicant and the witnesses he may produce.  Indeed, the Judge is allowed to propound leading questions if, for   instance,   the  witness   is   a   child  or   is   suffering   from mental   illness,  or   if   the  questions  are preliminary or clarificatory, or when there is difficulty in getting direct and intelligent answers from the witness who is ignorant.

But it can hardly be justifiably claimed that, by propounding leading questions only on the complainant   and   the   witnesses   he   may   produce,   the   Judge   thereby   conducts   probing   and exhaustive examination.   After all,  a   leading question  is  one which suggests   to  the witness   the answer which the examining party desires.[25] By propounding leading questions, the Judge thereby puts the words or answers in the mind of the witness to be echoed back.[26]

It bears stressing that the determination of the existence of probable cause must be made by a detached and neutral Judge.[27] If he resorts to propounding leading questions to the applicant and his witnesses to determine probable cause, the Judge may be perceived as being partial, or even in cahoots with the officers engaged in the often competitive enterprise of ferreting out crime.[28]

A search warrant is not thereby rendered invalid; nor is a finding of probable cause proscribed merely because the Judge propounded leading questions on the applicant and the witnesses he produces.  The entirety of the questions propounded by the court and the answers thereto must be considered and calibrated by the Judge.

The Judge Allowed theApplicant Nuguid to ExamineTan, His Witness, and Failedto Propound Searching Questions

34

Page 35: Search and Seizure Cases

The transcript of the stenographic notes taken when Nuguid and Tan testified is quoted, in toto, infra:

COURT…

COURT:Q    You are applying for a search warrant.A     Yes, Your Honor.Q    Where is this place to be searched?A     At no. 2006 Oroquieta St., Sta. Cruz, Manila, Your Honor.Q    Is there any person there whom you would want to search?A     Yes, Your Honor.Q    Who?A     In the name of Cesar Reyes, Your Honor, alias “Cesar Itlog.”Q    Why, what is it he is keeping in his custody?A     Undetermined quantities of suspected methamphetamine hydrochloride also known as “Shabu.”Q    How do you know that such things exist in his place?A     Thru my witness, Your Honor, we were able to test-buy and examine the contents in a plastic sachet.Q    Why, what did your witness do, if any?A     I asked my witness to buy from Cesar Reyes alias “Cesar Itlog” and she was able to buy the subject shabu.Q    How did you know that your witness was able to buy from Cesar Reyes and not from other source?A     She told me and according to her she got it from Cesar Reyes.Q    Who is this witness you are referring to?A     Alexis Tan, Your Honor.Q    Where is she?A     She is the one, Your Honor.  (Witness pointing to a lady who answered when asked of her name as Alexis Tan).COURT:Alright, I will ask her.(Swearing Ms. Tan) –Q    Do you swear to tell the truth and nothing but the whole truth?A     Yes, I do.Q    Please state your name, age and other personal circumstances.A     ALEXIS TAN, 34 years old, married, jobless and with address c/o WBD Drug Enforcement Section, U.N. Ave., Manila.Q    What is your occupation?A     None for the moment, Your Honor.Q    What was your occupation, if any, before?A     A plain housewife, Your Honor.COURT:(to SPO3 Nuguid) –You want to ask her questions on record?

SPO3 NUGUID:Your Honor, she has her statement …COURT:Yes, but for the record, you may ask her.SPO3 NUGUID:Yes, Your Honor.Q    Do you personally know one Cesar Reyes alias “Cesar Itlog?”A     Yes, Sir.Q    How long have you known Cesar Reyes?A     Maybe around 6 months ago.Q    How did you come to know him?A     I was introduced to him by a friend, Sir.COURT:Q    What was the purpose of introducing you to him by your friend?A     It started when I was separated from my husband when my friend taught me how to use shabu, Your Honor.Q    When you were separated from your husband, what has it something to do with introducing you to Cesar Reyes?A     Thru influence, Your Honor.Q    What connection does it have?A     They know each other, Your Honor.  My friend knows that Cesar Reyes is selling shabu, Your Honor.Q    Did you really find out if Cesar Reyes alias “Cesar Itlog” is selling shabu?A     Yes, Your Honor.Q    How?A     At first, I was accompanied by my friend, but later I went there on my own alone.Q    You mean, this Cesar Reyes is really in the business if (sic) selling shabu?A     Yes, Your Honor.Q    Knowing his prohibited ((sic) activity, does he also sell to any other people?A     Those known to him, Ma’am.Q    You consider yourself as very well known to him?A     Yes, Your Honor.Q    Have you also seen him in [the] company of that friend of yours who introduced you to him?A     Yes, Ma’am for many times.Q    And you have been going to this place of Cesar Reyes several times also.A     Yes, Your Honor, I bought shabu from him.Q    How did he sell it to you?A     I will call him first through the phone before I go to him.Q    You make an appointment with him first?A     Yes, Your Honor.Q    Where is this place?A     At Oroquieta Street, Sta. Cruz, Manila, Your Honor.Q    Everytime you go and buy shabu from him, is it always ready for sale to you?A     Yes, Your Honor.Q    You mean he has always in his custody and does not run out of stock?

35

Page 36: Search and Seizure Cases

A     There was one time when I called him if he could sell one for me he told me through the phone to call back after an hour because he will be getting it from other source.Q    Did you call him back after an hour?A     Yes, Your Honor, and he told me to come over to his place.Q    How much quantity (sic) did you buy from him?A     For P3,000.00, Your Honor.Q    I am referring to the quantity.A     3 grams, Your Honor, he does not sell lower than 3 grams, Your Honor, it must be 3 grams and above.Q    You have not yet bought from him only one gram?A     No, Your Honor, not less than 3 grams.Q    During the time you bought shabu from Cesar Reyes, were you the only customer?A     He entertains customer (sic) one at a time, Your Honor, but he has several customers.Q    How do you know that he has several customers?A     Because he also talked [to] callers on the phone.  During the time I bought shabu from him, he also talked to somebody on the phone.Q    That is only. . .Q    Since when did you start buying shabu from him?A     Between December 1997 and January 1998, Your Honor.Q    The shabu you had been buying from him, do you use it or sell it to some other person?A     No, Your Honor, I do not sell it.Q    You use it?A     Yes, Your Honor.Q    You know the house of Cesar Reyes after confirmation, in what particular part of the house does Cesar Reyes entertain you?A     In [the] living room, Your Honor.Q    When you go to his house, you usually go and see him in that (sic) living room and not elsewhere before he comes out from that house?A     Normally, Your Honor, when I go to his house, he would open the door for me and would say “come in” then ask me “how much.”  If, for example, I would say 3 grams, he would just go to his room and comes out with the item.Q    In other words, everytime you go to his place to buy shabu she (sic) is there ready to entertain you?A     Yes, Your Honor.Q    Is it not that he is still busy conversing with other people when he comes out from his room?A     When I go to his house, he is there ready to open for me (sic) he knows I am coming.Q    That is always the case, he is ready to open the door for you?A     There was also a time that his maid opened the door for me.Q    Aside from the maid, did you see other people inside that house?A     His family- his wife and a baby then he would usually let them stay away from the living room or just get inside the room.Q    What kind of a house does Cesar Reyes have?A     A two-story (sic) house, Your Honor.Q    Not an apartment?A     No, Your Honor.

Q    A single detach  (sic) house?A     Yes, Your Honor.Q    Is there any guard on (sic) the main gate of the house?A     None, Ma’am, it is just an ordinary house.Q    There are no people you usually see when you go there?A     There are some members of the family but usually he let (sic) them stay away from the living room.Q    Was there an occasion when somebody arrives when you see him?A     None, Your Honor.Q    So, this is a one-on-one affair.A     Yes, Your Honor.Q    How do you know that these things are stored in his house?A     Everytime I bought shabu from him, he would get the money from me and then get inside his room to get a sachet of shabu and give it to me.Q    Are you sure that these things (shabu) are stocked in his house permanently or maybe they are just brought there from somewhere because he knows you are coming to buy and get it?A     There was once when I called him by phone and he asked me "how much” and I told him I will buy for (sic) P4,000.00 worth of shabu and he said “you just proceed to my place by 2 o’clock in the afternoon,” I will have to get it from other source.”Q    He got it from other source for you?A     According to him, if it is by large (sic) quantity and he will just repack it in his house.Q    He himself told you?A     Yes, Your Honor, but he does not reveal from whom.Q    You have no idea?A     No, Your Honor.Q    You did not ask him?A     No, Your Honor.Q    You used to transact business on the ground floor of his house?A     Yes, Your Honor.Q    Did you notice if his house has several rooms?A     There is one room on the ground floor, Ma’am, but sometimes he also goes upstairs and comes down with the shabu item.  Most of the time of the transaction just on the ground floor.Q    Is there a partition in that particular room?A     It is just a single room, Ma’am.Q    Did you not notice if there are other people in that room in the ground floor?A     I did not notice but there was one time when I saw a child but he let that child stay away from the visitor.Q    You mean this room where you saw him come out serve as storeroom of shabu?A     Yes, Ma’am.Q    Did he tell you about it?A     Yes, Ma’am.Q    Why did he tell you?A     Because when he entertained me, he left me for the moment and I said “where do you go” and he said “I will get inside that room to get the shabu.”

36

Page 37: Search and Seizure Cases

Q    So, since you were requested by the police officer to purchase shabu from Cesar Reyes, how many times?A     That was the only time, Your Honor.COURT: (to SPO3 NUGUID)Q    Did you make surveillance in that place?A     Yes, Your Honor, we made a surveillance after the test-buy.Q    What did you do?A     During the surveillance, we brought several witnesses.Q    Did you notice people going there to the house of Cesar Reyes?A     Yes, Your Honor.Q    And what have you observed?A     Some customers are even using cars.Q    You do not know if those people were visitors or not?A     We are not sure if those people are visitors of Cesar Reyes because we have no contact inside his house.COURT: (to Ms. TAN) -Q    When did you buy shabu from Cesar Reyes?A     June 13, 1998, Your Honor.Q    This Cesar Reyes at the time did not have any idea that you were there being sent by the police officers?A     No, Your Honor.Q    Did you really go to his place and successfully bought the shabu from Cesar Reyes?A     Yes, Your Honor.[29]

The questions propounded on Nuguid by Judge Lorenzo were not searching and probing, but merely superficial and perfunctory.  The records show that in his application for a search warrant, Nuguid described the place to be searched as the house located at “No. 2600 Oroquieta Street, Sta. Cruz, Manila,” under the name of Cesar Reyes alias “Cesar Itlog.”  However, the Judge ignored this inconsistency and did not bother to inquire from Nuguid why he applied for a search warrant of the premises   at   No.   2600   Oroquieta   Street,   Sta.   Cruz,   Manila,   when   the   house   where   Tan   had apparently purchased shabu from the respondent was located at No. 2006 Oroquieta Street, Sta. Cruz, Manila.

Nuguid declared that he and the police officers conducted a test-buy on June 13, 1998, using Tan as the buyer for said purpose.  The ordinary procedure for a test-buy is for the police officers to monitor and observe, at a distance, the sale of illicit drugs by the suspect to the buyer.  In this case, when   the   Judge  asked  Nuguid  how  Tan  was  able   to  buy   shabu   from  the   respondent,  Nuguid reported that Tan told him that he got shabu from Cesar Reyes, implying that he did not witness the test-buy;   however,   Nuguid   also   declared   that   he   was   at   a   distance   when   the   test-buy   was conducted.

The Judge also failed to ask Nuguid the circumstances upon which he and the other police officers came to know how Tan was able to purchase shabu from the respondent.   Inexplicably, Nuguid conducted a search of the house of the respondent only after the test-buy and not before then.  However, the Judge failed to inquire why the application for a search warrant was made only on June 18, 1998, or after the lapse of five days from the time the test-buy was conducted on June 13, 1998.  The Judge also failed to ask Nuguid why no surveillance was made before the test-buy 

and whether any report on the surveillance operations conducted on the respondent’s house after the test-buy was submitted.

Even a cursory reading of the transcript will show that most of the questions propounded on Tan by the Judge were leading questions, and    that those which were not leading were merely based on or related to the answers earlier given to the leading questions.  By asking such leading questions, the Judge thereby supplied the answers to her questions.  Although Tan testified that she used   to  buy  at   least   three   (3)  grams   for P3,000.00   from  the   respondent  during   the  period  of December  1997   to   January  1998,   the   Judge  did  not  even  bother   to   inquire   from Tan,  a  plain housewife   who   was   separated   from   her   husband,   how   she   could   afford   to   purchase   shabu for P3,000.00 on several occasions during the period of December 1997 to January 1998.

Indeed, there was an interregnum of more than four (4) months from the time Tan purchased shabu from the respondent up to the time when the test-buy was supposedly made.  However, the Judge was not even curious as to why Tan failed to purchase shabu from the respondent for such a long period of time, considering that from her testimony, Tan made it plain that she was a regular user of shabu. The Judge should have asked Tan why she did not buy shabu from the respondent for more than four months.

The Judge even failed to inquire from Tan when and under what circumstances Nuguid was able to meet with her to discuss how she would be utilized for the test-buy. The curiosity of the Judge was not even aroused when, in answer to her question on the location of the house of “Cesar  Reyes,” Tan replied that it was located at Oroquieta Street, Sta. Cruz, Manila, without specifying the house number.  At the very least, it behooved the Judge to require Tan to specify the house number if only to test her credibility.   And yet, immediately after propounding the questions on Tan and Nuguid, the Judge announced that she was issuing the search warrant.

A     June 13, 1998, Your Honor.

Q    This Cesar Reyes at the time did not have any idea that you were there being sent by the police officers?A     No, Your Honor.Q    Did you really go to his place and successfully bought the shabu from Cesar Reyes?A     Yes, Your Honor.COURT(to SPO3 NUGUID) –Q    During the time that Alexis Tan was being sent there to buy shabu from Cesar Reyes, where were you then?A     We were at a distance, Your Honor.COURT:Do you have something to add questions from her?SPO3 NUGUID:No more at the moment, Your Honor.COURT:

That will be all for now and the Court will issue the Search Warrant.[30]

37

Page 38: Search and Seizure Cases

The   Judge   allowed   and   even   egged   on   Nuguid   to   examine   Tan   and   elicit   facts   and circumstances from her relating to the alleged purchase of shabu from the respondent.  What is so worrisome is that Nuguid, besides being the applicant, was the same police officer who asked Tan to buy   shabu   from   the   respondent   and   the   one   who,   along   with   other   officers,   arrested   the respondent. That Nuguid propounded comparatively fewer questions on Tan is beside the point.  By allowing Nuguid himself to examine Tan, the Judge thereby compromised her impartiality.

We echo, once again, the oft-cited caveat of the Court:

It has been said that of all the rights of a citizen, few are of greater importance or more essential to his peace and happiness than the right of personal security, and that involves the exemption of his private affairs, books, and papers from inspection and scrutiny of others.  While the power to search and seize is necessary to the public welfare, still it must be exercised and the law enforced without transgressing the constitutional rights of the citizens, for the enforcement of no statute is of sufficient importance to justify indifference to the basic principles of government.

Thus, in issuing a search warrant, the Judge must strictly comply with the requirements of the Constitution and the statutory provisions.  A liberal construction should be given in favor of the individual to prevent stealthy encroachment upon, or gradual depreciation of the rights secured by the Constitution.  No presumption of regularity is to be invoked in aid of the process when an officer undertakes to justify it.[31]

IN LIGHT OF ALL THE FOREGOING, the petition is DENIED.

PEOPLE OF THE PHILIPPINES, plaintiff-appellee, vs. MODESTO TEE a.k.a. ESTOY TEE, accused-appellant.

For  automatic   review   is   the   consolidated   judgment[1] of   the  Regional   Trial  Court   (RTC)  of Baguio City, Branch 6, dated September 17, 1999,  in Criminal Cases Nos. 15800-R and 15822-R, involving  violations of  Section 8,  Article   II,  of   the Dangerous  Drugs  Law.[2]   Since appellant  was acquitted in the second case, we focus on the first case, where appellant has been found guilty and sentenced to death and fined one million pesos.

The decretal portion of the trial court’s decision reads:

WHEREFORE, judgment is hereby rendered, as follows:

1. In Crim. Case No. 15800-R, the Court finds the accused Modesto Tee guilty beyond reasonable doubt of the offense of illegal possession of marijuana of about 591.81 kilos in violation of Section 8, Article II of RA 6425 as amended by Section 13 of RA 7659 as charged in the Information, seized by virtue of a search warrant and sentences him to the supreme penalty of death and to pay a fine of 1 million pesos without subsidiary imprisonment in case of insolvency.

The 591.81 kilos of marijuana contained in 26 boxes and one yellow sack (Exhibits U-1 to U-27) are ordered forfeited in favor of the State to be destroyed immediately in accordance with law.

2. In Crim. Case No. 15822-R, the Court finds that the prosecution failed to prove the guilt of accused Modesto Tee beyond reasonable doubt and hereby acquits him of the charge of illegal possession of marijuana in violation of Section 8, Art. 2 of RA 6425 as amended by Section 13 of RA 7659 as charged in the Information since the marijuana confiscated have to be excluded in evidence as a product of unreasonable search and seizure.

The 336.93 kilos of marijuana contained in 13 sacks and four boxes (Exh. B to S and their component parts) although excluded in evidence as the product(s) of unreasonable search and seizure, are nevertheless ordered forfeited in favor of the State to be destroyed immediately in accordance with law considering that they are prohibited articles.

The City Jail Warden is, therefore, directed to release the accused Modesto Tee in connection with Crim. Case No. 15822-R unless held on other charges.

Appellant is a Chinese national in his forties, a businessman, and a resident of Baguio City. A raid conducted by operatives of the National Bureau of Investigation (NBI) and Philippine National Police Narcotics Command (PNP NARCOM) at premises allegedly  leased by appellant and at his residence yielded huge quantities of marijuana.

On July 20, 1998, appellant moved to quash the search warrant on the ground that it was too general and that the NBI had not complied with the requirements for the issuance of a valid search warrant. The pendency of said motion, however, did not stop the filing of the appropriate charges against appellant. In an information dated July 24, 1998, docketed as Criminal Case No. 15800-R, the City Prosecutor of Baguio City charged Modesto Tee, alias “Estoy Tee,” with illegal possession of marijuana, allegedly committed as follows:

That on or about the 1st day of July, 1998 in the City of Baguio, Philippines, and within the jurisdiction of this Honorable Court, the above-named accused, did then and there willfully, unlawfully, feloniously and knowingly have in his possession the following, to wit:

1. Ninety-two (92) bricks of dried flowering tops separately contained in four (4) boxes; and

2. One hundred fifty-eight (158) bricks, twenty-one (21) blocks, and twenty-three (23) bags of dried flowering tops separately contained in thirteen (13) sacks, with a total weight of 336.93 kilograms; and

3 Six hundred two (602) bricks of dried flowering tops separately contained in twenty-six (boxes) and a yellow sack, weighing 591.81 kilograms,

all having a grand total weight of 928.74 kilograms, a prohibited drug, without the authority of law to possess, in violation of the above-cited provision of law.

CONTRARY TO LAW.[4]

38

Page 39: Search and Seizure Cases

On   August   7,   1998,   the   prosecution   moved   to   “amend”   the   foregoing   charge   sheet “considering that subject marijuana were seized in two (2) different places.”[5]

As a result, the information in Criminal Case No. 15800-R was amended to read as follows:

That on or about the 1st day of July, 1998, in the City of Baguio, Philippines, and within the jurisdiction of this Honorable Court, the above-named accused, did then and there willfully, unlawfully, feloniously and knowingly have in his possession the following, to wit:

- Six hundred two (602) bricks of dried flowering tops separately contained in twenty-six (26) boxes and a yellow sack, weighing 591.81 kilograms

a prohibited drug, without the authority of law to possess, in violation of the above-cited provision of law.

CONTRARY TO LAW.[6]

A separate amended information docketed as Criminal Case No. 15822-R was likewise filed, the accusatory portion of which reads:

That on or about the 1st day of July, 1998 in the City of Baguio, Philippines, and within the jurisdiction of this Honorable Court, the above-named accused, did then and there willfully, unlawfully, feloniously and knowingly have in his possession the following, to wit:

1. Ninety-two (92) bricks of dried flowering tops separately contained in four (4) boxes; and

2. hundred fifty-eight (158) bricks, twenty-one (21) blocks, and twenty-three (23) bags of dried flowering   tops  separately  contained   in   thirteen   (13)   sacks,  with  a   total  weight  of  336.93 kilograms;

a prohibited drug, without the authority of law to possess, in violation of the above-cited provision of law.

CONTRARY TO LAW.[7]

On September 4, 1998, the trial court denied the motion to quash the search warrant and ordered appellant’s arraignment.

When arraigned in Criminal Cases Nos. 15800-R and 15822-R, appellant refused to enter a plea. The trial court entered a plea of not guilty for him.[8] Trial on the merits then ensued.

The facts of this case, as gleaned from the records, are as follows:

Prosecution witness Danilo Abratique, a Baguio-based taxi driver, and the appellant Modesto Tee are well acquainted with each other, since Abratique’s wife is the sister of Tee’s sister-in-law.[9]

Sometime in late June 1998, appellant asked Abratique to find him a place for the storage of smuggled cigarettes.[10] Abratique brought  appellant  to  his   friend,  Albert  Ballesteros,  who had a house for rent  in Bakakeng, Baguio City. After negotiating the terms and conditions, Ballesteros agreed to rent out his place to appellant. Appellant then brought several boxes of purported “blue seal” cigarettes to the leased premises.

Shortly thereafter, however, Ballesteros learned that the boxes stored in his place were not “blue seal” cigarettes but marijuana. Fearful of being involved, Ballesteros informed Abratique. Both later prevailed upon appellant to remove them from the premises.[11]

Appellant   then   hired   Abratique’s   taxi   and   transported   the   boxes   of   cannabis   from   the Ballesteros place to appellant’s residence at Km. 6, Dontogan, Green Valley, Sto. Tomas, Baguio City.[12]

On June 30,  1998, appellant hired Abratique to drive him to La Trinidad,  Benguet on the pretext  of  buying and transporting strawberries.  Upon reaching La Trinidad,  however,  appellant directed Abratique to proceed to Sablan, Benguet, where appellant proceeded to load several sacks of marijuana in Abratique’s taxi. He then asked Abratique to find him a place where he could store the contraband.[13]

Abratique   brought   appellant   to   his   grandmother’s   house   at   No.   27   Dr.   Cariño   St.,   QM Subdivision, Baguio City, which was being managed by Abratique’s aunt, Nazarea Abreau. Nazarea agreed to rent a room to appellant.  Abratique and appellant unloaded and stored there the sacks of marijuana brought from Sablan.[14] Abratique was aware that they were transporting marijuana as some of the articles in the sacks became exposed in the process of loading.[15]

Eventually, Abratique and Nazarea were bothered by the nature of the goods stored in the rented room. She confided to her daughter, Alice Abreau Fianza, about their predicament. As Alice Fianza’s  brother-in-law,  Edwin  Fianza,  was  an  NBI  agent,  Alice  and Abratique  phoned him and disclosed what had transpired.[16]

On the morning of July 1, 1998, alerted by information that appellant would retrieve the sacks of prohibited drugs that day, Edwin Fianza and other NBI operatives conducted a stake out at No. 27, Dr. Cariño St. While the NBI agents were conducting their surveillance, they noticed that several PNP   NARCOM   personnel   were   also   watching   the   place.[17] The   NBI   then   learned   that   the   PNP NARCOM had received a tip from one of their informers regarding the presence of a huge amount of drugs in that place. The NBI and PNP NARCOM agreed to have a joint operation.

As the day wore on and appellant did not show up, the NBI agents became apprehensive that the whole operation could be jeopardized. They sought the permission of Nazarea Abreau to enter the room rented by appellant. She acceded and allowed them entry. The NBI team then searched the rented premises and found four (4) boxes and thirteen (13) sacks of marijuana, totaling 336.93 kilograms.[18]

Later that evening, NBI Special Agent Darwin Lising, with Abratique as his witness, applied for a search warrant from RTC Judge Antonio Reyes at his residence.[19] Judge Reyes ordered the NBI agents to fetch the Branch Clerk of Court, Atty. Delilah Muñoz, so the proceedings could be properly recorded. After Atty. Muñoz arrived, Judge Reyes questioned Lising and Abratique.  Thereafter, the 

39

Page 40: Search and Seizure Cases

judge issued a warrant directing the NBI to search appellant’s residence at Km. 6, Dontogan, Green Valley, Baguio City for marijuana.[20]

The NBI  operatives,  with some PNP NARCOM personnel   in   tow,  proceeded to appellant’s residence where they served the warrant upon appellant himself.[21] The search was witnessed by appellant,  members of his family,  barangay officials,  and members of the media.[22] Photographs were  taken  during   the  actual   search.[23] The   law enforcers   found 26  boxes  and a  sack  of  dried marijuana[24] in the water tank, garage, and storeroom of appellant’s residence.[25] The total weight of the haul was 591.81 kilograms.[26] Appellant was arrested for illegal possession of marijuana.

The seized items were then submitted to the NBI laboratory for testing. NBI Forensic Chemist Maria   Carina   Madrigal   conducted   the   tests.   Detailed   microscopic   and   chromatographic examinations of the items taken from appellant’s rented room at No. 27, Dr. Cariño St., as well as those from his residence at Green Valley, showed these to be marijuana.[27]

In his defense, appellant contended that the physical evidence of the prosecution was illegally obtained, being the products of an unlawful search, hence inadmissible. Appellant insisted that the search warrant was too general and the process by which said warrant was acquired did not satisfy the constitutional requirements for the issuance of a valid search warrant. Moreover, Abratique’s testimony, which was heavily relied upon by the judge who issued the warrant, was hearsay.

In Criminal Case No. 15822-R, the trial  court agreed with appellant that the taking of the 336.93   kilograms   of   marijuana   was   the   result   of   an   illegal   search   and   hence,   inadmissible   in evidence against appellant. Appellant was accordingly acquitted of the charge. However, the trial court  found that the prosecution’s  evidence was more than ample to prove appellant’s  guilt   in Criminal   Case   No.   15800-R   and   as   earlier   stated,   duly   convicted   him   of   illegal   possession   of marijuana and sentenced him to death.

Hence, this automatic review.

Before us, appellant submits that the trial court erred in:

1…UPHOLDING THE LEGALITY OF THE SEARCH WARRANT DESPITE LACK OF COMPLIANCE OF (sic) SEVERAL REQUIREMENTS BEFORE IT SHOULD HAVE BEEN ISSUED AND IT BEING A GENERAL WARRANT;

2….GRAVELY ABUSED ITS DISCRETION IN REOPENING THE CASE AND ALLOWING ABRITIQUE TO TESTIFY AGAINST APPELLANT;

3…GIVING CREDENCE TO THE TESTIMONY OF ABRITIQUE;

4. NOT ACQUITTING THE ACCUSED IN BOTH CASES AND SENTENCING HIM TO DEATH DESPITE THE ILLEGALLY OBTAINED EVIDENCE AS FOUND IN THE FIRST CASE.[28]

We find that the pertinent issues for resolution concern the following: (1) the validity of the search conducted at the appellant’s residence; (2) the alleged prejudice caused by the reopening of the case and absences  of   the prosecution witness,  on appellant’s   right   to  speedy trial;   (3)   the sufficiency of the prosecution’s evidence to sustain a finding of guilt with moral certainty; and (4) the propriety of the penalty imposed.

1. On the Validity of the Search Warrant; Its Obtention and Execution

Appellant initially contends that the warrant, which directed the peace officers to search for and seize “an undetermined amount of marijuana,” was too general and hence, void for vagueness. He insists that Abratique could already estimate the amount of marijuana supposed to be found at appellant’s residence since Abratique helped to transport the same.

For the appellee, the Office of the Solicitor General (OSG) counters that a search warrant is issued if a judge finds probable cause that the place to be searched contains prohibited drugs, and not that he believes the place contains a specific amount of it. The OSG points out that, as the trial court observed, it is impossible beforehand to determine the exact amount of prohibited drugs that a person has on himself.

Appellant  avers   that   the  phrase  “an  undetermined amount  of  marijuana”  as  used  in   the search warrant fails to satisfy the requirement of Article III, Section 2[29] of the Constitution that the things to be seized must be particularly described. Appellant’s contention, in our view, has no leg to stand on. The constitutional requirement of reasonable particularity of description of the things to be seized is primarily meant to enable the law enforcers serving the warrant to: (1) readily identify the properties to be seized and thus prevent them from seizing the wrong items;[30] and (2) leave said   peace   officers   with   no   discretion   regarding   the   articles   to   be   seized   and   thus   prevent unreasonable searches and seizures.[31] What the Constitution seeks to avoid are search warrants of broad or general characterization or sweeping descriptions, which will authorize police officers to undertake a  fishing expedition to  seize and confiscate any and all  kinds  of  evidence or  articles relating   to  an  offense.[32] However,   it   is  not   required   that   technical  precision  of  description  be required,[33] particularly, where by the nature of the goods to be seized, their description must be rather general, since the requirement of a technical description would mean that no warrant could issue.[34]

Thus, it has been held that term “narcotics paraphernalia” is not so wanting in particularity as to create a general warrant.[35] Nor is the description “any and all narcotics” and “all implements, paraphernalia, articles, papers and records pertaining to” the use, possession, or sale of narcotics or dangerous drugs so broad as to be unconstitutional.[36] A search warrant commanding peace officers to seize “a quantity of loose heroin” has been held sufficiently particular.[37]

Tested   against   the   foregoing   precedents,   the   description   “an   undetermined   amount   of marijuana”   must   be   held   to   satisfy   the   requirement   for   particularity   in   a   search   warrant. Noteworthy,  what   is   to  be   seized   in   the   instant   case   is  property  of   a   specified   character, i.e., marijuana, an illicit drug. By reason of its character and the circumstances under which it would be found, said article is illegal. A further description would be unnecessary and ordinarily impossible, except as to such character, the place, and the circumstances.[38]  Thus, this Court has held that the description “illegally in possession of undetermined quantity/amount of dried marijuana leaves and Methamphetamine Hydrochloride (Shabu) and sets of paraphernalia” particularizes the things to be seized.[39]

The  search  warrant   in   the  present   case,   given   its  nearly   similar  wording,   “undetermined amount of marijuana or Indian hemp,” in our view, has satisfied the Constitution’s requirements on 

40

Page 41: Search and Seizure Cases

particularity  of  description.  The  description   therein   is:   (1)  as   specific  as   the  circumstances  will ordinarily allow; (2) expresses a conclusion of fact – not of law – by which the peace officers may be guided in making the search and seizure; and (3) limits the things to be seized to those which bear direct   relation  to   the  offense   for  which   the  warrant   is  being   issued.[40] Said  warrant   imposes  a meaningful restriction upon the objects to be seized by the officers serving the warrant.  Thus, it prevents exploratory searches, which might be violative of the Bill of Rights.

Appellant next assails the warrant for merely stating that he should be searched, as he could be guilty of violation of Republic Act No. 6425. Appellant claims that this is a sweeping statement as said   statute   lists  a  number  of  offenses  with   respect   to   illegal  drugs.  Hence,  he  contends,   said warrant is a general warrant and is thus unconstitutional.

For the appellee, the OSG points out that the warrant clearly states that appellant has in his possession and control marijuana or Indian hemp, in violation of Section 8 of Republic Act No. 6425.

We   have   carefully   scrutinized   Search   Warrant   No.   415   (7-98),[41] and   we   find   that   it   is captioned “For Violation of R.A. 6425, as amended.”[42] It is clearly stated in the body of the warrant that   “there   is   probable   cause   to   believe   that   a   case   for   violation   of   R.A.   6425,   as   amended, otherwise known as the Dangerous Drugs Act of 1972, as further amended by R.A. 7659 has been and is being committed by one MODESTO TEE a.k.a. ESTOY TEE of Km. 6, Dontogan Bgy., Green Valley, Sto. Tomas, Baguio City by having in his possession and control an UNDETERMINED AMOUNT OF MARIJUANA or INDIAN HEMP in violation of the aforementioned law.”[43] In an earlier case, we held that though the specific section of the Dangerous Drugs Law is not pinpointed, “there is no question at all of the specific offense alleged to have been committed as a basis for the finding of probable   cause.”[44]Appellant’s  averment   is,   therefore,  baseless.  Search  Warrant  No.  415   (7-98) appears clearly issued for one offense, namely, illegal possession of marijuana.

Appellant next faults the Judge who issued Search Warrant No. 415 (7-98) for his failure to exhaustively   examine   the  applicant   and   his   witness.   Appellant   points  out   that   said   magistrate should not have swallowed all of Abratique’s statements – – hook, line, and sinker. He points out that since Abratique consented to assist in the transport of the marijuana, the examining judge should have elicited from Abratique his participation in the crime and his motive for squealing on appellant. Appellant further points out that the evidence of the NBI operative who applied for the warrant is merely hearsay and should not have been given credit at all by Judge Reyes.

Again, the lack of factual basis for appellant’s contention is apparent. The OSG points out that Abratique personally assisted appellant in  loading and transporting the marijuana to the latter’s house and to appellant’s rented room at No. 27 Dr. Cariño St., Baguio City. Definitely, this indicates personal   knowledge  on  Abratique’s  part.   Law enforcers   cannot   themselves  be  eyewitnesses   to every crime; they are allowed to present witnesses before an examining judge. In this case, witness Abratique  personally   saw and  handled   the  marijuana.  Hence,   the  NBI  did  not   rely  on  hearsay information in applying for a search warrant but on personal knowledge of the witness, Abratique.

Before a valid search warrant is issued, both the Constitution[45] and the 2000 Revised Rules of Criminal  Procedure[46] require   that   the   judge  must  personally  examine   the  complainant  and  his witnesses   under   oath   or   affirmation.   The   personal   examination   must   not   be   merely   routinary or pro forma,  but must be probing and exhaustive.[47] In the  instant case,   it   is  not disputed that Judge Antonio Reyes personally examined NBI Special Investigator III Darwin A. Lising, the applicant 

for the search warrant as well as his witness, Danilo G. Abratique. Notes of the proceedings were taken by Atty. Delilah Muñoz, Clerk of Court, RTC of Baguio City, Branch 61, whom Judge Reyes had ordered to be summoned.  In the letter of transmittal of the Clerk of Court of the RTC of Baguio City, Branch 61 to Branch 6  of  said court,  mention  is  made of  “notes”  at  “pages 7-11.” [48]  We have thoroughly   perused   the   records   of   Search   Warrant   No.   415   (7-98)   and   nowhere   find   said “notes.”  The depositions of Lising and Abratique were not attached to Search Warrant No. 415 (7-98) as required by the Rules of Court. We must stress, however, that the purpose of the Rules in requiring  depositions  to  be  taken  is   to   satisfy   the examining  magistrate  as   to   the existence of probable cause.[49] The Bill of Rights does not make it an imperative necessity that depositions be attached   to   the   records   of   an   application   for   a   search   warrant.   Hence,   said   omission   is   not necessarily   fatal,   for  as   long  as   there   is  evidence  on   the   record   showing  what   testimony  was presented.[50] In the testimony of witness Abratique, Judge Reyes required Abratique to confirm the contents of his affidavit;[51] there were instances when Judge Reyes questioned him extensively.[52] It is  presumed   that  a   judicial   function  has  been   regularly  performed,[53] absent  a   showing   to   the contrary. A magistrate’s determination of probable cause for the issuance of a search warrant is paid   great   deference   by   a   reviewing   court,[54] as   long   as   there   was   substantial   basis   for   that determination.[55] Substantial basis means that the questions of the examining judge brought out such facts and circumstances as would lead a reasonably discreet and prudent man to believe that an offense has been committed, and the objects in connection with the offense sought to be seized are in the place sought to be searched.

On record, appellant never raised the want of adequate depositions to support Warrant No. 415   (7-98)   in  his  motion   to  quash  before   the   trial   court.   Instead,  his  motion contained  vague generalities that Judge Reyes failed to ask searching questions of the applicant and his witness. Belatedly, however, he now claims that Judge Reyes perfunctorily examined said witness. [56] But it is settled that when a motion to quash a warrant is filed, all grounds and objections then available, existent or known, should be raised in the original or subsequent proceedings for the quashal of the warrant, otherwise they are deemed waived.[57]

In this case, NBI Special Investigator Lising’s knowledge of the illicit drugs stored in appellant’s house was indeed hearsay. But he had a witness, Danilo Abratique, who had personal knowledge about said drugs and their particular location. Abratique’s statements to the NBI and to Judge Reyes contained credible and reliable details.  As the NBI’s witness,  Abratique was a person on whose statements Judge Reyes could rely. His detailed description of appellant’s activities with respect to the seized drugs was substantial.  In relying on witness Abratique, Judge Reyes was not depending on casual rumor circulating in the underworld, but on personal knowledge Abratique possessed.

In Alvarez vs. Court of First Instance of Tayabas, 64 Phil. 33, 44 (1937), we held that:

The true test of sufficiency of a deposition or affidavit to warrant issuance of a search warrant is whether it has been drawn in such a manner that perjury could be charged thereon and affiant be held liable for damages caused.[58]

Appellant argues that the address indicated in the search warrant did not clearly indicate the place to be searched.  The OSG points out that the address stated in the warrant is as specific as can 

41

Page 42: Search and Seizure Cases

be. The NBI even submitted a detailed sketch of the premises prepared by Abratique, thus ensuring that there would be no mistake.

A description of the place to be searched is sufficient if the officer serving the warrant can, with reasonable effort,  ascertain and identify the place intended[59] and distinguish it  from other places in the community.[60] A designation or description that points out the place to be searched to the exclusion of all  others, and on inquiry unerringly  leads the peace officers to  it,  satisfies the constitutional requirement of definiteness.

Appellant finally harps on the use of unnecessary force during the execution of the search warrant. Appellant fails, however, to point to any evidentiary matter in the record to support his contention. Defense witness Cipriana Tee, appellant’s mother, testified on the search conducted but she said nothing that indicated the use of force on the part of the NBI operatives who conducted the search and seizure.[61] What the record discloses is that the warrant was served on appellant,[62] who was given time to read it,[63] and the search was witnessed by the barangay officials, police operatives, members of the media, and appellant’s kith and kin.[64] No breakage or other damage to the   place   searched   is   shown.  No   injuries   sustained   by   appellant,   or   any   witness,   appears   on record.  The execution of the warrant, in our view, has been orderly and peaceably performed.

2. On The Alleged Violation of Appellant’s Substantive RightsAppellant   insists   that   the  prosecution’s  unjustified and willful  delay   in  presenting witness 

Abratique unduly delayed the resolution of his case. He points out that a total of eight (8) scheduled hearings had to be reset due to the failure or willful refusal of Abratique to testify against him. Appellant insists that said lapse on the prosecution’s part violated Supreme Court Circular No. 38-98.[65] Appellant now alleges that the prosecution deliberately resorted to delaying the case to cause him untold miseries.

For the appellee, the OSG points out that the two-month delay in the trial is not such a great length of time as to amount to a violation of appellant’s right to a speedy trial. A trial is always subject   to   reasonable  delays  or  postponements,  but  absent  any showing that  these delays  are capricious   and   oppressive,   the   State   should   not   be   deprived   of   a   reasonable   opportunity   to prosecute the criminal action.

On record, the trial court found that prosecution witness Danilo G. Abratique failed to appear in no less than eighteen (18) hearings, namely those set for February 1, 2, 3, 4, 8, 9, 10, and 24; March 9, 15, 22, and 23; April 6, 7, 8, 16, and 19, all in 1999.[66] No less than four (4) warrants of arrest were issued against him to compel him to testify.[67] The NBI agent who supposedly had him in custody was found guilty of contempt of court for failing to produce Abratique at said hearings and sanctioned.[68] The prosecution had to write the NBI Regional Director in Baguio City and NBI Director   in   Manila   regarding   the   failure   of   the   Bureau’s   agents   to   bring   Abratique   to   court.[69] Nothing on record discloses the reason for Abratique’s aforecited absences.  On the scheduled hearing of June 7, 1999, he was again absent thus causing the trial court to again order his arrest for the fifth time.[70] He also failed to show up at the hearing of June 8, 1999.[71]

Appellant now stresses that the failure of Abratique to appear and testify on twenty (20) hearing dates violated appellant’s constitutional[72] and statutory right to a speedy trial.

A speedy trial means a trial conducted according to the law of criminal procedure and the rules and regulations, free from vexatious, capricious, and oppressive delays.[73] InConde v. Rivera and Unson, 45 Phil. 650, 652 (1924), the Court held that “where a prosecuting officer, without good cause, secures postponements of the trial of a defendant against his protest beyond a reasonable period of  time, as  in this  instance,   for  more than a year,  the accused  is  entitled to relief  by a proceeding in mandamus to compel a dismissal of the information, or if  he be restrained of his liberty, by habeas corpus to obtain his freedom.”

The concept of speedy trial is necessarily relative. A determination as to whether the right has been violated involves the weighing of several factors such as the length of the delay, the reason for the delay, the conduct of the prosecution and the accused, and the efforts exerted by the defendant to assert his right, as well as the prejudice and damage caused to the accused.[74]

The Speedy Trial Act of 1998, provides that the trial period for criminal cases in general shall be one hundred eighty (180) days.[75] However, in determining the right of an accused to speedy trial, courts should do more than a mathematical computation of the number of postponements of the scheduled hearings of the case.[76] The right to a speedy trial is deemed violated only when: (1) the   proceedings   are   attended   by   vexatious,   capricious,   and   oppressive   delays;[77] or   (2)   when unjustified postponements are asked for and secured;[78] or (3) when without cause or justifiable motive a long period of time is allowed to elapse without the party having his case tried.[79]

In the present case, although the absences of prosecution witness Abratique totaled twenty (20) hearing days, there is no showing whatsoever that prosecution capriciously caused Abratique’s absences so as to vex or oppress appellant and deny him his rights.  On record, after Abratique repeatedly failed to show up for the taking of his testimony, the prosecution went to the extent of praying that the trial  court order the arrest of Abratique to compel his attendance at trial.  The prosecution likewise tried to get the NBI to produce Abratique as the latter was in the Bureau’s custody, but to no avail. Eventually, the trial court ordered the prosecution to waive its right to present Abratique and rest its case on the evidence already offered.[80]

Nor do we find a delay of twenty (20) hearing days to be an unreasonable length of time. Delay of less than two months has been found, in fact, to be not an unreasonably lengthy period of time.[81]

Moreover, nothing on record shows that appellant Modesto Tee objected to the inability of the prosecution to produce its witness.  Under the Rules, appellant could have moved the trial court to require that witness Abratique post bail to ensure that the latter would testify when required.[82] Appellant   could   have   moved   to   have   Abratique   found   in   contempt   and   duly sanctioned.  Appellant did neither.  It is a bit too late in the day for appellant to invoke now his right to speedy trial.

No persuasive reason supports appellant’s claim that his constitutional right to speedy trial was violated. One must take into account that a trial is always subject to postponements and other causes of delay.  But in the absence of a showing that delays were unreasonable and capricious, the State should not be deprived of a reasonable opportunity of prosecuting an accused.[83]

Appellant   next   contends   that   the   trial   court   gravely   abused   its   discretion,   and  exhibited partiality, when it allowed the reopening of the case after the prosecution had failed to present 

42

Page 43: Search and Seizure Cases

Abratique on several occasions and had been directed to rest its case. Appellant stresses that the lower court’s order to reopen the case to receive Abratique’s further testimony is an indication that the trial court favored the prosecution and unduly prejudiced appellant.

On appellee’s behalf, the Solicitor General points out that the trial court’s order was in the interest of substantial justice and hence, cannot be termed as an abuse of discretion. The OSG points out that the prosecution had not formally rested its case and had yet to present its formal offer of evidence, hence, the submission of additional testimony by the same witness cannot be prejudicial   to   the   accused,   it   being   but   the   mere   continuation   of   an   uncompleted   testimony. Furthermore, appellant did not properly oppose the prosecution’s motion to reopen the case.

At the time Criminal Cases Nos. 15800-R and 15822-R were being tried, the 1985 Rules of Criminal Procedure were in effect. There was no specific provision at that time governing motions to reopen.[84] Nonetheless, long and established usage has led to the recognition and acceptance of a motion to reopen. In view of the absence of a specific procedural rule, the only controlling guideline governing a motion to reopen was the paramount  interests  of   justice.  As a rule,  the matter of reopening of a case for reception of further evidence after either prosecution or defense has rested its case is within the discretion of the trial court.[85] However, a concession to a reopening must not prejudice the accused or deny him the opportunity to introduce counter evidence.[86]

Strictly speaking, however, there was no reopening of the cases in the proceedings below. A motion to reopen may properly be presented only after either or both parties have formally offered and closed their evidence, but before judgment.[87] In the instant case, the records show that on April 19, 1999, the prosecution was directed to close its evidence and given 15 days to make its formal offer of evidence.[88] This order apparently arose from the manifestation of the prosecution on April 16, 1999 that should they fail to produce witness Abratique on the next scheduled hearing the prosecution would rest  its case.[89] On April  19, 1999, which was the next scheduled hearing after April 16, 1999, Abratique was absent notwithstanding notices, orders, and warrants of arrest. However, on April 27, 1999, or before the prosecution had formally offered its evidence, Abratique was brought to the trial court by the NBI. In its order of said date, the trial court pointed out that the prosecution could move to “reopen” the case for the taking of Abratique’s testimony.[90] On May 7, 1999, the prosecution so moved, stressing that it had not yet formally offered its evidence and that the substantial rights of the accused would not be prejudiced inasmuch as the latter had yet to present  his  evidence.  Appellant  filed  no  opposition   to   the  motion.  The   trial   court  granted   the motion six days later. Plainly, there was nothing to reopen, as the prosecution had not formally rested   its   case. Moreover,   the   taking   of   Abratique’s   testimony   was   not   for   the   purpose of presenting   additional   evidence,   but   more   properly   for   the completion of   his   unfinished testimony.   In U.S. vs. Base,[91] we held that  a  trial  court   is  not  in error,   if   it  opts to reopen the proceedings of a case, even after both sides had rested and the case submitted for decision, by the calling  of   additional  witnesses  or   recalling  of  witnesses   so  as   to   satisfy   the   judge’s  mind  with reference to particular facts involved in the case. A judge cannot be faulted should he require a material witness to complete his testimony, which is what happened in this case. It is but proper that the judge’s mind be satisfied on any and all questions presented during the trial, in order to serve the cause of justice.

Appellant’s claim that the trial court’s concession to “reopen” the case unduly prejudiced him is not well taken. We note that appellant had every opportunity to present his evidence to support 

his case or to refute the prosecution’s evidence point-by-point, after the prosecution had rested its case. In short, appellant was never deprived of his day in court. A day in court is the touchstone of the right to due process in criminal justice.[92] Thus, we are unable to hold that a grave abuse of discretion was committed by the trial court when it ordered the so-called “reopening” in order to complete the testimony of a prosecution witness.

3. On the Sufficiency of the Prosecution’s EvidenceIn bidding for acquittal, appellant assails the credibility of Abratique as a witness.  Appellant 

insists   that   Abratique’s   testimony   is   profuse   with   lies,   contrary   to   human   nature,   hence incredible.  According to appellant, Abratique was evasive from the outset with respect to certain questions   of   the   trial   court.  He   adds   that   it   appeared   the   court   entertained   in   particular   the suspicion that witness Abratique had conspired with appellant in committing the crime charged. Appellant  questions Abratique’s  motive  in   informing   the NBI  about  his  activities   related  to   the marijuana taking, transfer, and warehousing.

The OSG contends that Abratique’s testimony, taken as a whole, is credible. It points out that Abratique testified in a straightforward manner as to his knowledge of the huge cache of prohibited drugs stashed by appellant in two different places. His testimony, said the OSG, when fused with the physical evidence consisting of 591.81 kilograms of marijuana found by law enforcers at appellant’s residence, inexorably leads to the inculpation of appellant.

It is the bounden duty of the courts to test the prosecution evidence rigorously, so that no innocent  person  is  made to suffer the unusually  severe penalties meted out  for  drug offenses.[93] Though   we   scrutinized   minutely   the   testimony   of   Abratique,   we   find   no   cogent   reason   to disbelieve him. From his account, Abratique might appear aware treading the thin line between innocence   and   feeling   guilty,   with   certain   portions   of   his   story   tending   to   be   self-exculpatory.   However, his whole testimony could not be discredited.  The established rule is that testimony of a witness may be believed in part and disbelieved in other parts, depending on the corroborative evidence and the probabilities and improbabilities of the case. But it is accepted, as a matter of common sense, that if certain parts of a witness’ testimony are found true, his testimony cannot be disregarded entirely.[94]

Abratique testified in open court that appellant rented the taxicab he was driving, and he helped appellant transport huge amounts of marijuana to appellant’s rented room at No. 27 Dr. Cariño St., Baguio City and to appellant’s residence at Km. 6, Dontogan, Green Valley, Sto. Tomas, Baguio City. He also declared on the witness stand that out of fear of being involved, he decided to divulge his knowledge of appellant’s possession of large caches of marijuana to the NBI. When the places referred to by Abratique were searched by the authorities, marijuana in staggering quantities was   found   and   seized   by   the   law  enforcers.   Stated  plainly,   the   physical   evidence   in   this   case corroborated Abratique’s testimony on material points.

Appellant   imputes   questionable   motives   to   Abratique   in   an   effort   to   discredit   him.   He demands that Abratique should likewise be prosecuted. However, by no means is the possible guilt of Abratique a tenable defense for appellant. Nor would Abratique’s prosecution mean appellant’s absolution.

43

Page 44: Search and Seizure Cases

In a prosecution for illegal possession of dangerous drugs, the following facts must be proven with moral certainty: (1) that the accused is in possession of the object identified as prohibited or regulated drug; (2) that such possession is not authorized by law; and (3) that the accused freely and consciously possessed the said drug.[95]

We find the  foregoing  elements  proven  in  Criminal  Case No.  15800-R  beyond reasonable doubt.

In said case, the testimony of Abratique and the recovery of 591.81 kilograms of marijuana from   appellant’s   residence   served   to   prove   appellant’s   possession   of   a   prohibited   drug.   Tests conducted by the NBI forensic chemist proved the seized articles to be marijuana. These articles were seized pursuant to a valid search warrant and hence, fully admissible in evidence.

In People v. de los Reyes, 239 SCRA 439 (1994), we held that the Dangerous Drugs Act applies generally to all persons and proscribes the sale of dangerous drugs by any person, and no person is authorized  to  sell   such drugs.  Said  doctrine   is  equally  applicable  with   respect   to  possession of prohibited drugs. Republic Act No. 6425, which penalizes the possession of prohibited drugs, applies equally to all persons in this jurisdiction and no person is authorized to possess said articles, without authority of law.

Anent the third element, we have held that to warrant conviction, possession of illegal drugs must   be   with   knowledge   of   the   accused   or   that animus possidendi existed   together   with   the possession or control of said articles.[96] Nonetheless, this dictum must be read in consonance with our ruling that possession of a prohibited drug per se constitutes prima facie evidence of knowledge or animus possidendi sufficient   to   convict  an  accused  absent  a   satisfactory  explanation  of   such possession.[97] In effect, the onus probandi is shifted to accused to explain the absence of knowledge or animus possidendi[98] in this situation.

Appellant Modesto Tee opted not to testify in his defense. Instead, he presented his mother as his lone witness, who testified on matters totally irrelevant to his case.   We can only conclude that,   failing   to   discharge   the   burden   of   the   evidence   on   the   possession   of   prohibited   drug, appellant’s guilt in Criminal Case No. 15800-R was established beyond reasonable doubt.

3. On The Proper PenaltyUnder Republic Act No. 6425 as amended by Republic Act No. 7659, the penalty of reclusion

perpetua to death and a fine ranging from five hundred thousand pesos (P500,000.00) to ten million pesos (P10,000,000.00)[99] shall be imposed if the quantity of marijuana involved in a conviction for possession of marijuana or Indian hemp shall be 750 grams or more.[100]

In the present case, the quantity of marijuana involved has been shown by the prosecution to be far in excess of 750 grams, as stressed by the trial court:

The volume is rather staggering. It is almost one whole house or one whole room. In fact, when they were first brought to the court, it took hours to load them on the truck and hours also to unload them prompting the court to direct that the boxes and sack of marijuana be instead kept at the NBI office in Baguio. And the identification of said marijuana during the trial was made in the NBI premises itself by the witnesses since it was physically cumbersome and inconvenient to keep bringing them to the court during every trial.[101]

In sentencing appellant to death, the trial court noted not only the huge quantity of marijuana bales involved, but also “the acts of accused of hiding them in different places…and transferring them from place   to  place and making  them appear  as  boxes  of  cigarettes  to  avoid  and evade apprehension and detection.” They showed his being a big supplier, said the trial court, [whose] criminal perversity and craft that “deserve the supreme penalty of death.”[102]

We are unable to agree, however, with the penalty imposed by the trial court. The legislature never intended that where the quantity involved exceeds those stated in Section 20 of Republic Act No. 6425 the maximum penalty of death shall automatically be imposed.[103] The statute prescribes two  indivisible  penalties: reclusion perpetua and death.  Hence,  the penalty  to be  imposed must conform with  Article  63[104] of   the Revised Penal  Code.  As  already held,   the death penalty   law, Republic Act No. 7659 did not amend Article 63 of the Revised Penal Code.[105]  The rules in Article 63 apply although the prohibited drugs involved are in excess of the quantities provided for in Section 20 of Republic Act No. 6425.[106]Thus, finding neither mitigating nor aggravating circumstances in the present case, appellant’s possession of 591.81 kilograms of marijuana in Criminal Case No. 15800-R, does not merit capital punishment but only the lesser penalty of reclusion perpetua.

The trial court imposed a fine on appellant in the sum of One Million Pesos (P1,000,000.00), without subsidiary imprisonment in case of  insolvency.  The imposition of a fine is mandatory in cases of conviction of possession of illegal drugs. This being within the limits allowed by the law, the amount of the fine must be sustained.  All these sanctions might not remedy all the havoc wrought by prohibited drugs on the moral fiber of our society, especially the youth.[107] But these penalties should  warn  peddlers  of  prohibited  drugs   that   they  cannot  ply   their   trade  in  our   streets  with impunity.

WHEREFORE,  the decision of the Regional Trial Court of Baguio City, Branch 6,  in Criminal Case No. 15800-R, convicting appellant MODESTO TEE alias “ESTOY” TEE of violation of Section 8 of Republic Act No. 6425, as amended, is AFFIRMED with the MODIFICATION that appellant is hereby sentenced to suffer the penalty of reclusion perpetua. The fine of  ONE MILLION (P1,000,000.00) PESOS imposed on him is sustained.  Appellant is likewise directed to pay the costs of suit.

PAPER INDUSTRIES CORPORATION OF THE PHILIPPINES, EVARISTO M. NARVAEZ JR., RICARDO G. SANTIAGO, ROBERTO A. DORMENDO, REYDANDE D. AZUCENA, NICEFORO V. AVILA, FLORENTINO M. MULA, FELIX O. BAITO, HAROLD B. CELESTIAL, ELMEDENCIO C. CALIXTRO, CARLITO S. LEGACION, ALBINO T. LUBANG, JEREMIAS I. ABAD and HERMINIO V. VILLAMIL,petitioners, vs. JUDGE MAXIMIANO C. ASUNCION, Presiding Judge, Branch 104, Regional Trial Court of Quezon City; STATE PROSECUTOR LEO B. DACERA III; and the SPECIAL OPERATIONS UNIT OF THE PNP TRAFFIC MANAGEMENT COMMAND,respondents.

To   preserve   and   to   uphold   the   constitutional   right   against   unreasonable   searches   and seizures, the requisites for the issuance of a search warrant must be followed strictly.  Where the judge fails topersonally examine the applicant for a search warrant and the latter’s witnesses, or where the witnesses testify on matters not of their own personal knowledge, the search warrant must be struck down.

The Case

44

Page 45: Search and Seizure Cases

Before us is a Petition for Certiorari and Prohibition[1] praying for (1) the nullification of Search Warrant No. 799 (95) and the Orders dated March 23, 1993 and August 3, 1995,  issued by the Regional   Trial  Court   (RTC),  Branch  104,  of  Quezon  City;[2] and   (2)   the   issuance  of   a   temporary restraining order (TRO) or an injunction against State Prosecutor Leo B. Dacera III, ordering him to desist from proceeding with IS No. 95-167.

In its October 23, 1995 Resolution,[3] this Court issued the TRO prayed for and required the respondents to comment on the said Petition.  On December 20,  1995,  Respondent PNP Traffic Management Command filed its 31-page Opposition[4] to the Petition, together with 90 pages of annexes.[5] On February 22, 1996, the Office of the Solicitor General filed its Comment[6] agreeing with petitioners that the writs prayed for must be granted.  After petitioners filed a Reply to the Opposition,  the Court  gave due course to the Petition and required the parties to submit  their respective memoranda.

In view of the contrary opinion of the Office of the Solicitor General, the Court, in its February 5, 1997 Resolution,[7] required State Prosecutor Leo B. Dacera to prepare the memorandum for the public respondents.  After issuing a show-cause order to Dacera on June 23, 1997,[8] the Court in its September 24,  1997 Resolution gave him a non-extendible  period ending on October  31,  1997 within which to file the required memorandum.  In view of Dacera’s manifestation that he was only a nominal party and that he had yet to receive the records of the case from the PNP, the Court, in its  December 8,  1999 Resolution,  ordered the Special  Operations Unit   (SOU) of  the PNP Traffic Management Command to file  its memorandum within thirty days from notice; “otherwise,  the petition will be deemed submitted for decision.”[9] Even after the expiration of the said period, the required pleading was not yet received by this Court.

Hence, this Court considered Respondent SOU’s refusal/failure to submit its memorandum as a waiver of its privilege to do so.

The Facts On January 25, 1995, Police Chief Inspector Napoleon B. Pascua applied for a search warrant before the said RTC of Quezon City, stating:[10]

“1. That the management of Paper Industries Corporation of the Philippines, located at PICOP compound, Barangay Tabon, Bislig, Surigao del Sur, represented by its Sr. Vice President Ricardo G[.] Santiago, is in possession or ha[s] in [its] control high powered firearms, ammunitions, explosives, which are the subject of the offense, or used or intended to be used in committing the offense, and which xxx are [being kept] and conceal[ed] in the premises herein described.

“2. That a Search Warrant should be issued to enable any agent of the law to take possession and bring to this Honorable Court the following described properties:

'Seventy (70) M16 Armalite rifles cal. 5.56, ten (10) M16 US rifles, two (2) AK-47 rifle[s], two (2) UZI submachinegun[s], two (2) M203 Grenade Launcher[s] cal.40mm, ten (10) cal.45 pistol[s], ten (10) cal.38 revolver[s], two (2) ammunition reloading machine[s], assorted ammunitions for said calibers of firearms and ten (10) handgrenades.'

Attached to the application[11] were the joint Deposition of SPO3 Cicero S. Bacolod and SPO2 Cecilio T. Morito,[12] as well as a summary of the information and the supplementary statements of Mario Enad and Felipe Moreno.

After propounding several  questions to Bacolod,  Judge Maximiano C.  Asuncion  issued the contested search warrant,[13] the pertinent portion of which reads:

“It appearing to the satisfaction of the undersigned, after examining under oath, SPO3 Cicero S. Bacolod, that there is probable cause to believe that the management of Paper Industries Corporation of the Philippines, located at PICOP Compound, Barangay Tabon, Bislig, Surigao del Sur, represented by its Sr. Vice President Ricardo G. Santiago, has in its possession or control the following:

Seventy (70) M16 Armalite rifles cal. 5.56Ten (10) M14 US riflesTwo (2) AK-47 rifle[s]Two (2) UZI submachinegun[s]Two (2) M203 Grenade Launcher[s] cal. 40mm.Ten (10) cal 45 pistol[s]Ten (10) cal 38 revolver[s]Two (2) ammunition reloading machine[s]Assorted ammunitions for said calibers of firearmsTen (10) handgrenades

in violation of the Provisions of PD 1866 (Illegal Possession of Firearms, Ammunition and Explosives), and the same should be seized and brought before this Court.

“NOW, THEREFORE, you are hereby authorized to make an immediate search daytime between 8:00 a.m. [and] 4:00 p.m. of the aforementioned premises and to seize and bring the articles above-described and make an immediate return there[of]”[14]

On February 4, 1995, the police enforced the search warrant at the PICOP compound and seized the following:[15]

Guns guns guns

Believing that the warrant was invalid and the search unreasonable, the petitioners filed a “Motion to Quash”[16] before the trial court.  Subsequently, they also filed a “Supplemental Pleading to the Motion to Quash” and a “Motion to Suppress Evidence.”[17]

On   March   23,   1995,   the   RTC   issued   the   first   contested   Order   which   denied   petitioners’ motions.[18] On   August   3,   1995,   the   trial   court   rendered   its   second   contested   Order[19] denying petitioners’ Motion for Reconsideration.[20]

Hence, this recourse to this Court on pure questions of law.

45

Page 46: Search and Seizure Cases

Issues In their Memorandum, petitioners submit the following grounds in support of their cause:[21]

 “IPetitioners respectfully submit that Judge Asuncion has committed grave abuse of discretion or has exceeded his jurisdiction in refusing to quash Search Warrant No. 799(95).  Probable cause [has] 

not xxx been sufficiently established and partaking as it does of the nature of a general warrant.

“IIPetitioners respectfully submit that Judge Asuncion has committed grave abuse of discretion or has exceeded his jurisdiction in refusing to quash Search Warrant No. 799(95) on the ground that it 

was unlawfully served or implemented.

“IIIPetitioners respectfully submit that State Prosecutor Dacera is acting with grave abuse of discretion or exceeding his jurisdiction in continuing with the proceedings in IS No. 95-167 on the 

basis of illegally seized evidence.”

In the main, petitioners question the validity of the search warrant.  As a preliminary matter, we shall also discuss respondents’ argument that the Petition should be dismissed for raising factual questions.

This Court’s RulingIn   their   Opposition,   respondents   argue   that   the   Petition   should   be   dismissed   for   raising 

questions of fact, which are not proper in a petition for certiorari under Rule 65.  They maintain that the Petition merely assails the “factual basis for the issuance of the warrant and the regularity of its implementation.”[22]

This argument is not convincing.  It is settled that “there is a question of fact when the doubt arises  as   to   the   truth  or   the  falsity  of  alleged  facts.”[23] In   the present  case,  petitioners  do not question the truth of the facts as found by the judge; rather, they are assailing the way in which those findings were arrived at, a procedure which they contend was violative of the Constitution and the Rules of Court.  We agree that the Petition raises only questions of  law, which may be resolved  in the present case.

Main Issue:Validity of the Search Warrant

The fundamental right against unreasonable searches and seizures and the basic conditions for the issuance of a search warrant are laid down in Section 2, Article III of the 1987 Constitution, which reads:

“The right of the people to be secure in their persons, houses, papers and effects against unreasonable searches and seizures of whatever nature and for any purpose shall be inviolable, and no search warrant or warrant of arrest shall issue except upon probable cause to be determined personally by the judge after examination under oath or affirmation of the complainant and the witnesses he may produce, and particularly describing the place to be searched and the persons or things to be seized.” (Emphasis  supplied)

Consistent with the foregoing constitutional provision, Sections 3 and 4, Rule 126 of the Rules of Court,[24] detail the requisites for the issuance of a valid search warrant as follows:

“SEC. 3. Requisite for issuing search warrant. -- A search warrant shall not issue but upon probable cause in connection with one specific offense to be determined personally by the judge after examination under oath or affirmation of the complainant and the witnesses he may produce, and particularly describing the place to be searched and the things to be seized.”

“SEC. 4. Examination of complainant; record. -- The judge must, before issuing the warrant, personally examine in the form of searching questions and answers, in writing and under oath the complainant and any witnesses he may produce on facts personally known to them and attach to the record their sworn statements together with any affidavits submitted.”

More   simply   stated,   the   requisites   of   a   valid   search   warrant   are:  (1)   probable   cause   is present;   (2)  such presence  is  determined personally  by the  judge;   (3)  the complainant  and the witnesses he or she may produce are personally examined by the judge, in writing and under oath or affirmation; (4) the applicant and the witnesses testify on facts personally known to them; and (5) the warrant specifically describes the place to be searched and the things to be seized. [25] In the present case, the search warrant is invalid because (1) the trial court failed to examine personally the complainant  and the  other  deponents;   (2)  SPO3 Cicero  Bacolod,  who appeared during   the hearing for the issuance of the search warrant, had no personal knowledge that petitioners were not licensed to possess the subject firearms; and (3) the place to be searched was not described with particularity.

No Personal Examination of the WitnessesIn his Order dated March 23, 1995, the trial judge insisted that the search warrant was valid, 

stating that “before issuing the subject warrant, the court propounded searching questions to the applicant and the witnesses in   order   to   determine   whether   there   was   probable   cause   x   x x.”[26] (Emphasis supplied.)  This was supported by the Opposition to the Motion to Quash, which argued   that   “it   is   erroneous   for   PICOP   to   allege   that   the   Honorable   Court   did   not   propound searching   questions   upon   applicant   P/Chief   Inspector   Napoleon   Pascua   and   the   witnesses   he produced.”[27] The records, however, proclaim otherwise.

As earlier stated, Chief Inspector Pascua’s application for a search warrant was supported by (1) the joint Deposition of SPO3 Cicero S. Bacolod and SPO2 Cecilio T. Morito, (2) a summary of information   and   (3)   supplementary   statements   of   Mario   Enad   and   Felipe   Moreno.  Except   for Pascua and Bacolod,  however,  none of   the aforementioned witnesses  and policemen appeared before the trial court.  Moreover, the applicant’s participation in the hearing for the issuance of the search warrant consisted only of introducing Witness Bacolod:[28]

“COURT:          Where is the witness for this application for search warrant?P/Chief Insp. NAPOLEON PASCUA:          SPO3 CICERO S. BACOLOD, Your Honor.COURT:          Swear the witness.STENOGRAPHER: (To the witness)          Please raise your right hand, sir.  Do you swear to tell the truth, the whole truth and nothing but the truth before this Court?

46

Page 47: Search and Seizure Cases

WITNESS:          Yes Ma’am.STENOGRAPHER:          Please   state   your   name,   age,   civil   status,   occupation,   address   and   other   personal circumstances.WITNESS:

          SPO3 Cicero S. Bacolod, 42 years old, married, policeman, c/o Camp Crame, Quezon City, SOU, TMC.

Chief Inspector Pascua was asked nothing else, and he said nothing more.  In fact, he failed even   to   affirm   his   application.  Contrary   to   his   statement,   the   trial   judge   failed   to   propound questions, let alone probing questions, to the applicant and to his witnesses other than Bacolod (whose testimony, as will later be shown, is also improper).  Obviously, His Honor relied mainly on their affidavits.  This Court has frowned on this practice in this language:

“Mere affidavits of the complainant and his witnesses are thus not sufficient.  The examining Judge has to take depositions in writing of the complainant and the witnesses he may produce and attach them to the record.  Such written deposition is necessary in order that the Judge may be able to properly determine the existence or non-existence of the probable cause, to hold liable for perjury the person giving it if it will be found later that his declarations are false.

 “It is axiomatic that the examination must be probing and exhaustive, not merely routinary or pro-forma, if the claimed probable cause is to be established.  The examining magistrate must not simply rehash the contents of the affidavit but must make his own inquiry on the intent and justification of the application.”[29]

Bacolod’s Testimony Pertained Not to Facts Personally Known to HimBacolod appeared during the hearing and was extensively examined by the judge.  But his 

testimony showed that he did not have personal knowledge that the petitioners, in violation of PD 1866,  were  not   licensed   to  possess  firearms,  ammunitions  or  explosives.  In  his  Deposition,  he stated:

“Q   How do you know that said properties were subject of the offense?

A     Sir,   as   a   result   of   our   intensified   surveillance   and   case   build   up   for   several   days, we gathered informations from reliable sources that   subject   properties   [which]   are   in   their   possession   and control [are] the herein described properties subject of the offense.  (Summary of Information dtd Oct ‘94, SS’s of Mario Enad and Felipe Moreno both dtd 30 Nov ‘94 are hereto attached).”[30]

When   questioned   by   the   judge,   Bacolod   stated   merely   that   he   believed   that   the   PICOP security guards had no license to possess the subject firearms.  This, however, does not meet the requirement that a witness must testify on his personal knowledge, not belief.  He declared:

“Q   This   is  an  application  for  Search  Warrant  against  Paper   Industries  Corporation  located  at  PICOP Compound, Barangay Tabon, Bislig, Surigao del Sur.  How come that you have knowledge that there are illegal firearms in that place?

A     At Camp Crame, Quezon City, I was dispatched by our Commander to investigate the alleged assassination plot of Congressman Amante.Q     In the course of your investigation, what happened?A     We found out that some of the suspects in the alleged assassination plot are employees of PICOP.Q     Know[ing] that the suspects are employees of PICOP, what did you do?A     We conducted the surveillance in that area inside the compound of PICOP in Tabon.Q     What did you find xxx?A     I found xxx several high-powered firearms.Q     How were you able to investigate the compound of PICOP?A     I exerted effort to enter the said compound.Q     By what means?A     By pretending to have some official business with the company.Q     So, in that aspect, you were able to investigate the compound of PICOP?A     Yes, sir.Q     What did you f[i]nd xxxt?A     I found xxx several high-powered firearms being kept in the compound of PICOP.Q     Where are those located?A     Sir, there are firearms kept inside the ammo dam.Q     Inside the compound?A     Located inside the compound.Q     Then what?A     Others, sir, were kept in the security headquarters or office.Q     You mean to say that this Paper Industries Corporation has its own security guards?A     Yes, they call it Blue Guards.Q     You mean to say that their own security guards guarded the PICOP?A     Yes, sir.Q     So, it is possible that the firearms used by the security guards are illegally obtained?A     I believe they have no license to possess high-powered firearms. As far as the verification at FEU, Camp Crame, [is concerned,] they have no license. (Emphasis supplied.)Q     Have you investigated the Blue Guards Security Agency?A     I conducted the inquiry.Q     What did you find out?A     They are using firearms owned by PICOP.Q     Using firearms owned by PICOP?A     Yes, sir.Q     You mean to say that this Blue Guard Security Agency has no firearms of  their own?A     No high-powered firearms.Q     By the way, Mr. Witness, what kind of firearms have you seen inside the compound of PICOP?A     There are M-16 armalite rifles.Q     What else?

A     AK-47, armalites, M-203 Grenade Launcher, M-14 US rifles, .38 caliber revolvers,   .45 caliber pistols, several handgrenades and ammos.”[31] (Emphasis supplied)

47

Page 48: Search and Seizure Cases

Moreover, Bacolod failed to affirm that none of the firearms seen inside the PICOP compound was   licensed.  Bacolod   merely   declared   that   the   security   agency   and   its   guards   were   not licensed.  He also said that some of the firearms were owned by PICOP.  Yet, he made no statement before the trial court that PICOP, aside from the security agency, had no license to possess those firearms.  Worse, the applicant and his witnesses inexplicably failed to attach to the application a copy of the aforementioned “no license” certification from the Firearms and Explosives Office (FEO) of the PNP, or to present it during the hearing.  Such certification could have been easily obtained, considering   that   the   FEO   was   located   in   Camp   Crame   where   the   unit   of   Bacolod   was   also based.  In People v. Judge Estrada,[32] the Court held:

“The facts and circumstances that would show probable cause must be the best evidence that could be obtained under the circumstances.  The introduction of such evidence is necessary in cases where the issue is the existence of the negative ingredient of the offense charged – for instance, the absence of a license required by law, as in the present case – and such evidence is within the knowledge and control of the applicant who could easily produce the same.  But if the best evidence could not be secured at the time of the application, the applicant must show a justifiable reason therefor during the examination by the judge.”

Particularity of the Place to Be SearchedIn view of the manifest objective of the constitutional safeguard against unreasonable search, 

the Constitution and the Rules limit the place to be searched only to those described in the warrant.[33] Thus, this Court has held that “this constitutional right [i]s the embodiment of a spiritual concept: the belief that to value the privacy of home and person and to afford its constitutional protection against the long reach of government is no less than to value human dignity, and that his privacy must  not  be disturbed except   in case of  overriding social  need,  and then only  under stringent procedural safeguards.”[34] Additionally, the requisite of particularity is related to the probable cause requirement in that, at least under some circumstances, the lack of a more specific description will make it apparent that there has not been a sufficient showing to the magistrate that the described items are to be found in a particular place.[35]

In   the   present   case,   the   assailed   search   warrant   failed   to   describe   the   place   with particularity.  It simply authorizes a search of “the aforementioned premises,” but it did not specify such premises.  The warrant identifies only one place, and that is the “Paper Industries Corporation of   the Philippines,   located at  PICOP Compound,  Barangay Tabon,  Bislig[,]  Surigao del  Sur.”  The PICOP compound,  however,   is  made up of  “200 offices/buildings,  15 plants,  84  staff houses,  1 airstrip,   3   piers/wharves,   23   warehouses,   6   POL   depots/quick   service   outlets   and   some   800 miscellaneous   structures,   all   of   which   are   spread   out   over   some   one   hundred   fifty-five hectares.”[36] Obviously, the warrant gives the police officers unbridled and thus illegal authority to search all the structures found inside the PICOP compound.[37]

In their Opposition, the police state that they complied with the constitutional requirement, because   they   submitted   sketches   of   the   premises   to   be   searched   when   they   applied   for   the warrant.  They add that not one of the PICOP Compound housing units was searched, because they were not among those identified during the hearing.[38]

These arguments are not convincing.  The sketches allegedly submitted by the police were not made integral parts of the search warrant issued by Judge Asuncion.  Moreover, the fact that the 

raiding police   team knew which of   the buildings or  structures   in   the PICOP Compound housed firearms   and  ammunitions  did  not   justify   the   lack  of   particulars   of   the   place   to   be   searched.[39] Otherwise, confusion would arise regarding the subject of the warrant – the place indicated in the warrant or the place identified by the police.  Such conflict invites uncalled for mischief or abuse of discretion on the part of law enforcers.

Thus,  in People v. Court of Appeals,[40] this  Court ruled that the police had no authority to search the apartment behind the store, which was the place indicated in the warrant, even if they reallyintended it  to be the subject of their application.  Indeed, the place to be searched cannot be changed, enlarged or amplified by the police, viz.:

“x x x.  In the instant case, there is no ambiguity at all in the warrant.  The ambiguity lies outside the instrument, arising from the absence of a meeting of the minds as to the place to be searched between the applicants for the warrant and the Judge issuing the same; and what was done was to substitute for the place that the Judge had written down in the warrant, the premises that the executing officers had in their mind.  This should not have been done.  It [was] neither fair nor licit to allow police officers to search a place different from that stated in the warrant on the claim that the place actually searched – although not that specified in the warrant – [was] exactly what they had in view when they applied for the warrant and had demarcated in their supporting evidence.  What is material in determining the validity of a search is the place stated in the warrant itself, not what the applicants had in their thoughts, or had represented in the proofs they submitted to the court issuing the warrant. Indeed, following the officers’ theory, in the context of the facts of this case, all four (4) apartment units at the rear of Abigail's Variety Store would have been fair game for a search.

“The place to be searched, as set out in the warrant, cannot be amplified or modified by the officers’ own personal knowledge of the premises, or the evidence they adduced in support of their application for the warrant. Such a change is proscribed by the Constitution which requires inter alia the search warrant to particularly describe the place to be searched as well as the persons or things to be seized.  It would concede to police officers the power of choosing the place to be searched, even if it not be that delineated in the warrant.  It would open wide the door to abuse of the search process, and grant to officers executing a search warrant that discretion which the Constitution has precisely removed from them.  The particularization of the description of the place to be searched may properly be done only by the Judge, and only in the warrant itself; it cannot be left to the discretion of the police officers conducting the search.”  (Emphasis supplied.)Seized Firearms and Explosives Inadmissible in Evidence

As a result of the seizure of the firearms, effected pursuant to Search Warrant No. 799 (95) issued by the respondent judge, the PNP filed with the Department of Justice a complaint docketed as   IS  No.  95-167  against  herein  petitioners   for   illegal  possession  of  firearms.  State  Prosecutor Dacera,   to  whom the Complaint  was  assigned   for  preliminary   investigation,   issued a   subpoena requiring petitioners to file their counter-affidavits.

Instead   of   complying   with   the   subpoena,   petitioners   asked   for   the   suspension   of   the preliminary investigation, pending the resolution of their motion to quash the search warrant.  They argued, as they do now, that the illegally obtained firearms could not be the basis of the criminal Complaint.  Their  motion was  denied.  A   subsequent  Motion   for  Reconsideration  met   the  same 

48

Page 49: Search and Seizure Cases

fate.  In the present Petition for Certiorari and Prohibition, petitioners assert that “State Prosecutor Dacera cannot have any tenable basis for continuing with the proceedings in IS No. 95-167.”[41]

Because the search warrant was procured in violation of the Constitution and the Rules of Court, all the firearms, explosives and other materials seized were “inadmissible for any purpose in any   proceeding.”[42] As   the   Court   noted   in   an   earlier   case,   the   exclusion   of   unlawfully   seized evidence   was   “the   only   practical   means   of   enforcing   the   constitutional   injunction   against unreasonable  searches  and seizures.”[43] Verily,   they   are   the   “fruits   of   the   poisonous tree.”  Without this exclusionary rule, the constitutional right “would be so ephemeral and so neatly severed from its conceptual nexus with the freedom from all brutish means of coercing evidence xxx.”[44]

In the present case, the complaint for illegal possession of firearms is based on the firearms and other materials seized pursuant to Search Warrant No. 799 (95).  Since these illegally obtained pieces of evidence are inadmissible, the Complaint and the proceedings before State Prosecutor Dacera have no more leg to stand on.

This Court sympathizes with the police effort to stamp out criminality and to maintain peace and order in the country;  however, it reminds the law enforcement authorities that they must do so only upon strict  observance of the constitutional  and statutory rights of our people.  Indeed, “there is a right way to do the right thing at the right time for the right reason.”[45]

WHEREFORE, the instant petition for certiorari and prohibition is hereby GRANTED and Search Warrant No. 799 (95) accordingly declared NULL and VOID.  The temporary restraining order issued by this Court on October 23, 1995 is hereby MADE PERMANENT.  No pronouncement as to costs.

MAYOR BAI UNGGIE D. ABDULA and ODIN ABDULA, petitioners, vs. HON. JAPAL M. GUIANI, in his capacity as Presiding Judge, of Branch 14 of the Regional Trial Court of Cotabato City, respondent.

At bench is a petition for certiorari and prohibition to set aside the warrant of arrest issued by herein respondent Japal M. Guiani, then presiding judge of Branch 14 of the Regional Trial Court of Cotabato City, ordering the arrest of petitioners without bail in Criminal Case No. 2376 for murder.

The antecedent facts of the case are as follows:

On 24 June 1994, a complaint for murder, docketed as I.S. No. 94-1361, was filed before the Criminal Investigation Service Command, ARMM Regional Office XII against herein petitioners and six (6) other persons[1] in connection with the death of a certain Abdul Dimalen, the former COMELEC Registrar of Kabuntalan, Maguindanao.[2] The complaint alleged that herein petitioners paid the six other respondents the total amount of P200,000.00 for the death of Abdul Dimalen.[3]

Acting on this complaint, the Provincial Prosecutor of Maguindanao, Salick U. Panda, in a Resolution dated 22 August 1994[4], dismissed the charges of murder against herein petitioners and five other respondents on a finding that there was no prima facie case for murder against them. Prosecutor Panda, however, recommended the filing of an information for murder against one of the 

respondents, a certain Kasan Mama. Pursuant to this Resolution, an information for murder was thereafter filed against Kasan Mama before the sala of respondent Judge.

In an Order dated 13 September 1994[5], respondent Judge ordered that the case, now docketed as Criminal Case No. 2332, be returned to the Provincial Prosecutor for further investigation. In this Order, respondent judge noted that although there were eight (8) respondents in the murder case, the information filed with the court "charged only one (1) of the eight (8) respondents in the name of Kasan Mama without the necessary resolution required under Section 4, Rule 112 of the Revised Rules of Court to show how the investigating prosecutor arrived at such a conclusion." As such, the respondent judge reasons, the trial court cannot issue the warrant of arrest against Kasan Mama.

Upon the return of the records of the case to the Office of the Provincial Prosecutor for Maguindanao, it was assigned to 2nd Assistant Prosecutor Enok T. Dimaraw for further investigation. In addition to the evidence presented during the initial investigation of the murder charge, two new affidavits of witnesses were submitted to support the charge of murder against herein petitioners and the other respondents in the murder complaint. Thus, Prosecutor Dimaraw treated the same as a refiling of the murder charge and pursuant to law, issued subpoena to the respondents named therein.[6] On December 6, 1994, herein petitioners submitted and filed their joint counter-affidavits.

After evaluation of the evidence, Prosecutor Dimaraw, in a Resolution dated 28 December 1994,[7] found a prima facie case for murder against herein petitioners and three (3) other respondents.[8] He thus recommended the filing of charges against herein petitioners Bai Unggie Abdula and Odin Abdula, as principals by inducement, and against the three (3) others, as principals by direct participation.

Likewise in this 28 December 1994 Resolution, Provincial Prosecutor Salick U. Panda, who conducted the earlier preliminary investigation of the murder charge, added a notation stating that he was inhibiting himself from the case and authorizing the investigating prosecutor to dispose of the case without his approval. The reasons he cited were that the case was previously handled by him and that the victim was the father-in-law of his son.[9]

On 2 January 1995, an information for murder dated 28 December 1994[10] was filed against the petitioner spouses and Kasan Mama, Cuenco Usman and Jun Mama before Branch 14 of the Regional Trial Court of Cotabato City, then the sala of respondent judge. This information was signed by investigating prosecutor Enok T. Dimaraw. A notation was likewise made on the information by Provincial Prosecutor Panda, which explained the reason for his inhibition.[11]

The following day, or on 3 January 1995, the respondent judge issued a warrant[12] for the arrest of petitioners. Upon learning of the issuance of the said warrant, petitioners filed on 4 January 1995 an Urgent Ex-parte Motion[13] for the setting aside of the warrant of arrest on 4 January 1995. In this motion, petitioners argued that the enforcement of the warrant of arrest should be held in abeyance considering that the information was prematurely filed and that the petitioners intended to file a petition for review with the Department of Justice.

49

Page 50: Search and Seizure Cases

A petition for review[14] was filed by the petitioners with the Department of Justice on 11 January 1995.[15] Despite said filing, respondent judge did not act upon petitioner’s pending Motion to Set Aside the Warrant of Arrest.

Hence, this Petition for Certiorari and Prohibition wherein petitioners pray for the following:

"1. upon filing of this petition, a temporary restraining order be issued enjoining the implementation and execution of the order of arrest dated January 3, 1995 and enjoining the respondent judge from further proceeding with Criminal Case No. 2376 entitled People of the Philippines vs. Bai Unggie D. Abdula, et al. upon such bond as may be required by the Honorable Court;

2. this petition be given due course and the respondent be required to answer;

3. after due hearing, the order of arrest dated January 3, 1995 be set aside and declared void ab initio and the respondent judge be disqualified from hearing Criminal Case No. 2376 entitled People of the Philippines vs. Bai Unggie D. Abdula, et al."[16]

In a Resolution[17] dated 20 February 1995, this Court resolved to require respondent judge to submit a comment to the petition. The Court further resolved to issue a temporary restraining order[18] enjoining the respondent judge from implementing and executing the Order of Arrest dated 3 January 1995 and from further proceeding with Criminal Case No. 2376.

At the onset, it must be noted that petitions for certiorari and prohibition require that there be no appeal, nor any plain, speedy and adequate remedy in the ordinary course of law available to the petitioner.[19] In the instant case, it cannot be said that petitioners have no other remedy available to them as there is pending before the lower court an Urgent Motion[20] praying for the lifting and setting aside of the warrant of arrest. Ordinarily, we would have dismissed the petition on this ground and let the trial court decide the pending motion. However, due to the length of time that the issues raised in the petition have been pending before the courts, it behooves us to decide the case on the merits in order to preclude any further delay in the resolution of the case.

Respondent Japal M. Guiani retired from the judiciary on 16 April 1996. For this reason, respondent is no longer the presiding judge of the Regional Trial Court Branch 14 of Cotabato City; and the prayer of petitioner as to respondent’s disqualification from hearing Criminal Case No. 2376 is now moot and academic. Thus, there remain two issues left for the determination of the Court: first, the legality of the second information for murder filed before respondent’s court; and second, the validity of the warrant of arrest issued against petitioners.

With respect to the first issue, petitioners aver that it is the respondent judge himself who is orchestrating the filing of the alleged murder charge against them. In support, petitioners cite five (5) instances wherein respondent judge allegedly issued illegal orders in a mandamus case pending in respondent’s sala filed against petitioner Mayor Bai Unggie Abdula. These allegedly illegal orders formed the basis for a criminal complaint which they filed on 6 October 1994 against respondent 

and ten (10) others before the Office of the Ombudsman for Mindanao.[21] In this complaint, herein petitioners alleged that the respondent judge illegally ordered the release of the total amount of P1,119,125.00 from the municipal funds of Kabuntalan, Mindanao to a certain Bayoraiz Saripada, a purported niece of respondent judge. The Office of the Ombudsman for Mindanao, in an Order[22] dated 12 December 1994, found "sufficient basis to proceed with the preliminary investigation of the case" and directed the respondents therein to file their respective counter-affidavits and controverting evidence. From these facts, petitioners argue, it is clear that it is the respondent judge himself who is orchestrating and manipulating the charges against the petitioner.

Petitioners further state that respondent judge exhibited extreme hostility towards them after the filing of the said complaint before the Ombudsman. Petitioners claim that immediately after the issuance of the Order of the Ombudsman requiring respondent judge to file his counter-affidavit, respondent allegedly berated petitioner Bai Unggie Abdula in open court when she appeared before him in another case Allegedly, in full view of the lawyers and litigants, respondent judge uttered the following words in the Maguindanaoan dialect:

"If I cannot put you in jail within your term, I will cut my neck. As long as I am a judge here, what I want will be followed."[23]

Respondent judge, in compliance with the Order of this Court, filed a Comment dated 3 March 1995.[24] In this Comment, he argues that petitioners’ enumeration of "incontrovertible facts" is actually a list of misleading facts which they are attempting to weave into Criminal Case No. 2376 for the purpose of picturing respondent as a partial judge who abused his discretion to favor petitioner’s accuser.[25] He claims that the anti-graft charge filed by petitioners against him is a harassment suit concocted by them when they failed to lay their hands on the amount of P1,119,125.00 of municipal funds which respondent had previously ruled as rightfully belonging to the municipal councilors of Kabuntalan, Maguindanao. Respondent vehemently denies having personally profited from the release of the municipal funds. Moreover, respondent points out that the allegations in the complaint seem to imply that the Vice Mayor of Kabuntalan, Bayoraiz Sarupada, was a party to the mandamus case filed with respondent’s court when in truth, there was no case filed by the vice mayor pending in his court. Finally, respondent denies berating petitioner Bai Unggie Abdula and uttering the words attributed to him in the petition. According to respondent, the last time petitioner Bai Unggie Abdula appeared in his sala on December 28, 1994, in connection with the lifting of an order for her apprehension in another case, he neither berated nor scolded her and in fact, he even lifted the said order of arrest.

In its Comment with Urgent Motion for the Lifting of the Temporary Restraining Order dated 5 June 1995,[26] the Office of the Solicitor-General states that petitioner’s allegation that the respondent judge was biased and prejudiced was pure speculation as no proof was presented that respondent assumed the role of prosecutor. Moreover, the OSG argued that the fact that the respondent judge and petitioners had pending cases against each other did not necessarily result in the respondent’s bias and prejudice.

50

Page 51: Search and Seizure Cases

An analysis of these arguments shows that these should have been properly raised in a motion for the disqualification or inhibition of respondent judge. As previously stated however, the issue as to whether respondent should be disqualified from proceeding with the case has been rendered moot and academic as he is no longer hearing the case against petitioners. As such, there is no need for a prolonged discussion on this issue. It is sufficient to say that in order to disqualify a judge on the ground of bias and prejudice, petitioner must prove the same by clear and convincing evidence.[27] This is a heavy burden which petitioners have failed to discharge. This Court has to be shown acts or conduct of the judge clearly indicative of arbitrariness or prejudice before the latter can be branded the stigma of being biased and partial.[28]

Petitioners next argue that the act of respondent in motu proprio ordering a reinvestigation of the murder charge against them is another indication of the latter’s bias and prejudice.[29] They claim that the filing of their complaint against respondent motivated the latter’s Order of 13 September 1994 which ordered the return of the records of the murder case to the provincial prosecutor. Furthermore, they posit that the latter had no authority to order the reinvestigation considering that same had already been dismissed as against them by the provincial prosecutor in his Resolution dated 22 August 1994.

A review of the pertinent dates in the petition however show that respondent could not have been motivated by the Ombudsman’s complaint when he issued the 13 September 1994 Order. Petitioner Bai Unggie Abdula filed the complaint before the Ombudsman of Cotabato City on October 6, 1994[30] or about a month after the issuance of the 13 September 1994 Order. As such, when respondent issued the said Order, the same could not have been a retaliatory act considering that at that time, there was as yet no complaint against him.

With respect to the allegation that the respondent had no legal authority to order a reinvestigation of the criminal charge considering that the said charge had been previously dismissed as against them, we hold that respondent did not abuse his discretion in doing so.[31]

It is true that under the circumstances, the respondent judge, upon seeing that there were no records attached to the complaint, could have simply ordered the office of the provincial prosecutor to forward the same. Upon receipt of these records, respondent judge would then have sufficient basis to determine whether a warrant of arrest should issue. However, from the bare terms of the questioned order alone, we fail to see any illegal reason that could have motivated the judge in issuing the same. The order merely stated that the records of the case should be returned to the Office of the Provincial Prosecutor for further investigation or reinvestigation. He did not unduly interfere with the prosecutor’s duty to conduct a preliminary investigation by ordering the latter to file an information against any of the respondents or by choosing the fiscal who should conduct the reinvestigation which are acts certainly beyond the power of the court to do.[32] It was still the prosecutor who had the final say as to whom to include in the information.[33]

As pointed out by the Office of the Solicitor General, petitioners only imputed bias against the respondent judge and not against the investigating prosecutor.[34] Consequently, this imputation is of no moment as the discretion to file an information is under the exclusive control and supervision 

of the prosecutor and not of respondent judge. Furthermore, petitioners cannot claim that they were denied due process in the reinvestigation of the charges against them as they actively participated therein by submitting their joint counter-affidavit.

Petitioners likewise allege that the information charging petitioners with murder is null and void because it was filed without the authority of the Provincial Prosecutor. They note that in the Resolution dated 28 December 1994 and in the corresponding information, it clearly appears that the same were not approved by the Provincial Prosecutor as it was signed only by the investigating prosecutor, Anok T. Dimaraw.

Petitioners’ contention is not well-taken.

The pertinent portion of the Rules of Court on this matter state that "(n)o complaint or information shall be filed or dismissed by an investigating fiscal without the prior written authorityor approval of the provincial or city fiscal or chief state prosecutor (underscoring ours)." In other words, a complaint or information can only be filed if it is approved or authorized by the provincial or city fiscal or chief state prosecutor.

In the case at bench, while the Resolution and the Information were not approved by Provincial Prosecutor Salick U. Panda, the filing of the same even without his approval was authorized. Both the Resolution and information contain the following notation:*

"The herein Provincial Prosecutor is inhibiting himself from this case and Investigating Prosecutor Enok Dimaraw may dispose of the case without his approval on the following ground:

That this case has been previously handled by him, and whose findings differ from the findings of Investigating Prosecutor Dimaraw; and the victim is a relative by affinity, he being a father-in-law of his son.

(Signed) Salick U. Panda Provincial Prosecutor

It must be stressed that the Rules of Court speak of authority or approval by the provincial, city, or chief state prosecutor. The notation made by Prosecutor Panda clearly shows that Investigating Prosecutor Dimaraw was authorized to "dispose of the case without his approval." In issuing the resolution and in filing the information, the investigating prosecutor was acting well within the authority granted to him by the provincial prosecutor. Thus, this resolution is sufficient compliance with the aforecited provision of the Rules of Court.

Having thus ruled on the validity of the information filed against the respondents, we now address the issue as to the legality of the warrant of arrest issued by respondent judge by virtue of the said information.

51

Page 52: Search and Seizure Cases

On this issue, petitioners, citing the case of Allado vs. Diokno[35] argue that the warrant for his arrest should be recalled considering that the respondent judge "did not personally examine the evidence nor did he call the complainant and his witnesses in the face of their incredible accounts." As proof, he points to the fact that the information was filed at around 4:00 p.m. of the January 2, 1995 and the order of arrest was immediately issued the following day or on January 3, 1995. Moreover, petitioner argues, respondent judge did not even issue an order stating that there is probable cause for the issuance of the warrant of arrest, a clear violation of the guidelines set forth in the Allado case.

Respondent, in his Comment, denies any irregularity in the issuance of the warrant of arrest. He argues as follows:

"Written authority having been granted by the Provincial Prosecutor, as required by the third paragraph of Section 4, Rule 112 of (the) Rules on Criminal Procedure, and there having been no reason for the respondent to doubt the validity of the certification made by the Assistant Prosecutor that a preliminary investigation was conducted and that probable cause was found to exist as against those charged in the Information filed, and recognizing the prosecution’s legal authority to initiate and control criminal prosecution (Rule 110, Section 5) and considering that the court cannot interfere in said prosecution’s authority (People vs. Moll, 68 Phil. 626), the respondent issued the warrant for the arrest of the accused pursuant to paragraph (a), section 6, Rule 112;"[36]

The OSG, in defending the act of respondent judge, argues that the allegation that respondent did not personally examine the evidence is not supported by current jurisprudence. In support, the OSG invokes the pronouncement in Soliven vs. Makasiar[37] that "(I)n satisfying himself of the existence of probable cause, the judge is not required to personally examine the complainant and his witnesses." Moreover, the OSG points out that the judge enjoys a wide degree of latitude in the determination of probable cause for the issuance of warrants of arrest depending on the circumstances of each case.[38]

The OSG further argues that the case of Allado vs. Diokno, relied upon by petitioners, has no application in the case at bar considering that in the cited case, the documents submitted before the court failed to establish any probable cause as they were conflicting and contradictory. Significantly, the OSG continues, petitioners could not point out a single flaw in the evidence presented by the prosecutor to negate the existence of probable cause. Finally, the OSG points out that petitioner’s unfounded allegations cannot prevail over the well-settled rule that official duty is presumed to be regularly performed.[39]

After a careful analysis of these arguments, we find merit in the contention of petitioners.

The pertinent provision of the Constitution reads:

"Section 2 [Article III]. The right of the people to be secure in their persons, houses, papers, and effects against unreasonable searches and seizures of whatever nature and for any purpose shall be inviolable, and no search warrant or warrant of arrest shall issue except upon probable cause to be 

determined personally by the judge after examination under oath or affirmation of the complainant and the witnesses he may produce      and particularly describing the place to be searched and the persons or things to be seized." (Undersoring supplied.)

It must be stressed that the 1987 Constitution requires the judge to determine probable cause "personally," a requirement which does not appear in the corresponding provisions of our previous constitutions. This emphasis evinces the intent of the framers to place a greater degree of responsibility upon trial judges than that imposed under previous Constitutions.[40]

In Soliven vs. Makasiar, this Court pronounced:

"What the Constitution underscores is the exclusive and personal responsibility of the issuing judge to satisfy himself of the existence of probable cause. In satisfying himself of the existence of probable cause for the issuance of a warrant of arrest, the judge is not required to personally examine the complainant and his witnesses. Following established doctrine and procedure, he shall: (1) personally evaluate the report and the supporting documents submitted by the fiscal regarding the existence of probable cause and, on the basis thereof, issue a warrant of arrest; or (2) if on the basis thereof he finds no probable cause, he may disregard the fiscal’s report and require the submission of supporting affidavits of witnesses to aid him in arriving at a conclusion as to the existence of probable cause."

Ho vs. People[41] summarizes existing jurisprudence on the matter as follows:

"Lest we be too repetitive, we only wish to emphasize three vital matters once more: First, as held in Inting, the determination of probable cause by the prosecutor is for a purpose different from that which is to be made by the judge. Whether there is reasonable ground to believe that the accused is guilty of the offense charged and should be held for trial is what the prosecutor passes upon. The judge, on the other hand, determines whether a warrant of arrest should be issued against the accused,i.e., whether there is a necessity for placing him under immediate custody in order not to frustrate the ends of justice. Thus, even if both should base their findings on one and the same proceeding or evidence, there should be no confusion as to their distinct objectives.

Second, since their objectives are different, the judge cannot rely solely on the report of the prosecutor in finding probable cause to justify the issuance of a warrant of arrest. Obviously and understandably, the contents of the prosecutor’s report will support his own conclusion that there is reason to charge the accused for an offense and hold him for trial. However, the judge must decide independently. Hence, he must have supporting evidence, other than the prosecutor’s bare report, upon which to legally sustain his own findings on the existence (or nonexistence) of probable cause to issue an arrest order. This responsibility of determining personally and independently the existence or nonexistence of probable cause is lodged in him by no less than the most basic law of the land. Parenthetically, the prosecutor could ease the burden of the judge and speed up the litigation process by forwarding to the latter not only the information and his bare resolution finding probable cause, but also so much of the records and the evidence on 

52

Page 53: Search and Seizure Cases

hand as to enable the His Honor to make his personal and separate judicial finding on whether to issue a warrant of arrest.

Lastly, it is not required that the complete or entire records of the case during the preliminary investigation be submitted to and examined by the judge. We do not intend to unduly burden trial courts by obliging them to examine the complete records of every case all the time simply for the purpose of ordering the arrest of an accused. What is required, rather, is that the judge must have sufficient supporting documents (such as the complaint, affidavits, counter-affidavits, sworn statements of witnesses or transcript of stenographic notes, if any) upon which to make his independent judgment or, at the very least, upon which to verify the findings of the prosecutor as to the existence of probable cause. The point is: he cannot rely solely and entirely on the prosecutor’s recommendation, as Respondent Court did in this case. Although the prosecutor enjoys the legal presumption of regularity in the performance of his official duties and functions, which in turn gives his report the presumption of accuracy, the Constitution, we repeat, commands the judge to personally determine probable cause in the issuance of warrants of arrest. This Court has consistently held that a judge fails in his bounden duty if he relies merely on the certification or the report of the investigating officer." (citations omitted)

In the case at bench, respondent admits that he issued the questioned warrant as there was "no reason for (him) to doubt the validity of the certification made by the Assistant Prosecutor that a preliminary investigation was conducted and that probable cause was found to exist as against those charged in the information filed." The statement is an admission that respondent relied solely and completely on the certification made by the fiscal that probable cause exists as against those charged in the information and issued the challenged warrant of arrest on the sole basis of the prosecutor’s findings and recommendations. He adopted the judgment of the prosecutor regarding the existence of probable cause as his own.

Although the prosecutor enjoys the legal presumption of regularity in the performance of his official duties, which in turn gives his report the presumption of accuracy, nothing less than the fundamental law of the land commands the judge to personally determine probable cause in the issuance of warrants of arrest. A judge fails in this constitutionally mandated duty if he relies merely on the certification or report of the investigating officer.

To be sure, we cannot determine beforehand how cursory or exhaustive the respondent’s examination of the records should be.[42] The extent of the judge’s examination depends on the exercise of his sound discretion as the circumstances of the case require. In the case at bench, the respondent had before him two different informations and resolutions charging two different sets of suspects. In the face of these conflicting resolutions, it behooves him not to take the certification of the investigating prosecutor at face value. The circumstances thus require that respondent look beyond the bare certification of the investigating prosecutor and examine the documents supporting the prosecutor’s determination of probable cause. The inordinate haste that attended the issuance of the warrant of arrest and respondent’s own admission are circumstances that tend to belie any pretense of the fulfillment of this duty.

Clearly, respondent judge, by merely stating that he had no reason to doubt the validity of the certification made by the investigating prosecutor has abdicated his duty under the Constitution to determine on his own the issue of probable cause before issuing a warrant of arrest. Consequently, the warrant of arrest should be declared null and void.

WHEREFORE, premises considered, the petition for certiorari and prohibition is GRANTED. The temporary restraining order we issued on 20 February 1995 in favor of petitioners insofar as it enjoins the implementation and execution of the order of arrest dated 3 January 1995 is made permanent. Criminal Case No. 2376 is REMANDED to Branch 14 of the Regional Trial Court of Cotabato City for a proper determination of whether a warrant of arrest should be issued and for further proceedings.

DIOSDADO JOSE ALLADO and ROBERTO L. MENDOZA, petitioners, vs.HON. ROBERTO C. DIOKNO, Presiding Judge, Br. 62, Regional Trial Court, Makati, Metro Manila, and PRESIDENTIAL ANTI-CRIME COMMISSION, respondents.

On balance at the fulcrum once again are the intrinsic right of the State to prosecute perceived transgressors of the law, which can be regulated, and the innate value of human liberty, which can hardly be weighed.

Some twelve years ago we were confronted with a similar problem when former Senator Jovito R. Salonga invoked before this Court his "right to life and liberty guaranteed by the due process clause, alleging that no prima faciecase has been established to warrant the filing of an information for subversion against him." 1 We resolved the issue then and sustained him. He is now back before us, this time as counsel pleading the cause of petitioners herein who, he claims, are in a situation far worse than his predicament twelve (12) years ago. He postulates that no probable cause likewise exists in this case, and what is worse is that no bail is recommended.

This petition gives us an opportunity to revisit the concept and implication of probable cause, the existence of which is necessary for the prosecutor to have an accused held for trial and for a trial judge to issue a warrant for his arrest. It is mandatory therefore that there be probable cause before an information is filed and a warrant of arrest issued. Unfortunately, however, at times a criminal case is filed, a warrant of arrest issued and a person consequently incarcerated on unsubstantiated allegations that only feign probable cause.

Petitioners Diosdado Jose Allado and Roberto L. Mendoza, alumni of the College of Law, University of the Philippines, are partners of the Law Firm of Salonga, Hernandez and Allado. In the practice of their profession, and on the basis of an alleged extrajudicial confession of a security guard, they have been accused of the heinous crime of kidnapping with murder by the Presidential Anti-Crime Commission (PACC) and ordered arrested without bail by respondent judge.

The focal source of the information against petitioners is the sworn statement dated 16 September 1993 of Security Guard Escolastico Umbal, a discharge of the Philippine Constabulary, implicating them as the brains behind the alleged kidnapping and slaying of one Eugen Alexander Van Twest, a 

53

Page 54: Search and Seizure Cases

German national. 2 In that extrajudicial confession, Umbal claimed that he and his companions were met by petitioners at Silahis Hotel and in exchange for P2.5M the former undertook to apprehend Van Twest who allegedly had an international warrant of arrest against him. Thus, on 16 June 1992, after placing him under surveillance for nearly a month, Umbal, Ex-policeman Rolando Gamatero, AFPCIG Agent Roberto Santiago and SPO2 Sergio Antonino abducted Van Twest. They blocked his blue Nissan Pathfinder under the Alabang overpass and forced him into their car. They brought him to a "safe house" just behind the New Bilibid Prisons. Umbal was tasked to watch over their quarry. After four (4) days, Gamatero, Santiago and Antonino returned to the "safe house" together with petitioners and SPO2 Roger Bato, known to Umbal also as "Batok." SPO2 Bato faked the interrogation of Van Twest, pretending it was official, and then made him sign certain documents. The following day, Gamatero shot Van Twest in the chest with a baby armalite, after which Antonino stabbed him repeatedly, cut off his private part, and later burned his cadaver into fine ashes using gasoline and rubber tires. Umbal could not recall the exact date when the incident happened, but he was certain it was about a year ago.

A day after Umbal executed his extrajudicial confession, the operatives of the PACC, armed with a search warrant issued by Judge Roberto A. Barrios of the Regional Trial Court of Manila, Br. 11, 3 separately raided the two (2) dwellings of Santiago, one located at No. 7 Sangley Street, and the other, along Amalingan Street, both in Green Heights Subdivision, Parañaque. The raiders recovered a blue Nissan Pathfinder and assorted firearms and ammunition and placed Santiago and his trusted aide, Efren Madolid, under arrest. Also arrested later that day were Antonio and Bato who were found to have in their possession several firearms and ammunition and Van Twest's Cartier sunglasses.

After evaluating the pieces of evidence gathered by PACC operatives, Sr., Supt. Panfilo Lacson, Chief of PACC Task Force Habagat, referred the case to the Department of Justice for the institution of criminal proceedings against AFPCIG Agent Roberto Santiago, SPO1 Sergio Antonino, SPO2 Roger Bato, Ex-policeman Rolando Gamatero, Efren Madolid, and petitioners herein, Atty. Diosdado Jose Allado and Atty. Roberto L. Mendoza, for illegal possession of firearms and ammunition, carnapping, kidnapping for ransom with murder, and usurpation of authority. 4 In his letter to the State Prosecutor dated 17 September 1993, Sr. Supt. Lacson charged that —

Atty. Roberto L. Mendoza and Atty. Allado of Salonga, Hernandez and Allado Law Offices . . . planned and conspired with other suspects to abduct and kill the German national Alexander Van Twest in order to eliminate him after forcing the victim to sign several documents transferring ownership of several properties amounting to several million pesos and caused the withdrawal of P5M deposit from the victim's bank account.

Thereafter, Senior State Prosecutor Ferdinand prosecutor Ferdinand R. Abesamis issued a subpoena to petitioners informing them that a complaintwas filed against them by PACC TF-Habagat, directing them to appear on30 September 1993 at the Multi-Purpose Hall of the Department of Justice and to submit their counter-affidavits. Attached to the subpoena were copies of the affidavits executed by Umbal and members of the team who raided the two (2) dwellings of Santiago. 5

Not satisfied merely with the affidavits attached to the subpoena, petitioner Mendoza moved for the production of other documents for examination and copying to enable him to fully prepare for his defense and to submit an intelligible counter-affidavit. 6 Specifically, petitioner Mendoza was interested in (a) the "several documents transferring ownership of several properties amounting to several million pesos and the withdrawal of P5M deposits from the victim's bank account," as stated in the complaint; (b) the complete records of the PACC's investigation, including investigations on other suspects and their disposition, PACC's Order of Battle for 1992 and early 1993; and, (c) such other written statements issued in the above-entitled case, and all other documents intended to be used in this case. 7 Petitioners likewise sought the inhibition of the members of the panel of prosecutors, which was created to conduct the preliminary investigation, on the ground that they were members of the legal staff assigned to PACC and thus could not act with impartiality.

In its Order of 11 October 1993, 8 the new panel of prosecutors composed of Senior State Prosecutor Bernelito R. Fernandez as Chairman, with Rogelio F. Vista and Purita M. Deynata as Members, confirmed that the motion for inhibition of the members of the old panel as well as the appeal to the Secretary of Justice was resolved on 8 October 1993 resulting in the creation of a new panel. Thereafter, the new panel granted the prayer of petitioner Mendoza for the production of additional documents used or intended to be used against him. Meanwhile, Task Force Habagat, in compliance with the order, submitted only copies of the request for verification of the firearms seized from the accused, the result of the request for verification, and a Philippine Times Journal article on the case with a marginal note of President Fidel V. Ramos addressed to the Chief of the Philippine National Police directing the submission of a report and summary of actions taken thereon.

Not having been provided with the requested documents, petitioners nevertheless submitted their respective counter-affidavits denying the accusations against them. 9

After a preliminary hearing where clarificatory questions were additionally propounded, the case was deemed submitted for resolution. But before the new panel could resolve the case, SPO2 Bato filed a manifestation stating that he was reconsidering the earlier waiver of his right to file counter- affidavit, 10 and "in the greater interest of truth, justice and fair play" moved for the admissions of his counter-affidavit 11 confessing participation in the abduction and slaying of Van Twest and implicating petitioners Allado and Mendoza. Sometime in January 1994, however, before petitioners could refute Bato's counter-affidavit, he moved to suppress it on the ground that it was extracted through intimidation and duress.

On 3 February 1994, with the new penal failing to act on the twin motions of SPO2 Bato, petitioners heard over the radio that the panel had issued a resolution finding a prima facie case against them and that an information had already been filed in court. Upon verification with the Department of Justice, however, petitioners were informed that the resolution was not yet ready for release, but later that afternoon they were able to secure a copy of the information for kidnapping with murder against them 12 and the 15-page undated resolution under the letterhead of PACC, signed by the panel of prosecutors, with the Head of the PACC Task Force recommending approval thereof. 13 That same day, the information was filed before the Regional Trial Court of Makati and raffled off to Branch 62 presided by respondent Judge Roberto C. Diokno.

54

Page 55: Search and Seizure Cases

On 4 February 1994, respondent judge, in response to petitioners' request, gave them until 8 February 1994 to submit their opposition to the issuance of a warrant of arrest against all the accused. 14 On 7 February 1994, petitioners complied with the order of respondent judge. 15 The following day,8 February 1994, petitioner Allado filed an appeal with the Secretary of Justice seeking review and reversal of the undated resolution of the panelof prosecutors, 16 which appeal was adopted by petitioner Mendoza. 17 On11 February 1994, petitioner Allado moved to defer the proceedings before the trial court pending resolution of his appeal before the Secretary of Justice. 18 However, on even date, respondent judge issued the assailed warrant of arrest against petitioners. 19 Hence, on 15 February 1994, petitioners filed with us the instant petition forcertiorari and prohibition with prayer for a temporary restraining order.

On 16 February 1994, we required respondents to comment on the petition and set the case for hearing on 28 February 1994. After the hearing, we issued a temporary restraining order enjoining PACC from enforcing the warrant of arrest and respondent judge from conducting further proceedings on the case and, instead, to elevate the records to us. Meanwhile, on 27 February 1994, petitioners voluntarily surrendered at the Headquarters of the Capital Command (CAPCOM), Philippine National Police (PNP), Camp Bagong Diwa, Bicutan, Metro Manila, and on 29 February 1994, they were released on the basis of our temporary restraining order.

Petitioners, in their 335-page petition, inclusive of annexes, principally contend that respondent judge acted with grave abuse of discretion and in excess of jurisdiction in "whimsically holding that there is probable cause against petitioners without determining the admissibility of the evidence against petitioners and without even stating the basis of his findings," 20 and in "relying on the Resolution of the Panel and their certification that probable cause exists when the certification is flawed." 21 Petitioners maintain that the records of the preliminary investigation which respondent judge solely relied upon failed to establish probable cause against them to justify the issuance of the warrant of arrest. Petitioners likewise assail the prosecutors' "clear sign of bias and impartiality (sic)." 22

On the other hand, the Office of the Solicitor General argues that the determination of probable cause is a function of the judge who is merely required to personally appreciate certain facts to convince him that the accused probably committed the crime charged.

Section 2, Art. III, of the 1987 Constitution, lays down the requirements for the issuance of a warrant of arrest, i.e., a warrant of arrest shall issue only upon probable cause to be determined personally by the judge after examination under oath or affirmation of the complainant and the witnesses he may produce.

As early as 1915, in Buchanan v. Viuda de Esteban, 23 this Court speaking through Associate Justice Sherman Moreland defined probable cause as "the existence of such facts and circumstances as would excite the belief, in a reasonable mind, acting on the facts within the knowledge of the prosecutor, that the person charged was guilty of the crime for which he was prosecuted." This 

definition is still relevant today as we continue to cite it in recent cases. 24 Hence, probable cause for an arrest or for the issuance of a warrant of arrest has been defined as such facts and circumstances which would lead a reasonable discreet and prudent man to believe that an offense has been committed by the person sought to be arrested. 25 And as a protection against false prosecution and arrest, it is the knowledge of facts, actual or apparent, strong enough to justify a reasonable man in the belief that he was lawful grounds for arresting the accused. 26

Pilapil v. Sandiganbayan 27 sets a standard for determining the existence of probable cause. While it appears in that case that we have granted the prosecutor and the trial judge seemingly unlimited latitude in determining the existence of absence of probable cause by affirming the long-standing procedure that they can base their findings merely on their personal opinion and reasonable belief, yet, this permissiveness should not be interpreted as giving them arbitrary powers and letting them loose in the determination of the existence of probable cause, a delicate legal question which can result in the harassment and deprivation of liberty of the person sought to be charged or arrested. There we said —

Probable cause is a reasonable ground of presumption that a matter is, or may be, well founded, such a state of facts in the mind of the prosecutor as would lead a person of ordinary caution and prudence to believe, or entertain an honest or strong suspicion, that a thing is so. The term does not mean "actual and positive cause" nor does it import absolute certainty. It is merely based on opinion and reasonable belief. Thus, a finding of probable cause does not require an inquiry into whether there is sufficient evidence to procure a conviction. It is enough that it is it believed that the act or omission complained of constitutes the offense charged. Precisely, there is a trial for the reception of evidence of the prosecution in support of the charge.

Whether an act was done causing undue injury to the government and whether the same was done with manifest partiality or evident bad faith can only be made out by proper and sufficient testimony. Necessarily, a conclusion can be arrived at when the case has already proceeded on sufficient proof. 28

Accordingly, before issuing a warrant of arrest, the judge must satisfy himself that based on the evidence submitted there is sufficient proof that a crime has been committed and that the person to be arrested is probably guilty thereof. In the Order of respondent judge dated 11 February 1994, it is expressly stated that "[t]his court after careful evaluation of the evidence on record, believes and rules that probable cause exists; and therefore, a warrant of arrest should be issued." However, we are unable to see how respondent judge arrived at such ruling. We have painstakingly examined the records and we cannot find any support for his conclusion. On the contrary, we discern a number of reasons why we consider the evidence submitted to be insufficient for a finding of probable cause against petitioners.

The Presidential Anti-Crime Commission relies heavily on the sworn statement of Security Guard Umbal who supposedly confessed his participation in the alleged kidnapping and murder of Van Twest. For one, there is serious doubt on Van Twest's reported death since the corpus delicti has not been established, nor have his remains been recovered. Umbal claims that Van Twest was 

55

Page 56: Search and Seizure Cases

completely burned into ashes with the use of gasoline and rubber tires from around ten o'clock in the evening to six o'clock the next morning. 29 This is highly improbable, if not ridiculous. A human body cannot be pulverized into ashes by simply burning it with the use of gasoline and rubber tires in an open field. Even crematoria use entirely closed incinerators where the corpse is subjected to intense heat. 30 Thereafter, the remains undergo a process where the bones are completely ground to dust.

In the case of Van Twest, there is not even any insinuation that earnest efforts were exerted to recover traces of his remains from the scene of the alleged cremation. 31 Could it be that the government investigators did to the place of cremation but could not find any? Or could it be that they did not go at all because they knew that there would not be any as no burning ever took place? To allege then that the body of Van Twest was completely burned to ashes in an open field with the use merely of tires and gasoline is a tale too tall to gulp.

Strangely, if not awkwardly, after Van Twest's reported abduction on16 June 1992 which culminated in his decimation by cremation, his counsel continued to represent him before judicial and quasi-judicial proceedings. Thus on 31 July 1992, his counsel filed in his behalf a petition for review before this Court, docketed as G.R. Nos. 106253, and on 18 March 1993, a memorandum before the Securities and Exchange Commission in SEC Case No. 3896. On26 November 1993, during the preliminary investigation conducted by the panel of prosecutors, counsel again manifested that "even then and even as of this time, I stated in my counter-affidavit that until the matter of death is to be established in the proper proceedings, I shall continue to pursue my duties and responsibilities as counsel for Mr. Van Twest." 32 Hence, even Asst. Solicitor General Estoesta believes that counsel of Van Twest doubted the latter'sdeath. 33 Obviously, counsel himself does not believe that his client is in fact already dead otherwise his obligation to his client would have ceased except to comply with his duty "to inform the court promptly of such death . . . and to give the name and residence of his executor, administrator, guardian or other legal representative," 34 which he did not.

Under the circumstances, we cannot discount petitioners' theory that the supposed death of Van Twest who is reportedly an international fugitive from justice, a fact substantiated by petitioners and never refuted by PACC, is a likely story to stop the international manhunt for his arrest. In this regard, we are reminded of the leading case ofU.S. v. Samarin 35 decided ninety-two years ago where this Court ruled that when the supposed victim is wholly unknown, his body not found, and there is but one witness who testifies to the killing, the corpus delicti is not sufficiently proved.

Then, the extrajudicial statement of Umbal suffers from material inconsistencies. In his sworn statement, he said that he together with his cohorts was met by petitioners in Silahis Hotel where they hatched the plan to abduct Van Twest. 36 However, during the preliminary investigation, he stated that he was not part of the actual meeting as he only waited outside in the car for his companions who supposedly discussed the plan inside Silahis Hotel. 37

Umbal also said that petitioners arrived with Bato and conducted a mock interrogation of Van Twest who thereafter signed various documents upon being compelled to do so. 38 During the clarificatory 

questioning, however, Umbal changed his story and said that he was asked to go outside of the "safe house" at the time Van Twest was interrogated and thus did not see if Van Twest indeed signed certain documents. Why Umbal had to be sent out of the "safe house,"no explanation was offered. Did these documents really exist? Or could thenon-existence of these documents be the reason why PACC was not able to comply with the order of the prosecutors to produce them during the preliminary investigation? And then, what happened to the P2.5M that was supposedly offered by petitioners in exchange for the abduction of Van Twest? These and more remain unanswered.

Most perplexing however is that while the whole investigation was supposedly triggered off by Umbal's confession of 16 September 1993, the application of the PACC operatives for a search warrant to be served in thetwo (2) dwellings of Santiago was filed and granted by the Regional Trial Court of Manila on 15 September 1993, a day before Umbal executed his sworn statement. In support of the application, the PACC agents claimed that Umbal had been in their custody since 10 September 1993. Significantly, although he was said to be already under their custody, Umbal claims he was never interrogated until 16 September 1993 and only at the security barracks of Valle Verde V, Pasig, where he was a security guard. 39

The alleged counter-affidavit of SPO2 Bato, which the panel of prosecutors also considered in filing the charges against petitioners, can hardly be credited as its probative value has tremendously waned. The records show that the alleged counter-affidavit, which is self-incriminating, was filed after the panel had considered the case submitted for resolution. And before petitioners could refute this counter-affidavit, Bato moved to suppress the same on the ground that it was extracted through duress and intimidation.

For sure, the credibility of Umbal is badly battered. Certainly, his bare allegations, even if the State invokes its inherent right to prosecute, are insufficient to justify sending two lawyers to jail, or anybody for that matter. More importantly, the PACC operatives who applied for a warrant to search the dwellings of Santiago never implicated petitioners. In fact they claimed that according to Umbal, it was Santiago, and not petitioners, who masterminded the whole affair. 40 While there may be bits of evidence against petitioners'co-accused, i.e., referring to those seized from the dwellings of Santiago, these do not in the least prove petitioners' complicity in the crime charged. Based on the evidence thus far submitted there is nothing indeed, much less is there probable cause, to incriminate petitioners. For them to stand trial and be deprived in the meantime of their liberty, however brief, the law appropriately exacts much more to sustain a warrant for their arrest — facts and circumstances strong enough in themselves to support the belief that they are guilty of a crime that in fact happened. Quite obviously, this has not been met.

Verily, respondent judge committed grave abuse of discretion in issuing the warrant for the arrest of petitioners it appearing that he did not personally examine the evidence nor did he call for the complainant and his witnesses in the face of their incredible accounts. Instead, he merely relied on the certification of the prosecutors that probable cause existed. For, otherwise, he would have found out that the evidence thus far presented was utterly insufficient to warrant the arrest of 

56

Page 57: Search and Seizure Cases

petitioners. In this regard, we restate the procedure we outlined in various cases we have already decided.

In Soliven v. Makasiar, 41 we said that the judge (a) shall personally evaluate the report and the supporting documents submitted by the fiscal regarding the existence of probable cause and, on the basis thereof, issue a warrant of arrest; or, (b) if on the basis thereof he finds no probable cause, may disregard the fiscal's report and require the submission of supporting affidavits of witnesses to aid him in arriving at a conclusion on the existence of probable cause.

In People v. Inting, 42 we emphasized the important features of the constitutional mandate: (a) The determination of probable cause is a function of the judge; it is not for the provincial fiscal or prosecutor to ascertain. Only the judge and the judge alone makes this determination; (b) The preliminary inquiry made by a prosecutor does not bind the judge. It merely assists him in making the determination of probable cause. The judge does not have to follow what the prosecutor presents to him. By itself, the prosecutor's certification of probable cause is ineffectual. It is the report, the affidavits, the transcript of stenographic notes (if any), and all other supporting documents behind the prosecutor's certification which are material in assisting the judge in his determination of probable cause; and, (c) Judges and prosecutors alike should distinguish the preliminary inquiry which determines probable cause for the issuance of a warrant of arrest from the preliminary investigation proper which ascertains whether the offender should be held for trial or released. Even if the two inquiries be conducted in the course of one and the same proceeding, there should be no confusion about their objectives. The determination of probable cause for the warrant is made by the judge. The preliminary investigationproper — whether or not there is reasonable ground to believe that the accused is guilty of the offense charged and therefore, whether or not he should be subjected to the expense, rigors and embarrassment of trial — is a function of the prosecutor.

In Lim v. Felix, 43 where we reiterated Soliven v. Makasiar and People v. Inting, we said —

[T]he Judge does not have to personally examine the complainant and his witnesses. The Prosecutor can perform the same functions as a commissioner for the taking of the evidence. However, there should be a report and necessary documents supporting the Fiscal's bare certification. All these should be before the Judge.

The extent of the Judge's personal examination of the report and its annexes depends on the circumstances of each case. We cannot determine beforehand how cursory or exhaustive the Judge's examination should be. The Judge has to exercise sound discretion for, after all, the personal determination is vested in the Judge by the Constitution. It can be as brief or as detailed as the circumstances of each case require. To be sure, the judge must go beyond the Prosecutor's certification and investigation report whenever necessary. He should call for the complainant and witnesses themselves to answer the court's probing questions when the circumstances of the case so require.

Clearly, probable cause may not be established simply by showing that a trial judge subjectively believes that he has good grounds for his action. Good faith is not enough. If subjective good faith alone were the test, the constitutional protection would be demeaned and the people would be "secure in their persons, houses, papers and effects" only in the fallible discretion of the judge. 44 On the contrary, the probable cause test is an objective one, for in order that there be probable cause the facts and circumstances must be such as would warrant a belief by a reasonably discreet and prudent man that the accused is guilty of the crime which has just been committed.45 This, as we said, is the standard. Hence, if upon the filing of the information in court the trial judge, after reviewing the information and the documents attached thereto, finds that no probable cause exists must either call for the complainant and the witnesses themselves or simply dismiss the case. There is no reason to hold the accused for trial and further expose him to an open and public accusation of the crime when no probable cause exists.

But then, it appears in the instant case that the prosecutors have similarly misappropriated, if not abused, their discretion. If they really believed that petitioners were probably guilty, they should have armed themselves with facts and circumstances in support of that belief; for mere belief is not enough. They should have presented sufficient and credible evidence to demonstrate the existence of probable cause. For the prosecuting officer "is the representative not of an ordinary party to a controversy, but of a sovereignty whose obligation to govern impartially is as compelling as its obligation to govern all; and whose interest, therefore, in a criminal prosecution is not that it shall win a case, but that justice shall be done. As such, he is in a peculiar and very definite sense the servant of the law, the twofold aim of which is that guilt shall not escape or innocence suffer. He may prosecute with earnestness and vigor — indeed, he should do so. But, while he may strike hard blows, he is not at liberty to strike foul ones. It is as much his duty to refrain from improper methods calculated to produce a wrongful conviction as it is to use every legitimate means to bring about a just one" 46

In the case at bench, the undue haste in the filing of the information and the inordinate interest of the government cannot be ignored. From the gathering of evidence until the termination of the preliminary investigation, it appears that the state prosecutors were overly eager to file the case and secure a warrant for the arrest of the accused without bail and their consequent detention. Umbal's sworn statement is laden with inconsistencies and improbabilities. Bato's counter-affidavit was considered without giving petitioners the opportunity to refute the same. The PACC which gathered the evidence appears to have had a hand in the determination of probable cause in the preliminary inquiry as the undated resolution of the panel not only bears the letterhead of PACC but was also recommended for approval by the head of the PACC Task Force. Then petitioners were given the runaround in securing a copy of the resolution and the information against them.

Indeed, the task of ridding society of criminals and misfits and sending them to jail in the hope that they will in the future reform and be productive members of the community rests both on the judiciousness of judges and the prudence of prosecutors. And, whether it is a preliminary investigation by the prosecutor, which ascertains if the respondent should be held for trial, or a preliminary inquiry by the trial judge which determines if an arrest warrant should issue, the bottomline is that there is a standard in the determination of the existence of probable cause, i.e., there should be facts and circumstances sufficiently strong in themselves to warrant a prudent and 

57

Page 58: Search and Seizure Cases

cautious man to believe that the accused is guilty of the crime with which he is charged. Judges and prosecutors are not off on a frolic of their own, but rather engaged in a delicate legal duty defined by law and jurisprudence.

In this instance, Salonga v. Paño 47 finds application —

The purpose of a preliminary investigation is to secure the innocent against hasty, malicious and oppressive prosecution, and to protect him from an open and public accusation of crime, from the trouble, expense and anxiety of a public trial, and also to protect the state from useless and expensive trial (Trocio v. Manta, 118 SCRA 241, citing Hashim v. Boncan, 71 Phil. 216). The right to a preliminary investigation is a statutory grant, and to withhold it would be to transgress constitutional due process (People v. Oandasa, 25 SCRA 277). However, in order to satisfy the due process clause it is not enough that the preliminary investigation is conducted in the sense of making sure that the transgressor shall not escape with impunity. A preliminary investigation serves not only for the purposes of the State. More importantly, it is a part of the guarantees of freedom and fair play which are birthrights of all who live in the country. It is therefore imperative upon the fiscal or the judge as the case may be, to relieve the accused from the pain of going thru a trial once it is ascertained that the evidence is insufficient to sustain a prima facie case or that no probable cause exists to form a sufficient belief as to the guilt of the accused (emphasis supplied).

The facts of this case are fatefully distressing as they showcase the seeming immensity of government power which when unchecked becomes tyrannical and oppressive. Hence the Constitution, particularly the Bill of Rights, defines the limits beyond which lie unsanctioned state actions. But on occasion, for one reason or another, the State transcends this parameter. In consequence, individual liberty unnecessarily suffers. The case before us, if uncurbed, can be illustrative of a dismal trend. Needless injury of the sort inflicted by government agents is not reflective of responsible government. Judges and law enforcers are not, by reason of their high and prestigious office, relieved of the common obligation to avoid deliberately inflicting unnecessary injury.

The sovereign power has the inherent right to protect itself and its people from vicious acts which endanger the proper administration of justice; hence, the State has every right to prosecute and punish violators of the law. This is essential for its self- preservation, nay, its very existence. But this does not confer a license for pointless assaults on its citizens. The right of the State to prosecute is not a carte blanche for government agents to defy and disregard the rights of its citizens under the Constitution. Confinement, regardless of duration, is too high a price to pay for reckless and impulsive prosecution. Hence, even if we apply in this case the "multifactor balancing test" which requires the officer to weigh the manner and intensity of the interference on the right of the people, the gravity of the crime committed and the circumstances attending the incident, still we cannot see probable cause to order the detention of petitioners. 48

The purpose of the Bill of Rights is to protect the people against arbitrary and discriminatory use of political power. This bundle of rights guarantees the preservation of our natural rights which include personal liberty and security against invasion by the government or any of its branches or 

instrumentalities. Certainly, in the hierarchy of rights, the Bill of Rights takes precedence over the right of the State to prosecute, and when weighed against each other, the scales of justice tilt towards the former. Thus, relief may be availed of to stop the purported enforcement of criminal law where it is necessary to provide for an orderly administration of justice, to prevent the use of the strong arm of the law in an oppressive and vindictive manner, and to afford adequate protection to constitutional rights. 49

Perhaps, this case would not have reached this Court if petitioners were ordinary people submissive to the dictates of government. They would have been illegally arrested and detained without bail. Then we would not have the opportunity to rectify the injustice. Fortunately, the victims of injustice are lawyers who are vigilant of their rights, who fight for their liberty and freedom not otherwise available to those who cower in fear and subjection.

Let this then be a constant reminder to judges, prosecutors and other government agents tasked with the enforcement of the law that in the performance of their duties they must act with circumspection, lest their thoughtless ways, methods and practices cause a disservice to their office and maim their countrymen they are sworn to serve and protect. We thus caution government agents, particularly the law enforcers, to be more prudent in the prosecution of cases and not to be oblivious of human rights protected by the fundamental law. While we greatly applaud their determined efforts to weed society of felons, let not their impetuous eagerness violate constitutional precepts which circumscribe the structure of a civilized community.

WHEREFORE, the petition for certiorari and prohibition is GRANTED. The temporary restraining order we issued on 28 February 1994 in favor of petitioners, Atty. Diosdado Jose Allado and Atty. Roberto L. Mendoza, is made permanent. The warrant of arrest issued against them is SET ASIDE and respondent Judge Roberto C. Diokno is ENJOINED from proceeding any further against herein petitioners in Crim. Case No. 94-1757 of the Regional Trial Court of Makati.

CONGRESSMAN FRANCISCO B. ANIAG, JR., petitioner, vs.COMMISSION ON ELECTIONS and DEPARTMENT OF JUSTICE SPECIAL TASK FORCE, respondents.

PETITIONER assails in this petition (for declaratory relief, certiorari and prohibition) the following resolutions of the Commission on Elections: Resolution No. 2327 dated 26 December 1991 for being unconstitutional, and Resolution No. 92-0829 dated 6 April 1992 and Resolution No. 92-0999 dated 23 April 1992, for want of legal and factual bases.

The factual backdrop: In preparation for the synchronized national and local elections scheduled on 11 May 1992, the Commission on Elections (COMELEC) issued on 11 December 1991 Resolution No. 2323 otherwise referred to as the "Gun Ban," promulgating rules and regulations on bearing, carrying and transporting of firearms or other deadly weapons, on security personnel or bodyguards, on bearing arms by members of security agencies or police organizations, and organization or maintenance of reaction forces during the election period. 1Subsequently, on 26 December 1991 COMELEC issued Resolution No. 2327 providing for the summary disqualification of 

58

Page 59: Search and Seizure Cases

candidates engaged in gunrunning, using and transporting of firearms, organizing special strike forces, and establishing spot checkpoints. 2

On 10 January 1992, pursuant to the "Gun Ban," Mr. Serapio P. Taccad, Sergeant-at-Arms, House of Representatives, wrote petitioner who was then Congressman of the 1st District of Bulacan requesting the return of the two (2) firearms 3 issued to him by the House of Representatives. Upon being advised of the request on 13 January 1992 by his staff, petitioner immediately instructed his driver, Ernesto Arellano, to pick up the firearms from petitioner's house at Valle Verde and return them to Congress.

Meanwhile, at about five o'clock in the afternoon of the same day, the Philippine National Police (PNP) headed by Senior Superintendent Danilo Cordero set up a checkpoint outside the Batasan Complex some twenty (20) meters away from its entrance. About thirty minutes later, the policemen manning the outpost flagged down the car driven by Arellano as it approached the checkpoint. They searched the car and found the firearms neatly packed in their gun cases and placed in a bag in the trunk of the car. Arellano was then apprehended and detained. He explained that he was ordered by petitioner to get the firearms from the house and return them to Sergeant-at-Arms Taccad of the House of Representatives.

Thereafter, the police referred Arellano's case to the Office of the City Prosecutor for inquest. The referral did not include petitioner as among those charged with an election offense. On 15 January 1992, the City Prosecutor ordered the release of Arellano after finding the latter's sworn explanation meritorious. 4

On 28 January 1992, the City Prosecutor invited petitioner to shed light on the circumstances mentioned in Arellano's sworn explanation. Petitioner not only appeared at the preliminary investigation to confirm Arellano's statement but also wrote the City Prosecutor urging him to exonerate Arellano. He explained that Arellano did not violate the firearms ban as he in fact was complying with it when apprehended by returning the firearms to Congress; and, that he was petitioner's driver, not a security officer nor a bodyguard. 5

On 6 March 1992, the Office of the City Prosecutor issued a resolution which, among other matters, recommended that the case against Arellano be dismissed and that the "unofficial" charge against petitioner be also dismissed. 6

Nevertheless, on 6 April 1992, upon recommendation of its Law Department, COMELEC issued Resolution No. 92-0829 directing the filing of information against petitioner and Arellano for violation of Sec. 261, par. (q), of B.P. Blg. 881 otherwise known as the Omnibus Election Code, in relation to Sec. 32 of R.A. No. 7166; 7 and petitioner to show cause why he should not be disqualified from running for an elective position, pursuant to COMELEC Resolution No. 2327, in relation to Sec. 32, 33 and 35 of R.A. 7166, andSec. 52, par. (c), of B.P. Blg. 881. 8

On 13 April 1992, petitioner moved for reconsideration and to hold in abeyance the administrative proceedings as well as the filing of the information in court. 9 On 23 April 1992, the COMELEC denied petitioner's motion for reconsideration. 10 Hence, this recourse.

Petitioner questions the constitutionality of Resolution No. 2327. He argues that the rules and regulations of an administrative body must respect the limits defined by law; that the Omnibus Election Code provides for the disqualification of any person/candidate from running for or holding a public office, i.e., any person who has either been declared by competent authority as insane or incompetent or has been sentenced by final judgment for subversion, insurrection, rebellion or for any offense for which he has been sentenced to a penalty of more than eighteen months or for a crime involving moral turpitude; that gunrunning, using or transporting firearms or similar weapons and other acts mentioned in the resolution are not within the letter or spirit of the provisions of the Code; that the resolution did away with the requirement of final conviction before the commission of certain offenses; that instead, it created a presumption of guilt as a candidate may be disqualified from office in situations (a) where the criminal charge is still pending, (b) where there is no pending criminal case, and (c) where the accused has already been acquitted, all contrary to the requisite quantum of proof for one to be disqualified from running or holding public office under the Omnibus Election Code, i.e., proof beyond reasonable doubt. As a result, petitioner concludes, Resolution No. 2327 violates the fundamental law thus rendering it fatally defective.

But, the issue on the disqualification of petitioner from running in the11 May 1992 synchronized elections was rendered moot when he lost his bid for a seat in Congress in the elections that ensued. Consequently, it is now futile to discuss the implications of the charge against him on his qualification to run for public office.

However, there still remains an important question to be resolved, i.e., whether he can be validly prosecuted for instructing his driver to return to the Sergeant-at-Arms of the House of Representatives the two firearms issued to him on the basis of the evidence gathered from the warrantless search of his car.

Petitioner strongly protests against the manner by which the PNP conducted the search. According to him, without a warrant and without informing the driver of his fundamental rights the policemen searched his car. The firearms were not tucked in the waist nor within the immediate reach of Arellano but were neatly packed in their gun cases and wrapped in a bag kept in the trunk of the car. Thus, the search of his car that yielded the evidence for the prosecution was clearly violative of Secs. 2 and 3, par. (2), Art. III, of the Constitution. 11

Petitioner further maintains that he was neither impleaded as party respondent in the preliminary investigation before the Office of the City Prosecutor nor included in the charge sheet. Consequently, making him a respondent in the criminal information would violate his constitutional right to due process.

Petitioner disputes the charge that he violated Sec. 33 of R.A. 7166, which prohibits any candidate for public office during the election period from employing or availing himself or engaging the 

59

Page 60: Search and Seizure Cases

services of security personnel or bodyguards since, admittedly, Arellano was not a security officer or bodyguard but a civilian employee assigned to him as driver by the House of Representatives. Specifically, petitioner further argues, Arellano was instructed to return to Congress, as he did, the firearms in compliance with the directive of its Sergeant-at-Arms pursuant to the "Gun Ban," thus, no law was in fact violated. 12

On 25 June 1992, we required COMELEC to file its own comment on the petition 13 upon manifestation of the Solicitor General that it could not take the position of COMELEC and prayed instead to be excused from filing the required comment. 14

COMELEC claims that petitioner is charged with violation of Sec. 261, par. (q), in relation to Sec. 263, of B.P. Blg. 881 which provides that "the principals, accomplices and accessories, as defined in the Revised Penal Code, shall be criminally liable for election offenses." It points out that it was upon petitioner's instruction that Arellano brought the firearms in question outside petitioner's residence, submitting that his right to be heard was not violated as he was invited by the City Prosecutor to explain the circumstances regarding Arellano's possession of the firearms. Petitioner also filed a sworn written explanation about the incident. Finally, COMELEC claims that violation ofthe "Gun Ban" is mala prohibita, hence, the intention of the offender is immaterial. 15

Be that as it may, we find no need to delve into the alleged constitutional infirmity of Resolution No. 2327 since this petition may be resolved without passing upon this particular issue. 16

As a rule, a valid search must be authorized by a search warrant duly issued by an appropriate authority. However, this is not absolute. Aside from a search incident to a lawful arrest, a warrantless search had been upheld in cases of moving vehicles and the seizure of evidence in plain view, 17 as well as the search conducted at police or military checkpoints which we declared are not illegal per se, and stressed that the warrantless search is not violative of the Constitution for as long as the vehicle is neither searched nor its occupants subjected to a body search, and the inspection of the vehicle is merely limited to a visual search. 18

Petitioner contends that the guns were not tucked in Arellano's waist nor placed within his reach, and that they were neatly packed in gun cases and placed inside a bag at the back of the car. Significantly, COMELEC did not rebut this claim. The records do not show that the manner by which the package was bundled led the PNP to suspect that it contained firearms. There was no mention either of any report regarding any nervous, suspicious or unnatural reaction from Arellano when the car was stopped and searched. Given these circumstances and relying on its visual observation, the PNP could not thoroughly search the car lawfully as well as the package without violating the constitutional injunction.

An extensive search without warrant could only be resorted to if the officers conducting the search had reasonable or probable cause to believe before the search that either the motorist was a law offender or that they would find the instrumentality or evidence pertaining to the commission of a crime in the vehicle to be searched. 19 The existence of probable cause justifying the warrantless search is determined by the facts of each case. 20 Thus, we upheld the validity of the warrantless 

search in situations where the smell of marijuana emanated from a plastic bag owned by the accused, or where the accused was acting suspiciously, and attempted to flee. 21

We also recognize the stop-and-search without warrant conducted by police officers on the basis of prior confidential information which were reasonably corroborated by other attendant matters, e.g., where a confidential report that a sizeable volume of marijuana would be transported along the route where the search was conducted and appellants were caught in flagrante delicto transporting drugs at the time of their arrest; 22 where apart from the intelligence information, there were reports by an undercover "deep penetration" agent that appellants were bringing prohibited drugs into the country; 23 where the information that a Caucasian coming from Sagada bringing prohibited drugs was strengthened by the conspicuous bulge in accused's waistline, and his suspicious failure to produce his passport and other identification papers; 24 where the physical appearance of the accused fitted the description given in the confidential information about a woman transporting marijuana; 25 where the accused carrying a bulging black leather bag were suspiciously quiet and nervous when queried about its contents; 26 or where the identity of the drug courier was already established by police authorities who received confidential information about the probable arrival of accused on board one of the vessels arriving in Dumaguete City. 27

In the case at bench, we find that the checkpoint was set up twenty (20) meters from the entrance to the Batasan Complex to enforce ResolutionNo. 2327. There was no evidence to show that the policemen were impelled to do so because of a confidential report leading them to reasonably believe that certain motorists matching the description furnished by their informant were engaged in gunrunning, transporting firearms or in organizing special strike forces. Nor, as adverted to earlier, was there any indication from the package or behavior of Arellano that could have triggered the suspicion of the policemen. Absent such justifying circumstances specifically pointing to the culpability of petitioner and Arellano, the search could not be valid. The action then of the policemen unreasonably intruded into petitioner's privacy and the security of his property, in violation of Sec. 2, Art. III, of the Constitution. Consequently, the firearms obtained in violation of petitioner's right against warrantless search cannot be admitted for any purpose in any proceeding.

It may be argued that the seeming acquiescence of Arellano to the search constitutes an implied waiver of petitioner's right to question the reasonableness of the search of the vehicle and the seizure of the firearms.

While Resolution No. 2327 authorized the setting up of checkpoints, it however stressed that "guidelines shall be made to ensure that no infringement of civil and political rights results from the implementation of this authority," and that "the places and manner of setting up of checkpoints shall be determined in consultation with the Committee on Firearms Ban and Security Personnel created under Sec. 5, Resolution No. 2323." 28 The facts show that PNP installed the checkpoint at about five o'clock in the afternoon of 13 January 1992. The search was made soon thereafter, or thirty minutes later. It was not shown that news of impending checkpoints without necessarily giving their locations, and the reason for the same have been announced in the media to forewarn the citizens. Nor did the informal checkpoint that afternoon carry signs informing the public of the purpose of its operation. As a result, motorists passing that place did not have any inkling 

60

Page 61: Search and Seizure Cases

whatsoever about the reason behind the instant exercise. With the authorities in control to stop and search passing vehicles, the motorists did not have any choice but to submit to the PNP's scrutiny. Otherwise, any attempt to turnabout albeit innocent would raise suspicion and provide probable cause for the police to arrest the motorist and to conduct an extensive search of his vehicle.

In the case of petitioner, only his driver was at the car at that time it was stopped for inspection. As conceded by COMELEC, driver Arellano did not know the purpose of the checkpoint. In the face of fourteen (14) armed policemen conducting the operation, 29 driver Arellano being alone and a mere employee of petitioner could not have marshalled the strength and the courage to protest against the extensive search conducted in the vehicle. In such scenario, the "implied acquiescence," if there was any, could not be more than a mere passive conformity on Arellano's part to the search, and "consent" given under intimidating or coercive circumstances is no consent within the purview of the constitutional guaranty.

Moreover, the manner by which COMELEC proceeded against petitioner runs counter to the due process clause of the Constitution. The facts show that petitioner was not among those charged by the PNP with violation of the Omnibus Election Code. Nor was he subjected by the City Prosecutor to a preliminary investigation for such offense. The non-disclosure by the City Prosecutor to the petitioner that he was a respondent in the preliminary investigation is violative of due process which requires that the procedure established by law should be obeyed. 30

COMELEC argues that petitioner was given the change to be heard because he was invited to enlighten the City Prosecutor regarding the circumstances leading to the arrest of his driver, and that petitioner in fact submitted a sworn letter of explanation regarding the incident. This does not satisfy the requirement of due process the essence of which is the reasonable opportunity to be heard and to submit any evidence one may have in support of his defense. 31 Due process guarantees the observance of both substantive and procedural rights, whatever the source of such rights, be it the Constitution itself or only a statute or a rule of court. 32 In Go v. Court of Appeals, 33 we heldthat —

While the right to preliminary investigation is statutory rather than constitutional in its fundament, since it has in fact been established by statute, it is a component part of due process in criminal justice. The right to have a preliminary investigation conducted before being bound over to trial for a criminal offense and hence formally at risk of incarceration or some other penalty is not a mere formal or technical right; it is a substantive right . . . . [T]he right to an opportunity to avoid a process painful to anyone save, perhaps, to hardened criminals is a valuable right. To deny petitioner's claim to a preliminary investigation would be to deprive him of the full measure of his right to due process.

Apparently, petitioner was merely invited during the preliminary investigation of Arellano to corroborate the latter's explanation. Petitioner then was made to believe that he was not a party respondent in the case, so that his written explanation on the incident was only intended to 

exculpate Arellano, not petitioner himself. Hence, it cannot be seriously contended that petitioner was fully given the opportunity to meet the accusation against him as he was not apprised that he was himself a respondent when he appeared before the City Prosecutor.

Finally, it must be pointed out too that petitioner's filing of a motion for reconsideration with COMELEC cannot be considered as a waiver of his claim to a separate preliminary investigation for himself. The motion itself expresses petitioner's vigorous insistence on his right. Petitioner's protestation started as soon as he learned of his inclusion in the charge, and did not ease up even after COMELEC's denial of his motion for reconsideration. This is understandably so since the prohibition against carrying firearms bears the penalty of imprisonment of not less than one (1) year nor more than six (6) years without probation and with disqualification from holding public office, and deprivation of the right to suffrage. Against such strong stance, petitioner clearly did not waive his right to a preliminary investigation.

WHEREFORE, the instant petition is GRANTED. The warrantless search conducted by the Philippine National Police on 13 January 1992 is declared illegal and the firearms seized during the warrantless search cannot be used as evidence in any proceeding against petitioner. Consequently, COMELEC Resolution No. 92-0829 dated 6 April 1992 being violative of the Constitution is SET ASIDE.

The temporary restraining order we issued on 5 May 1992 is made permanent.

PEOPLE OF THE PHILIPPINES, plaintiff-appellee, vs.IDEL AMINNUDIN y AHNI, defendant-appellant.

The accused-appellant claimed his business was selling watches but he was nonetheless arrested, tried and found guilty of illegally transporting marijuana. The trial court, disbelieving him, held it was high time to put him away and sentenced him to life imprisonment plus a fine of P20,000.00. 1

Idel Aminnudin was arrested on June 25, 1984, shortly after disembarking from the M/V Wilcon 9 at about 8:30 in the evening, in Iloilo City. The PC officers who were in fact waiting for him simply accosted him, inspected his bag and finding what looked liked marijuana leaves took him to their headquarters for investigation. The two bundles of suspect articles were confiscated from him and later taken to the NBI laboratory for examination. When they were verified as marijuana leaves, an information for violation of the Dangerous Drugs Act was filed against him. 2Later, the information was amended to include Farida Ali y Hassen, who had also been arrested with him that same evening and likewise investigated. 3 Both were arraigned and pleaded not guilty. 4 Subsequently, the fiscal filed a motion to dismiss the charge against Ali on the basis of a sworn statement of the arresting officers absolving her after a 'thorough investigation." 5 The motion was granted, and trial proceeded only against the accused-appellant, who was eventually convicted . 6

According to the prosecution, the PC officers had earlier received a tip from one of their informers that the accused-appellant was on board a vessel bound for Iloilo City and was carrying marijuana. 7 He was Identified by name. 8 Acting on this tip, they waited for him in the evening of June 25, 1984, and approached him as he descended from the gangplank after the informer had 

61

Page 62: Search and Seizure Cases

pointed to him. 9 They detained him and inspected the bag he was carrying. It was found to contain three kilos of what were later analyzed as marijuana leaves by an NBI forensic examiner, 10 who testified that she conducted microscopic, chemical and chromatographic tests on them. On the basis of this finding, the corresponding charge was then filed against Aminnudin.

In his defense, Aminnudin disclaimed the marijuana, averring that all he had in his bag was his clothing consisting of a jacket, two shirts and two pairs of pants. 11 He alleged that he was arbitrarily arrested and immediately handcuffed. His bag was confiscated without a search warrant. At the PC headquarters, he was manhandled to force him to admit he was carrying the marijuana, the investigator hitting him with a piece of wood in the chest and arms even as he parried the blows while he was still handcuffed. 12 He insisted he did not even know what marijuana looked like and that his business was selling watches and sometimes cigarettes. 13 He also argued that the marijuana he was alleged to have been carrying was not properly Identified and could have been any of several bundles kept in the stock room of the PC headquarters. 14

The trial court was unconvinced, noting from its own examination of the accused that he claimed to have come to Iloilo City to sell watches but carried only two watches at the time, traveling from Jolo for that purpose and spending P107.00 for fare, not to mention his other expenses. 15 Aminnudin testified that he kept the two watches in a secret pocket below his belt but, strangely, they were not discovered when he was bodily searched by the arresting officers nor were they damaged as a result of his manhandling. 16 He also said he sold one of the watches for P400.00 and gave away the other, although the watches belonged not to him but to his cousin, 17 to a friend whose full name he said did not even know. 18 The trial court also rejected his allegations of maltreatment, observing that he had not sufficiently proved the injuries sustained by him. 19

There is no justification to reverse these factual findings, considering that it was the trial judge who had immediate access to the testimony of the witnesses and had the opportunity to weigh their credibility on the stand. Nuances of tone or voice, meaningful pauses and hesitation, flush of face and dart of eyes, which may reveal the truth or expose the lie, are not described in the impersonal record. But the trial judge sees all of this, discovering for himself the truant fact amidst the falsities.

The only exception we may make in this case is the trial court's conclusion that the accused-appellant was not really beaten up because he did not complain about it later nor did he submit to a medical examination. That is hardly fair or realistic. It is possible Aminnudin never had that opportunity as he was at that time under detention by the PC authorities and in fact has never been set free since he was arrested in 1984 and up to the present. No bail has been allowed for his release.

There is one point that deserves closer examination, however, and it is Aminnudin's claim that he was arrested and searched without warrant, making the marijuana allegedly found in his possession inadmissible in evidence against him under the Bill of Rights. The decision did not even discuss this point. For his part, the Solicitor General dismissed this after an all-too-short argument that the arrest of Aminnudin was valid because it came under Rule 113, Section 6(b) of the Rules of Court on warrantless arrests. This made the search also valid as incidental to a lawful arrest.

It is not disputed, and in fact it is admitted by the PC officers who testified for the prosecution, that they had no warrant when they arrested Aminnudin and seized the bag he was carrying. Their only justification was the tip they had earlier received from a reliable and regular informer who reported to them that Aminnudin was arriving in Iloilo by boat with marijuana. Their testimony varies as to the time they received the tip, one saying it was two days before the arrest, 20 another two weeks 21 and a third "weeks before June 25." 22 On this matter, we may prefer the declaration of the chief of the arresting team, Lt. Cipriano Querol, Jr., who testified as follows:

Q You mentioned an intelligence report, you mean with respect to the coming of Idel Aminnudin on June 25, 1984?

A Yes, sir.Q When did you receive this intelligence report?A Two days before June 25, 1984 and it was supported by reliable sources.Q Were you informed of the coming of the Wilcon 9 and the possible trafficking of marijuana leaves on that date?A Yes, sir, two days before June 25, 1984 when we received this information from that particular informer, prior to June 25, 1984 we have already reports of the particular operation which was being participated by Idel Aminnudin.Q You said you received an intelligence report two days before June 25, 1984 with respect to the coming of Wilcon 9?A Yes, sir.Q Did you receive any other report aside from this intelligence report?A Well, I have received also other reports but not pertaining to the coming of Wilcon 9. For instance, report of illegal gambling operation.COURT:Q Previous to that particular information which you said two days before June 25, 1984, did you also receive daily report regarding the activities of Idel AminnudinA Previous to June 25, 1984 we received reports on the activities of Idel Aminnudin.Q What were those activities?A Purely marijuana trafficking.Q From whom did you get that information?A It came to my hand which was written in a required sheet of information, maybe for security reason and we cannot Identify the person.Q But you received it from your regular informer?A Yes, sir.ATTY. LLARIZA:Q Previous to June 25, 1984, you were more or less sure that Idel Aminnudin is coming with drugs?A Marijuana, sir.Q And this information respecting Idel Aminnudin's coming to Iloilo with marijuana was received by you many days before you received the intelligence report in writing?A Not a report of the particular coming of Aminnudin but his activities.Q You only knew that he was coming on June 25,1984 two days before?A Yes, sir.Q You mean that before June 23, 1984 you did not know that minnudin was coming?

62

Page 63: Search and Seizure Cases

A Before June 23,1984, I, in my capacity, did not know that he was coming but on June 23, 1984 that was the time when I received the information that he was coming. Regarding the reports on his activities, we have reports that he was already consummated the act of selling and shipping marijuana stuff.COURT:Q And as a result of that report, you put him under surveillance?A Yes, sir.Q In the intelligence report, only the name of Idel Aminnudin was mentioned?A Yes, sir.Q Are you sure of that?A On the 23rd he will be coming with the woman.Q So that even before you received the official report on June 23, 1984, you had already gathered information to the effect that Idel Aminnudin was coming to Iloilo on June 25, 1984?A Only on the 23rd of June.Q You did not try to secure a search warrant for the seizure or search of the subject mentioned in your intelligence report?A No, more.Q Why not?A Because we were very very sure that our operation will yield positive result.Q Is that your procedure that whenever it will yield positive result you do not need a search warrant anymore?

A Search warrant is not necessary. 23

That last answer is a cavalier pronouncement, especially as it comes from a mere lieutenant of the PC. The Supreme Court cannot countenance such a statement. This is still a government of laws and not of men.

The mandate of the Bill of Rights is clear:

Sec. 2. The right of the people to be secure in their persons, houses, papers and effects against unreasonable searches and seizures of whatever nature and for any purpose shall be inviolable, and no search warrant or warrant of arrest shall issue except upon probable cause to be determined personally by the judge after examination under oath or affirmation of the complainant and the witnesses he may produce, and particularly describing the place to be searched and the persons or things to be seized.

In the case at bar, there was no warrant of arrest or search warrant issued by a judge after personal determination by him of the existence of probable cause. Contrary to the averments of the government, the accused-appellant was not caught in flagrante nor was a crime about to be committed or had just been committed to justify the warrantless arrest allowed under Rule 113 of the Rules of Court. Even expediency could not be invoked to dispense with the obtention of the warrant as in the case of Roldan v. Arca, 24 for example. Here it was held that vessels and aircraft are 

subject to warrantless searches and seizures for violation of the customs law because these vehicles may be quickly moved out of the locality or jurisdiction before the warrant can be secured.

The present case presented no such urgency. From the conflicting declarations of the PC witnesses, it is clear that they had at least two days within which they could have obtained a warrant to arrest and search Aminnudin who was coming to Iloilo on the M/V Wilcon 9. His name was known. The vehicle was Identified. The date of its arrival was certain. And from the information they had received, they could have persuaded a judge that there was probable cause, indeed, to justify the issuance of a warrant. Yet they did nothing. No effort was made to comply with the law. The Bill of Rights was ignored altogether because the PC lieutenant who was the head of the arresting team, had determined on his own authority that a "search warrant was not necessary."

In the many cases where this Court has sustained the warrantless arrest of violators of the Dangerous Drugs Act, it has always been shown that they were caught red-handed, as a result of what are popularly called "buy-bust" operations of the narcotics agents. 25 Rule 113 was clearly applicable because at the precise time of arrest the accused was in the act of selling the prohibited drug.

In the case at bar, the accused-appellant was not, at the moment of his arrest, committing a crime nor was it shown that he was about to do so or that he had just done so. What he was doing was descending the gangplank of the M/V Wilcon 9 and there was no outward indication that called for his arrest. To all appearances, he was like any of the other passengers innocently disembarking from the vessel. It was only when the informer pointed to him as the carrier of the marijuana that he suddenly became suspect and so subject to apprehension. It was the furtive finger that triggered his arrest. The Identification by the informer was the probable cause as determined by the officers (and not a judge) that authorized them to pounce upon Aminnudin and immediately arrest him.

Now that we have succeeded in restoring democracy in our country after fourteen years of the despised dictatorship, when any one could be picked up at will, detained without charges and punished without trial, we will have only ourselves to blame if that kind of arbitrariness is allowed to return, to once more flaunt its disdain of the Constitution and the individual liberties its Bill of Rights guarantees.

While this is not to say that the accused-appellant is innocent, for indeed his very own words suggest that he is lying, that fact alone does not justify a finding that he is guilty. The constitutional presumption is that he is innocent, and he will be so declared even if his defense is weak as long as the prosecution is not strong enough to convict him.

Without the evidence of the marijuana allegedly seized from Aminnudin, the case of the prosecution must fall. That evidence cannot be admitted, and should never have been considered by the trial court for the simple fact is that the marijuana was seized illegally. It is the fruit of the poisonous tree, to use Justice Holmes' felicitous phrase. The search was not an incident of a lawful arrest because there was no warrant of arrest and the warrantless arrest did not come under the 

63

Page 64: Search and Seizure Cases

exceptions allowed by the Rules of Court. Hence, the warrantless search was also illegal and the evidence obtained thereby was inadmissible.

The Court strongly supports the campaign of the government against drug addiction and commends the efforts of our law-enforcement officers against those who would inflict this malediction upon our people, especially the susceptible youth. But as demanding as this campaign may be, it cannot be more so than the compulsions of the Bill of Rights for the protection of the liberty of every individual in the realm, including the basest of criminals. The Constitution covers with the mantle of its protection the innocent and the guilty alike against any manner of high- handedness from the authorities, however praiseworthy their intentions.

Those who are supposed to enforce the law are not justified in disregarding the rights of the individual in the name of order. Order is too high a price for the loss of liberty. As Justice Holmes, again, said, "I think it a less evil that some criminals should escape than that the government should play an ignoble part." It is simply not allowed in the free society to violate a law to enforce another, especially if the law violated is the Constitution itself.

We find that with the exclusion of the illegally seized marijuana as evidence against the accused-appellant, his guilt has not been proved beyond reasonable doubt and he must therefore be discharged on the presumption that he is innocent.

ACCORDINGLY, the decision of the trial court is REVERSED and the accused-appellant is ACQUITTED. It is so ordered.

HORTENCIA SALAZAR, petitioner, vs.HON. TOMAS D. ACHACOSO, in his capacity as Administrator of the Philippine Overseas Employment Administration, and FERDIE MARQUEZ, respondents.

This concerns the validity of the power of the Secretary of Labor to issue warrants of arrest and seizure under Article 38 of the Labor Code, prohibiting illegal recruitment.

The facts are as follows:

1. On October 21, 1987, Rosalie Tesoro of 177 Tupaz Street, Leveriza, Pasay City, in a sworn statement filed with the Philippine Overseas Employment Administration (POEA for brevity) charged petitioner Hortencia Salazar, viz:

04. T: Ano ba ang dahilan at ikaw ngayon ay narito atnagbibigay ng salaysay.

S: Upang ireklamo sa dahilan ang aking PECC Card ay ayaw ibigay sa akin ng dati kong manager. — Horty Salazar — 615 R.O. Santos, Mandaluyong, Mla.

05. T: Kailan at saan naganap and ginawang panloloko sa iyo ng tao/mga taong inireklamo mo?

S. Sa bahay ni Horty Salazar.

06. T: Paano naman naganap ang pangyayari?

S. Pagkagaling ko sa Japan ipinatawag niya ako. Kinuha ang PECC Card ko at sinabing hahanapan ako ng booking sa Japan. Mag 9 month's na ako sa Phils. ayhindi pa niya ako napa-alis. So lumipat ako ng ibang company pero ayaw niyang ibigay and PECC Card ko.

2. On November 3, 1987, public respondent Atty. Ferdinand Marquez to whom said complaint was assigned, sent to the petitioner the following telegram:

YOU ARE HEREBY DIRECTED TO APPEAR BEFORE FERDIE MARQUEZ POEA ANTI ILLEGAL RECRUITMENT UNIT 6TH FLR. POEA BLDG. EDSA COR. ORTIGAS AVE. MANDALUYONG MM ON NOVEMBER 6, 1987 AT 10 AM RE CASE FILED AGAINST YOU. FAIL NOT UNDER PENALTY OF LAW.

4. On the same day, having ascertained that the petitioner had no license to operate a recruitment agency, public respondent Administrator Tomas D. Achacoso issued his challenged CLOSURE AND SEIZURE ORDER NO. 1205 which reads:

HORTY SALAZARNo. 615 R.O. Santos St.Mandaluyong, Metro Manila

Pursuant to the powers vested in me under Presidential Decree No. 1920 and Executive Order No. 1022, I hereby order the CLOSURE of your recruitment agency being operated at No. 615 R.O. Santos St., Mandaluyong, Metro Manila and the seizure of the documents and paraphernalia being used or intended to be used as the means of committing illegal recruitment, it having verified that you have —

(1) No valid license or authority from the Department of Labor and Employment to recruit and deploy workers for overseas employment;

(2) Committed/are committing acts prohibited under Article 34 of the New Labor Code in relation to Article 38 of the same code.

This ORDER is without prejudice to your criminal prosecution under existing laws.

64

Page 65: Search and Seizure Cases

Done in the City of Manila, this 3th day of November, 1987.

5. On January 26, 1988 POEA Director on Licensing and Regulation Atty. Estelita B. Espiritu issued an office order designating respondents Atty. Marquez, Atty. Jovencio Abara and Atty. Ernesto Vistro as members of a team tasked to implement Closure and Seizure Order No. 1205. Doing so, the group assisted by Mandaluyong policemen and mediamen Lito Castillo of the People's Journal and Ernie Baluyot of News Today proceeded to the residence of the petitioner at 615 R.O. Santos St., Mandaluyong, Metro Manila. There it was found that petitioner was operating Hannalie Dance Studio. Before entering the place, the team served said Closure and Seizure order on a certain Mrs. Flora Salazar who voluntarily allowed them entry into the premises. Mrs. Flora Salazar informed the team that Hannalie Dance Studio was accredited with Moreman Development (Phil.). However, when required to show credentials, she was unable to produce any. Inside the studio, the team chanced upon twelve talent performers — practicing a dance number and saw about twenty more waiting outside, The team confiscated assorted costumes which were duly receipted for by Mrs. Asuncion Maguelan and witnessed by Mrs. Flora Salazar.

6. On January 28, 1988, petitioner filed with POEA the following letter:

Gentlemen:On behalf of Ms. Horty Salazar of 615 R.O. Santos, Mandaluyong, Metro Manila, we respectfully request that the personal properties seized at her residence last January 26, 1988 be immediately returned on the ground that said seizure was contrary to law and against the will of the owner thereof. Among our reasons are the following:

1. Our client has not been given any prior notice or hearing, hence the Closure and Seizure Order No. 1205 dated November 3, 1987 violates "due process of law" guaranteed under Sec. 1, Art. III, of the Philippine Constitution.

2. Your acts also violate Sec. 2, Art. III of the Philippine Constitution which guarantees right of the people "to be secure in their persons, houses, papers, and effects against unreasonable searches and seizures of whatever nature and for any purpose."

3. The premises invaded by your Mr. Ferdi Marquez and five (5) others (including 2 policemen) are the private residence of the Salazar family, and the entry, search as well as the seizure of the personal properties belonging to our client were without her consent and were done with unreasonable force and intimidation, together with grave abuse of the color of authority, and constitute robbery and violation of domicile under Arts. 293 and 128 of the Revised Penal Code.

Unless said personal properties worth around TEN THOUSAND PESOS (P10,000.00) in all (and which were already due for shipment to Japan) are returned within twenty-four (24) hours from your receipt hereof, we shall feel free to take all legal action, civil and criminal, to protect our client's interests.

We trust that you will give due attention to these important matters.

7. On February 2, 1988, before POEA could answer the letter, petitioner filed the instant petition; on even date, POEA filed a criminal complaint against her with the Pasig Provincial Fiscal, docketed as IS-88-836. 1

On February 2, 1988, the petitioner filed this suit for prohibition. Although the acts sought to be barred are alreadyfait accompli, thereby making prohibition too late, we consider the petition as one for certiorari in view of the grave public interest involved.

The Court finds that a lone issue confronts it: May the Philippine Overseas Employment Administration (or the Secretary of Labor) validly issue warrants of search and seizure (or arrest) under Article 38 of the Labor Code? It is also an issue squarely raised by the petitioner for the Court's resolution.

Under the new Constitution, which states:. . . no search warrant or warrant of arrest shall issue except upon probable cause to be determined personally by the judge after examination under oath or affirmation of the complainant and the witnesses he may produce, and particularly describing the place to be searched and the persons or things to be seized. 2

it is only a judge who may issue warrants of search and arrest. 3 In one case, it was declared that mayors may not exercise this power:

But it must be emphasized here and now that what has just been described is the state of the law as it was in September, 1985. The law has since been altered. No longer does the mayor have at this time the power to conduct preliminary investigations, much less issue orders of arrest. Section 143 of the Local Government Code, conferring this power on the mayor has been abrogated, rendered functus officio by the 1987 Constitution which took effect on February 2, 1987, the date of its ratification by the Filipino people. Section 2, Article III of the 1987 Constitution pertinently provides that "no search warrant or warrant of arrest shall issue except upon probable cause to be determined personally by the judge after examination under oath or affirmation of the complainant and the witnesses he may produce, and particularly describing the place to be searched and the person or things to be seized." The constitutional proscription has thereby been manifested that thenceforth, the function of determining probable cause and issuing, on the basis thereof, warrants of arrest or search warrants, may be validly exercised only by judges, this being evidenced by the elimination in the present Constitution of the phrase, "such other responsible officer as may be authorized by law" found in the counterpart provision of said 1973 Constitution, who, aside from judges, might conduct preliminary investigations and issue warrants of arrest or search warrants. 4

Neither may it be done by a mere prosecuting body:

We agree that the Presidential Anti-Dollar Salting Task Force exercises, or was meant to exercise, prosecutorial powers, and on that ground, it cannot be said to be a neutral and detached "judge" to determine the existence of probable cause for purposes of arrest or search. Unlike a magistrate, a prosecutor is naturally interested in the success of his case. Although his office "is to see that justice is done and not necessarily to secure the conviction of the person accused," he stands, invariably, as 

65

Page 66: Search and Seizure Cases

the accused's adversary and his accuser. To permit him to issue search warrants and indeed, warrants of arrest, is to make him both judge and jury in his own right, when he is neither. That makes, to our mind and to that extent, Presidential Decree No. 1936 as amended by Presidential Decree No. 2002, unconstitutional. 5

Section 38, paragraph (c), of the Labor Code, as now written, was entered as an amendment by Presidential Decrees Nos. 1920 and 2018 of the late President Ferdinand Marcos, to Presidential Decree No. 1693, in the exercise of his legislative powers under Amendment No. 6 of the 1973 Constitution. Under the latter, the then Minister of Labor merely exercised recommendatory powers:

(c) The Minister of Labor or his duly authorized representative shall have the power to recommend the arrest and detention of any person engaged in illegal recruitment. 6

On May 1, 1984, Mr. Marcos promulgated Presidential Decree No. 1920, with the avowed purpose of giving more teeth to the campaign against illegal recruitment. The Decree gave the Minister of Labor arrest and closure powers:

(b) The Minister of Labor and Employment shall have the power to cause the arrest and detention of such non-licensee or non-holder of authority if after proper investigation it is determined that his activities constitute a danger to national security and public order or will lead to further exploitation of job-seekers. The Minister shall order the closure of companies, establishment and entities found to be engaged in the recruitment of workers for overseas employment, without having been licensed or authorized to do so. 7

On January 26, 1986, he, Mr. Marcos, promulgated Presidential Decree No. 2018, giving the Labor Minister search and seizure powers as well:

(c) The Minister of Labor and Employment or his duly authorized representatives shall have the power to cause the arrest and detention of such non-licensee or non-holder of authority if after investigation it is determined that his activities constitute a danger to national security and public order or will lead to further exploitation of job-seekers. The Minister shall order the search of the office or premises and seizure of documents, paraphernalia, properties and other implements used in illegal recruitment activities and the closure of companies, establishment and entities found to be engaged in the recruitment of workers for overseas employment, without having been licensed or authorized to do so. 8

The above has now been etched as Article 38, paragraph (c) of the Labor Code.

The decrees in question, it is well to note, stand as the dying vestiges of authoritarian rule in its twilight moments.

We reiterate that the Secretary of Labor, not being a judge, may no longer issue search or arrest warrants. Hence, the authorities must go through the judicial process. To that extent, we declare Article 38, paragraph (c), of the Labor Code, unconstitutional and of no force and effect.

The Solicitor General's reliance on the case of Morano v. Vivo 9 is not well-taken. Vivo involved a deportation case, governed by Section 69 of the defunct Revised Administrative Code and by Section 37 of the Immigration Law. We have ruled that in deportation cases, an arrest (of an undesirable alien) ordered by the President or his duly authorized representatives, in order to carry out a final decision of deportation is valid. 10 It is valid, however, because of the recognized supremacy of the Executive in matters involving foreign affairs. We have held: 11

The State has the inherent power to deport undesirable aliens (Chuoco Tiaco vs. Forbes, 228 U.S. 549, 57 L. Ed. 960, 40 Phil. 1122, 1125). That power may be exercised by the Chief Executive "when he deems such action necessary for the peace and domestic tranquility of the nation." Justice Johnson's opinion is that when the Chief Executive finds that there are aliens whose continued presence in the country is injurious to the public interest, "he may, even in the absence of express law, deport them". (Forbes vs. Chuoco Tiaco and Crossfield, 16 Phil. 534, 568, 569; In re McCulloch Dick, 38 Phil. 41).

The right of a country to expel or deport aliens because their continued presence is detrimental to public welfare is absolute and unqualified (Tiu Chun Hai and Go Tam vs. Commissioner of Immigration and the Director of NBI, 104 Phil. 949, 956). 12

The power of the President to order the arrest of aliens for deportation is, obviously, exceptional. It (the power to order arrests) can not be made to extend to other cases, like the one at bar. Under the Constitution, it is the sole domain of the courts.

Moreover, the search and seizure order in question, assuming, ex gratia argumenti, that it was validly issued, is clearly in the nature of a general warrant:

Pursuant to the powers vested in me under Presidential Decree No. 1920 and Executive Order No. 1022, I hereby order the CLOSURE of your recruitment agency being operated at No. 615 R.O. Santos St., Mandaluyong, Metro Manila and the seizure of the documents and paraphernalia being used or intended to be used as the means of committing illegal recruitment, it having verified that you have —

(1) No valid license or authority from the Department of Labor and Employment to recruit and deploy workers for overseas employment;

(2) Committed/are committing acts prohibited under Article 34 of the New Labor Code in relation to Article 38 of the same code.

This ORDER is without prejudice to your criminal prosecution under existing laws. 13

66

Page 67: Search and Seizure Cases

We have held that a warrant must identify clearly the things to be seized, otherwise, it is null and void, thus:

Another factor which makes the search warrants under consideration constitutionally objectionable is that they are in the nature of general warrants. The search warrants describe the articles sought to be seized in this wise:

1) All printing equipment, paraphernalia, paper, ink, photo equipment, typewriters, cabinets, tables, communications/ recording equipment, tape recorders, dictaphone and the like used and/or connected in the printing of the "WE FORUM" newspaper and any and all documents/communications, letters and facsimile of prints related to the "WE FORUM" newspaper.

2) Subversive documents, pamphlets, leaflets, books, and other publications to promote the objectives and purposes of the subversive organizations known as Movement for Free Philippines, Light-a-Fire Movement and April 6 Movement; and

3) Motor vehicles used in the distribution/circulation of the "WE FORUM" and other subversive materials and propaganda, more particularly,

1) Toyota-Corolla, colored yellow with Plate No. NKA 892;

2) DATSUN, pick-up colored white with Plate No. NKV 969;

3) A delivery truck with Plate No. NBS 542;

4) TOYOTA-TAMARAW, colored white with Plate No. PBP 665; and

5) TOYOTA Hi-Lux, pick-up truck with Plate No. NGV 472 with marking "Bagong Silang."

In Stanford v. State of Texas, the search warrant which authorized the search for "books, records, pamphlets, cards, receipts, lists, memoranda, pictures, recordings and other written instruments concerning the Communist Parties of Texas, and the operations of the Community Party in Texas," was declared void by the U.S. Supreme Court for being too general. In like manner, directions to "seize any evidence in connection with the violation of SDC 13-3703 or otherwise" have been held too general, and that portion of a search warrant which authorized the seizure of any "paraphernalia which could be used to violate Sec. 54-197 of the Connecticut General Statutes (the statute dealing with the crime of conspiracy)" was held to be a general warrant, and therefore invalid. The description of the articles sought to be seized under the search warrants in question cannot be characterized differently.

In the Stanford case, the U.S. Supreme court calls to mind a notable chapter in English history; the era of disaccord between the Tudor Government and the English Press, when "Officers of the Crown were given roving commissions to search where they pleased in order to suppress and destroy the 

literature of dissent both Catholic and Puritan." Reference herein to such historical episode would not be relevant for it is not the policy of our government to suppress any newspaper or publication that speaks with "the voice of non-conformity" but poses no clear and imminent danger to state security. 14

For the guidance of the bench and the bar, we reaffirm the following principles:

1. Under Article III, Section 2, of the l987 Constitution, it is only judges, and no other, who may issue warrants of arrest and search:

2. The exception is in cases of deportation of illegal and undesirable aliens, whom the President or the Commissioner of Immigration may order arrested, following a final order of deportation, for the purpose of deportation.

WHEREFORE, the petition is GRANTED. Article 38, paragraph (c) of the Labor Code is declared UNCONSTITUTIONAL and null and void. The respondents are ORDERED to return all materials seized as a result of the implementation of Search and Seizure Order No. 1205.

67